Patho Final

Ace your homework & exams now with Quizwiz!

16. Which of the following phenomena contributes to the difficulties with absorption, distribution, and elimination of drugs that are associated with kidney disease? A) Reductions in plasma proteins increase the amount of free drug and decrease the amount of protein-bound drug. B) Acute tubular necrosis is associated with impaired drug reabsorption through the tubular epithelium. C) Decreased retention by the kidneys often renders normal drug dosages ineffective. D) Dialysis removes active metabolites from circulation minimizing therapeutic effect.

Ans: A Feedback: A decrease in plasma proteins, particularly albumin, that occurs in many persons with renal failure results in less protein-bound drug and greater amounts of free drug. Drug elimination problems do not stem as directly from impaired tubular reabsorption, decreased retention, or the process of dialysis.

20. The nurse knows high incidences of infectious illnesses among the older adults who reside in a long-term care facility are most likely to have diminished immune capacity because of A) decreased numbers and responsiveness of T lymphocytes. B) decreased antigen recognition by B lymphocytes. C) overexpression of cytokines and receptors. D) altered function in peripheral lymphocytes.

Ans: A Feedback: Although this phenomenon is not well understood, increasing proportions of lymphocytes become unresponsive with age, and CD4+ T lymphocytes are the most severely affected. B lymphocytes recognize more antigens, not fewer, and expression of cytokines and their cellular receptors decreases.

17. Which of the following clients in a hospital medical unit is most clearly demonstrating the signs and symptoms of liver failure? An adult with A) low hemoglobin levels, low platelet levels, and spider angiomas present. B) blood pressure of 189/103, jaundice, and multiple thromboses. C) sudden onset of confusion, a history of alcohol abuse, and low levels of serum AST and ALT. D) ascites, fever, and recent onset of atrial fibrillation.

Ans: A Feedback: Anemia, thrombocytopenia, and the presence of spider angiomas are characteristic of liver failure. High blood pressure, excessive clotting, fever, and cardiac arrhythmias are not common symptoms of liver failure, and AST and ALT levels would rise, not fall.

18. All antiretroviral medications interfere with some stage of the HIV life cycle. What stage do protease inhibitors prevent? A) Cleavage of the polyprotein chain into the individual proteins that will be used to make new virus B) Addition of more nucleosides to the DNA chain C) Killing of the CD4+ T cell to release virions into the bloodstream D) Attachment of the virus to CD4+ cell receptors

Ans: A Feedback: By binding to the protease enzyme and inhibiting its function, protease inhibitors prevent cleavage of the polyprotein chain into individual proteins. Virions are still released into the body, but they are immature and noninfectious.

11. A 42-year-old female client with a long-standing history of chronic nausea and vomiting but a near-insatiable appetite has had her symptoms attributed to an enzyme deficiency. Further diagnostic testing indicates that she has inadequate pancreatic enzyme levels and that her large appetite is due to a lack of enzyme control of food intake inhibition. In which of the following enzymes is the woman most likely deficient? A) Cholecystokinin B) Ghrelin C) Gastrin D) Secretin

Ans: A Feedback: Cholecystokinin is responsible for inhibiting food intake as well as stimulating pancreatic enzyme secretion. Ghrelin stimulates food intake, while gastrin stimulates gastric acid production, and secretin inhibits it.

13. Which of the following medications used in the treatment of peptic ulcers and gastroesophageal reflux binds to H2 receptors and blocks the action of histamine on parietal cells? A) Cimetidine (Tagamet) B) Levbid (hyoscyamine) C) Lotronex (alosetron) D) Nexium (esomeprazole)

Ans: A Feedback: Cimetidine is a H2 receptor blocker used to treat peptic ulcers and GERD. It binds to H2 receptors and blocks the action of histamine on parietal cells. Levbid and Lotronex are anticholinergics, while Nexium is a proton pump inhibitor that inhibits gastric acid secretion.

11. Which of the following questions is most likely to be clinically useful in the differential diagnosis of sensorineural versus conductive hearing loss? A) ìWhat medications do you currently take?î B) ìWhat effect is this hearing loss having on your quality of life?î C) ìHas your hearing loss developed quickly or more slowly?î D) ìDo you ever hear a persistent ringing in your ears?î

Ans: A Feedback: Conductive hearing loss occurs when auditory stimuli are not adequately transmitted through the auditory canal, tympanic membrane, middle ear, or ossicle chain to the inner ear. It can be a temporary loss from impacted cerumen. Sensorineural hearing loss occurs with disorders that affect the inner ear, auditory nerve, or auditory pathways to the brain. Numerous drugs have ototoxic potential, a consequence of which is sensorineural hearing loss. The onset and course of hearing loss and the presence or absence of tinnitus do not necessarily help to differentiate between conductive and sensorineural hearing loss. The subjective effect of the client's hearing loss, while a valid concern, does not help with the differential diagnosis.

7. A 68-year-old woman with a new onset of vascular dementia has recently begun retaining urine. Which of the following physiological phenomena would her care providers most realistically expect to currently occur as a result of her urinary retention? A) Hypertrophy of the bladder muscle and increased bladder wall thickness B) Decreased urine production and nitrogenous waste excretion by the kidneys C) Decompensation, bladder stretching, and high residual urine volume D) Overflow incontinence and loss of contraction power

Ans: A Feedback: Early accompaniments to urinary retention include hypertrophy of the bladder muscle and increased thickness of the bladder wall. Renal effects are unlikely, and decompensation and loss of contraction power are most often later rather than early effects.

16. A 4-year-old boy who has been deaf since birth and has bilateral cataracts has been brought to the emergency department by his mother because she noticed blood in the toilet after he last voided. Urinalysis confirms heavy microscopic hematuria as well as proteinuria. What will the health care team's initial differential diagnosis most likely be? A) Alport syndrome B) Systemic lupus erythematosus glomerulonephritis C) Henoch-Schˆnlein purpura nephritis D) Immunoglobulin A nephropathy

Ans: A Feedback: Heavy microscopic hematuria, proteinuria, and sensorineural deafness and eye disorders are characteristic of Alport syndrome. This symptomatology is less characteristic of systemic lupus erythematosus glomerulonephritis, Henoch-Schˆnlein purpura nephritis, or immunoglobulin A nephropathy.

6. A deficiency in which of the following would result in an inhibition of the inflammatory response? A) Histamine B) Helper T cells C) B cells D) Vitamin K

Ans: A Feedback: Histamine is a key mediator in the inflammatory system, unlike helper T cells, B cells, or vitamin K.

18. Of the following list of patients, who would likely benefit the most from hyperbaric oxygen therapy? A) A trauma patient who developed Clostridium spp., an anaerobic bacterial infection in his femur B) A patient who developed a fistula between her bowel and vagina following cervical cancer surgery C) A school-aged child who fell on gravel and has terrible road rash D) A football player who has torn a meniscus in his knee for the third time this year

Ans: A Feedback: Hyperbaric oxygen is a treatment that has demonstrated improved wound healing in multiple types of injuries. It enhances wound healing by a number of mechanisms, including the increased killing of bacteria by neutrophils, impaired growth of anaerobic bacteria, and the promotion of angiogenesis.

4. Which of the following individual situations listed below best exemplifies the processes of innate immunity? A) A child who has experienced heat and swelling of his skinned knuckle B) An adult who complains of itching and is sneezing because he is allergic to pollen C) A client whose blood work indicates increased antibody titers during an acute illness D) A client who has experienced rejection of a donor liver after transplantation

Ans: A Feedback: In a skinned knuckle, one of the body's main innate defenses, the skin, is breached. The heat and swelling that accompany a breach in the skin are inflammatory responses, part of the body's innate immune defenses. Allergies are an inappropriate adaptive response mediated by immunoglobulin E; antibody titers increase during illness in response to the infection; and transplanted organs are rejected because the organ is recognized as foreign. These are all aspects of specific, acquired immunity.

15. A school nurse is teaching high school students about HIV and AIDS in the context of the school's sexual health curriculum. Which of the students' following statements would the nurse most likely want to correct or clarify? A) ìThey have to take a blood sample from you in order to test you for AIDS.î B) ìDrugs for AIDS reduce the virus in your body, but they don't get rid of it.î C) ìLots more heterosexual people get HIV these days than they used to.î D) ìCondoms provide really good protection from AIDS.î

Ans: A Feedback: Oral tests now exist for preliminary diagnosis of HIV. Medications for AIDS do not cure the disease, and incidence is increasing among heterosexuals. Condoms provide effective protection from the virus.

7. A 29-year-old woman has been diagnosed with otosclerosis after several years of progressive hearing loss. What pathophysiological process has characterized her diagnosis? A) New spongy bone has been formed around her stapes and oval window. B) Her incus, malleus, and stapes have become disconnected from her normal neural pathways. C) Her temporal bone is experiencing unusually rapid resorption. D) Her tympanic cavity is becoming filled with bone due to inappropriate osteogenesis.

Ans: A Feedback: Otosclerosis begins with resorption of bone in one or more foci. During active bone resorption, the bone structure appears spongy and softer than normal (i.e., osteospongiosis). The resorbed bone is replaced by an overgrowth of new, hard, sclerotic bone. Distortion of neural pathways, resorption of the temporal bone, and filling of the tympanic cavity do not occur with otosclerosis.

19. After several months on a waiting list, a 44-year-old male received a liver transplant 5 days ago. In the last 36 hours, he has developed a rash beginning on his palms and soles, along with abdominal pain and nausea. It has been determined by his care team that the immune response that is causing his symptoms originates not with his own compromised immune components but with those introduced with his new organ. This man's most likely medical diagnosis is A) graft versus host disease (GVHD). B) acute transplant rejection. C) hyperacute organ rejection. D) T-cellñmediated graft rejection.

Ans: A Feedback: Rash, gastrointestinal involvement, and pernicious activity by donor immune cells are the hallmarks of GVHD. The description does not suggest acute or hyperacute transplant rejection, and T-cellñmediated rejection is not a diagnosis in and of itself, but rather one of the mechanisms of transplant rejection.

6. A patient has just been admitted to a nursing unit with the diagnosis of obstructive jaundice. Which of the following assessment findings would the nurse expect to see in this patient? Select all that apply. A) Clay-colored stools B) Dark urine C) Elevated conjugated bilirubin levels D) Decreased serum alkaline phosphatase levels E) Severe itching

Ans: A, B, C, E Feedback: Obstructive jaundice occurs when bile flow is obstructed between the liver and the intestine. Among the causes are strictures of the bile duct, gallstones, and tumors of the bile duct or the pancreas. Conjugated bilirubin levels usually are elevated; the stools are clay colored; the urine is dark; the levels of serum alkaline phosphatase are markedly elevated; and the aminotransferase levels are slightly increased. The accumulation of bile acids in the blood leads to the development of pruritus (itching).

13. Which of the following signs and diagnostic findings are recognized components of the metabolic and morphologic changes that occur with HIV infection accompanied with lipodystrophy? Select all that apply. A) Hyperlipidemia B) Insulin resistance C) Deficiencies of anterior pituitary hormones D) Increased abdominal girth E) Breast enlargement

Ans: A, B, D, E Feedback: Hyperlipidemia and insulin resistance are aspects of lipodystrophy, a phenomenon that also frequently includes breast enlargement and increased abdominal girth. Pituitary hormone deficiencies are not a noted component of HIV-related metabolic changes.

19. When assessing a patient diagnosed with brain stem ischemia complaining of vertigo, the nurse will likely observe which of the following clinical manifestations? Select all that apply. A) Inability to coordinate voluntary muscular movements B) Difficulty in articulating words C) Feelings of ear fullness D) Deafness E) Facial weakness

Ans: A, B, E Feedback: Inability to coordinate voluntary muscular movements (ataxia), difficulty in articulating words (dysarthria), and facial weakness are usually associated with brain stem ischemia. Fullness in the ear is often a sign of MÈniËre disease. Deafness is usually not associated with brain stem ischemia.

2. A 22-year-old female with a history of intermittent flank pain, repeated UTIs, and hematuria has been diagnosed with autosomal dominant polycystic kidney disease (ADPKD). Which of the following phenomena has most likely contributed to the development of this diagnosis? A) UTIs coupled with an impaired immune response have caused her ADPKD. B) She has inherited a tendency for epithelial cells in her tubules to proliferate inappropriately. C) Severe hypertension and portal hypertension are likely precursors. D) She has inherited undersized kidneys that are prone to calculi formation.

Ans: B Feedback: ADPKD is an inherited condition, and the etiology is thought to involve cysts arising in segments of the renal tubules from a few epithelial cells that proliferate abnormally. UTIs are consequent, not causative, of the condition. Severe hypertension and portal hypertension are more commonly associated with ARPKD than ADPKD. Kidneys are typically oversized in ADPKD, and renal calculi are not noted sequelae.

11. Which of the following aspects of a patient's site of inflammation would help the care provider rule out chronic inflammation? A) High levels of macrophages B) Increased neutrophils C) Proliferation of fibroblasts D) Infiltration of lymphocytes

Ans: B Feedback: Chronic inflammation lacks the sudden and marked proliferation of neutrophils that is associated with acute inflammation. Chronic inflammation is associated with increased presence and action of fibroblasts, macrophages, and lymphocytes.

2. During male ejaculation, which of the following statements addresses why sperm is not normally seen inside the bladder? A) The parasympathetic nervous system keeps the seminal fluid inside the urethra. B) The musculature of the trigone area, bladder neck, and prostatic urethra contract at the same time. C) With ejaculation, the male expels some urine along with the seminal fluid to wash any extra sperm out of the bladder. D) The detrusor muscle relaxes allowing for the closing of the sphincter at the base of the bladder.

Ans: B Feedback: During male ejaculation, which is mediated by the SNS, the musculature of the trigone area and that of the bladder neck and prostatic urethra contracts and prevents the backflow of seminal fluid into the bladder.

8. Four weeks after returning from a tropical vacation, a 40-year-old man has presented to the emergency department with malaise, nausea, and ìyellow eyes.î Serology has confirmed a diagnosis of hepatitis A (HAV), to the shock of the client. What teaching is most appropriate for this client? A) ìYou can expect these symptoms to disappear after about 2 months, but you'll be a carrier of the disease indefinitely.î B) ìA vaccine before your trip would have prevented this, but be assured your body will rid itself of the virus in time.î C) ìYou likely came in contact with blood or body fluids at some point, and you'll have to ensure no one is subsequently exposed to your own blood or body fluids.î D) ìYou likely got this by way of what we call the 'fecalñoral' route; you will have chronic hepatitis now, but the symptoms can be controlled with medication.î

Ans: B Feedback: HAV is normally self-limiting and does not result in chronic hepatitis or carrier status. A vaccine is available, and the fecalñoral route of transmission, rather than contact with blood and body fluids, is typical.

2. A middle-aged male walks into the emergency department complaining of chest pain radiating to the neck, shortness of breath, and nausea. His heart rate is 120 and BP is 94/60. The ED physician recognizes the patient is having an acute MI with decreased cardiac output. The nurse identifies the nausea to be in response to A) the patient not having a very high pain tolerance. B) hypoxia exerting a direct effect on the chemoreceptor trigger zone. C) the patient not having digested his meal completely. D) fear of having to make major lifestyle changes.

Ans: B Feedback: Hypoxia exerts a direct effect on the vomiting center (chemoreceptor trigger zone), producing nausea and vomiting. This direct effect probably accounts for the vomiting that occurs during periods of decreased cardiac output, shock, and environmental hypoxia. We are given no information about the patient's pain tolerance, when he last had a meal, or his routine lifestyle. This patient is going into shock (rapid pulse, low BP) that can result in shunting of blood away from the gut and other organs. During shock, the priority organs for oxygenation include the heart, brain, lungs, and kidneys.

1. The nurse knows which of the following statements listed below relative to a client with malignant melanoma treated with alpha interferon (IFN-a) is accurate? Alpha interferon (IFN-a) A) will kill certain microorganisms that may help spread the cancer. B) plays an important role in the modulation of the inflammatory response. C) helps keep all the blood levels at a higher level. D) controls the migration of leukocytes to their primary site.

Ans: B Feedback: IFNs are cytokines that primarily protect the host against viral infections and play a role in the modulation of the inflammatory response.

16. To reduce hepatic blood flow and decrease portal pressures in persons with cirrhosis, the nurse should be prepared to administer which of the following medications? A) Bevacizumab, an angiogenesis inhibitor B) Octreotide, a long-acting synthetic analog of somatostatin C) Filgrastim, granulocyte colony-stimulating factor (G-CSF) analog D) Diltiazem (Cardizem), a calcium channel blocker

Ans: B Feedback: Ocetrotide, a long-acting synthetic analog of somatostatin, reduces splanchnic and hepatic blood flow and portal pressures in persons with cirrhosis. Bevacizumab was the first angiogenesis inhibitor that was shown to slow tumor growth and, more importantly, to extend the lives of patients with some cancers. Filgrastim rapidly reverses neutropenia and maintains normal ANC in patients with HIV infection. Diltiazem is used to treat a variety of conditions, such as high blood pressure, migraines, and Raynaud disease.

6. A 60-year-old man has been diagnosed with renal calculi after repeated episodes of excruciating flank pain in recent weeks. The man states that, ìI don't know how this could happen to me, since I'm so careful about eating a healthy diet.î What is the most appropriate response to the man's statement? A) ìYour diet may have played a part in this, but in fact, genetics are likely primarily to blame.î B) ìWhat you eat can influence your risk of stone formation, but many other factors like hormones and your metabolism are involved.î C) ìYou likely don't need to change your diet, but now that you have stones in one kidney, you're at very high risk of growing them in the other kidney.î D) ìYour diet might be normally healthy, but high intake of normally beneficial minerals like calcium and magnesium can lead to stones.î

Ans: B Feedback: Renal calculi have a complex etiology that includes diet, but also many other metabolic and endocrine factors, among others. Genetics are not identified as a contributor, and mineral intake is not likely to be the sole factor. Stone formation is normally unilateral.

18. As the eyes rotate upward, the upper eyelid reflexively retracts. Which cranial nerve is primarily responsible for this response? A) Cranial nerve I B) Cranial nerve III C) Cranial nerve VI D) Cranial nerve IV

Ans: B Feedback: The CN III (oculomotor) nucleus, which extends through a considerable part of the midbrain, contains clusters of lower motor neurons for each of the five eye muscles it innervates. Because of its plane of attachment, the inferior oblique rotates the eye in the frontal plane pulling the top of the eye laterally. In other words, as the eyes rotate upward, the upper eyelid is reflexively retracted, and in the downward gaze, it is lowered, restricting exposure of the conjunctiva to air and reducing the effects of drying. CN I is involved in olfactory function; CN VI (abducens nerve) innervates the lateral rectus, which abducts the eye. CN IV (trochlear nerve) innervates the superior oblique muscle, which depresses, rotates laterally, and intorts the eyeball.

20. During a family picnic, a relative of a nurse asks what he should do if there is blood in his urine and some pain in his lower abdomen. The best advice the nurse could give this family member would be for him to A) go to the emergency room right away. B) get an appointment with his family doctor. C) wait and see if it goes away without treatment. D) increase his intake of cranberry juice and other fluids.

Ans: B Feedback: There are many causes of blood in the urine (infection, kidney stones, cancer, etc.). Kidney cancer can be a silent disorder with symptoms occurring late into the disease process. Any blood in the urine should be referred to the physician for further testing (ultrasound, CT scan, MRI, etc.).

18. A middle-aged woman has acromegaly as a result of a pituitary adenoma that was found and removed when she was a teenager. The physician is suspecting that the tumor has returned and has ordered a diagnostic work-up. A glucose load is ordered. If the tumor has returned, the nurse would expect which of the following results? A) The glucoses load will suppress GH level. B) The growth hormone level will not be suppressed following glucose load. C) The glucose load will raise her serum glucose level to the point of requiring insulin. D) There will be no change in the serum growth hormone level following the glucose load.

Ans: B Feedback: When a GH-secreting tumor is suspected, the GH response to a glucose load is measured as part of the diagnostic workup. Normally, a glucose load would suppress GH levels. However, in adults with GH-secreting tumors (acromegaly), GH levels are not suppressed (and paradoxically increase in 50% of cases) to a glucose load.

4. Which of the following patients scheduled for an interventional radiology procedure requiring administration of radiocontrast dye would be considered at high risk for nephrotoxicity? Select all that apply. A) A 14-year-old with severe abdominal pain B) A 25-year-old with a history of glomerular nephritis who is complaining of severe flank pain C) A 67-year-old diabetic undergoing diagnostic testing for new-onset proteinuria D) A 45-year-old with elevated liver enzymes possibly due to fatty liver cirrhosis E) A 53-year-old male undergoing biopsy for a suspicious ìspotî on his chest x-ray

Ans: B, C Feedback: Radiocontrast media-induced nephrotoxicity is thought to result from direct tubular toxicity and renal ischemia. The risk for renal damage caused by radiocontrast media is greatest in older adults and those with preexisting kidney disease, volume depletion, diabetes mellitus, and recent exposure to other nephrotoxic agents.

12. A 24-year-old man is currently in a rehabilitation facility following a spinal cord injury at level T2. He is discussing his long-term options for continence management. Which of the following statements by the client demonstrates he has a clear understanding of the issue? A) ìSelf-catheterization can limit the recovery of my neural pathways that control my voiding if I do it too often.î B) ìIt's critical that intermittent catheterization be performed using sterile technique.î C) ìAn indwelling catheter certainly would work well, but it comes with a number of risks and possible complications.î D) ìAn indwelling urethral catheter is the option that best minimizes my chance of a urinary tract infection.î

Ans: C Feedback: Indwelling catheters carry a risk of infections and kidney stones. Catheterization does not influence the activity of the neural pathways, and intermittent catheterization can be performed using clean technique. Indwelling urethral catheters carry a high risk of urinary tract infections.

18. A 15-year-old boy with type 1 diabetes is exasperated by his regimen of blood sugar monitoring and insulin administration and has told his mother that he wants to scale both back. Which of the following responses by his mother is most accurate? A) ìI know it's not fun, but you're even more susceptible to complications when you're young if you don't stay on top of your diabetes.î B) ìEven though you might save some time and energy by doing this, remember that high blood sugars cause a lot of pain and will cause you to gain weight.î C) ìEven though it's hard to do, you need to continue, so you don't go blind or need a kidney transplant down the road.î D) ìYou need to be vigilant now if you want to be free of diabetes when you grow up.î

Ans: C Feedback: Nephropathies and retinopathies are common complications of poor glycemic control. Complications of diabetes are not noted to be more acute in younger individuals, and hyperglycemia is not normally associated with pain or weight gain. Conscientious diabetes control does not result in a resolution of the disease.

16. A patient with distal symmetric polyneuropathy usually begins to complain of A) burning in the big toe. B) painful cramps in the calves. C) loss of feeling or touch in the feet. D) seeping wounds in lower legs.

Ans: C Feedback: Peripheral neuropathy is often associated with the insensate foot. The loss of feeling, touch, sensation, and position sense increases the risk of falling, serious burns, and injuries to the feet. Burning in the big toe is usually associated with gout. Cramps in the legs may be electrolyte imbalances, and seeping wounds in lower legs could be a result of obesity, lymphedema, or diabetes to name a few.

13. As part of a health promotion initiative, a public health nurse is meeting with a group of older adult residents of an assisted living facility. Which of the following teaching points about hearing loss in the elderly should the nurse include in the teaching session? A) ìIt is actually a myth that seniors have worse and worse hearing as they age.î B) ìMost hearing loss in older adults is the result of easily fixed problems, such as impacted ear wax.î C) ìExperts don't quite know what causes seniors to lose their hearing with age, but drugs like aspirin can contribute to the problem.î D) ìWith older adults, the goal is to adjust lifestyle to accommodate diminished hearing rather than trying to treat the hearing loss itself.î

Ans: C Feedback: Presbycusis is an identified phenomenon that is thought to be multifactorial, and ototoxic drugs are known to contribute significantly to hearing loss in the elderly. Sensorimotor etiologies are most common, and while lifestyle modifications are often necessary, this does not rule out treatment of the hearing loss.

13. To maintain hematocrit levels in people with kidney failure, the nurse should be prepared to A) arrange for frequent blood transfusions in an outpatient clinic. B) administer iron dextran intravenously. C) administer a subcutaneous injection of recombinant human erythropoietin (rhEPO). D) administer prenatal vitamins twice a day.

Ans: C Feedback: Recombinant human erythropoietin (rhEPO) helps maintain hematocrit levels in people with kidney failure. Secondary benefits include improvement in appetite, energy level, sexual function, skin color, and hair and nail growth, and reduced cold intolerance. Blood transfusion is a treatment if symptomatic with low hemoglobin; however, they try to prevent this by giving EPO or IV iron dextran if there is severe iron deficiency anemia. Prenatal vitamins are high in vitamins but not helpful to renal failure patients.

3. One of the most reliable predictors for worsening autosomal dominant polycystic kidney disease is A) serum creatinine levels. B) blood urea nitrogen (BUN) level. C) urine albumin excretion (UAE). D) urine specific gravity.

Ans: C Feedback: Serum creatinine levels have not been found to be an effective predictor marker for worsening ADPKD, but urine albumin excretion (UAE) has been determined a reliable predictor, as have increased electrolytes and hematuria.

20. A 70-year-old male with a 40 pack-year history of smoking and long-standing nonñinsulin-dependent diabetes has been diagnosed with pancreatic cancer. Which of the following teaching points should the physician provide? A) ìWhile this is indeed serious, you should know that you have a good chance of beating this disease with appropriate treatment.î B) ìMost likely your pattern of high alcohol intake over the years contributed to your cancer.î C) ìYou will likely be facing surgery in the near future, but know that this is very unlikely to eliminate your cancer.î D) ìI know it may seem trivial at this point, but the levels of pain that accompany cancer of the pancreas are normally quite low.î

Ans: C Feedback: Surgery is the most common treatment modality for pancreatic cancer, but it does not commonly result in the resolution of the disease. Pancreatic cancer has a very poor prognosis, and alcohol is not a common contributor. Pancreatic cancer is noted to be exceptionally painful.

19. A 63-year-old woman has visited a physician because she has been intermittently passing blood-tinged urine over the last several weeks, and cytology has confirmed a diagnosis of invasive bladder cancer. Which of the following statements by the physician is most accurate? A) ìThere are new and highly effective chemotherapy regimens that we will investigate.î B) ìFortunately, bladder cancer has a very low mortality rate, and successful treatment is nearly always possible.î C) ìIt's likely that you'll need surgery, possibly a procedure called a cystectomy.î D) ìUnfortunately, there are nearly no treatment options for this type of cancer, but we will focus on addressing your symptoms.î

Ans: C Feedback: Surgical interventions are common in the treatment of bladder cancer. Effective chemotherapeutic regimens are not yet available, though there are certainly treatment options. The mortality rate of bladder cancer is high, at around 25%.

3. A nurse is providing care for a 17-year-old boy who has experienced recurrent sinus and chest infections throughout his life and presently has enlarged tonsils and lymph nodes. Blood work indicated normal levels of B cells and free immunoglobulins but a lack of differentiation into normal plasma cells. The boy is currently receiving intravenous immunoglobulin (IVIG) therapy. What is the boy's most likely diagnosis? A) X-linked hypogammaglobulinemia B) Transient hypoglobulinemia C) Common variable immunodeficiency D) IgG subclass deficiency

Ans: C Feedback: The lack of a terminal differentiation from B cells into plasma cells is the hallmark of common variable immunodeficiency. Recurrent infections, enlarged lymph nodes and tonsils, and IVIG therapy are also commonly associated.

2. A 51-year-old male has been diagnosed with alcohol-induced liver disease. He admits to the nurse providing his care that, ìI know what the lungs do, and I know what the heart does, but honestly I have no idea what the liver does in the body.î Which of the following statements would best underlie the explanation that the nurse provides? A) The liver is responsible for the absorption of most dietary nutrients as well as the production of growth hormones. B) The liver contributes to the metabolism of ingested food and provides the fluids that the GI tract requires. C) The liver metabolizes most components of food and also cleans the blood of bacteria and drugs. D) The liver maintains a balanced level of electrolytes and pH in the body and stores glucose, minerals, and vitamins.

Ans: C Feedback: The liver metabolizes protein, carbohydrate, and fat. As well, it metabolizes drugs and removes bacteria by Kupffer cells. Absorption of nutrients takes place in the intestines, and the liver does not produce the bulk of fluids secreted in the GI tract. The liver does not have a primary role in the maintenance of acidñbase or electrolyte balance.

18. When teaching a community education class about the seven warning signs of cancer, the nurse will note that the most common sign of bladder cancer is A) inability to empty the bladder fully. B) colic spasms of the ureters. C) painless bloody urine. D) passage of large clots after voiding.

Ans: C Feedback: The most common sign of bladder cancer is painless hematuria. Gross hematuria is a presenting sign in the majority of people with the disease, and microscopic hematuria is present in most others. Answer choice A refers to flaccid bladder; answer choice B refers to kidney stones; answer choice D refers to clots that are usually seen after surgery such as TURP where bladder irrigation is called for to prevent the clots from blocking urine output.

13. A client has been inhaling viruses periodically while on a cross-country flight. Which of the following situations listed below would most likely result in the stimulation of the client's T lymphocytes and adaptive immune system? A) Presentation of a foreign antigen by a familiar immunoglobulin B) Recognition of a foreign major histocompatibility complex (MHC) molecule C) Recognition of a foreign peptide bound to a self-major histocompatibility complex (MHC) molecule D) Cytokine stimulation of a T lymphocyte with macrophage or dendritic cell mediation

Ans: C Feedback: The stimulation of T cells requires the recognition of a foreign peptide bound to a self-major histocompatibility complex (MHC) molecule. Immunoglobulins do not play an antigen-presenting role, and foreign MHC molecules and cytokines do not stimulate the adaptive immune system.

20. A 3-year-old girl has been diagnosed with amblyopia. Which of the following pathophysiological processes is most likely to underlie her health problem? A) The child may have a congenital deficit of rods and/or cones. B) The girl may have chronic bacterial conjunctivitis. C) She may have been born with infantile cataracts. D) The child may have a neural pathway disorder.

Ans: C Feedback: There are multiple potential causes of amblyopia, including cataracts. A deficit of rods or cones, a neural pathway disorder, and chronic conjunctivitis are not noted to be common precursors.

9. Which of the following phenomena would be least likely to result in activation of the complement system? A) Recognition of an antibody bound to the surface of a microbe B) Increase in tissue blood flow and capillary permeability, so fluids/proteins can leak into the area C) Activation of toll-like receptors (TLRs) on complement proteins D) Direct recognition of microbial proteins

Ans: C Feedback: Toll-like receptors are not associated with the complement system. The complement system may be activated by antibody recognition, mannose binding, or microbial protein recognition.

17. A patient has just been diagnosed with pernicious anemia. The patient asks the nurse why his body is not able to absorb vitamin B12. The nurse responds, A) ìYou have too many ulcers in your stomach.î B) ìMore than likely, you were born with deficient vitamin B12 stores.î C) ìYour stomach is not secreting a substance known as intrinsic factor, which is needed to absorb vitamin B12.î D) ìYour daily intake of high saturated fats is interfering with the stomach's ability to absorb the nutrients that it needs, especially vitamin B12.î

Ans: C Feedback: Vitamin B12 is not absorbed in the absence of intrinsic factor, which is secreted by the parietal cells of the stomach. Ulcers in the stomach do not cause pernicious anemia. This is not a congenital problem, nor does the intake of fats interfere with the secretion of intrinsic factor.

9. A 39-year-old female with HIV infection has been characterized as a typical progressor by her care team and is experiencing an increase in her manifestations and health complaints as her CD4+ count declines. Which of the following health problems would her care team most likely attribute to a cause other than her HIV? A) Her recent diagnosis of bacterial pneumonia B) Her esophagitis that has been linked to herpes simplex infection C) Her decreased bone density and recent fractures D) Her increasing confusion and disorientation

Ans: C Feedback: While pneumonia, esophagitis, and cognitive deficits are all well-documented manifestations of HIV, changes in bone density are less likely to be a direct result of the virus.

13. A patient has just been diagnosed with acute glomerulonephritis. Which question should the nurse ask this client in attempting to establish a cause? A) ìDo you have a history of heart failure?î B) ìHave you recently had kidney stones?î C) ìHave you ever been diagnosed with diabetes?î D) ìHave you had any type of infection within the last 2 weeks?î

Ans: D Feedback: Acute postinfectious glomerulonephritis usually occurs after infection with certain strains of group A -hemolytic streptococci and is caused by deposition of immune complexes of antibody and bacterial antigens. Other organisms can also cause this infection.

10. A middle-aged man with diabetes reports that he must strain to urinate and that his urine stream is weak and dribbling. He also reports feeling that his bladder never really empties. The nurse knows that all of his complaints are likely caused by which of the following medical diagnoses? A) Detrusor muscle areflexia B) Detrusorñsphincter dyssynergia C) Uninhibited neurogenic bladder D) Bladder atony with dysfunction

Ans: D Feedback: Diabetes causes peripheral neuropathy, which can affect the sensory axons of the urinary bladder. Bladder atony with dysfunction is a frequent complication of diabetes mellitus.

4. Growth hormone (GH) secretion is inhibited by A) hypoglycemia. B) starvation. C) heavy exercise. D) obesity.

Ans: D Feedback: GH is inhibited by increased glucose levels, free fatty acid release, cortisol, and obesity. It is stimulated by hypoglycemia, fasting starvation, increased blood levels of amino acids, and stress conditions such as trauma, excitement, emotional stress, and heavy exercise.

20. A 79-year-old female resident of an assisted living facility receives care from a community nurse on a regular basis for treatment of a chronic venous leg ulcer. Which of the following factors would the nurse be most justified in ruling out as a contributing factor to the client's impaired wound healing? A) A lower skin collagen content than in younger adults B) Decreased fibroblast synthesis C) Slow reepithelialization D) Decreased antibody levels

Ans: D Feedback: Older adults do not normally have diminished antibody levels. Low collagen levels, decreased fibroblast activity, and slow reepithelialization are common impediments to wound healing in the elderly.

7. A 29-year-old female has been admitted to the emergency department following a suicide attempt by overdose of acetaminophen. What changes in the client's liver and diagnostic results would the care team most likely anticipate? A) Hepatocellular necrosis evidenced by increased ALT and AST levels B) Allergic inflammation accompanied by an increase in serum IgE and basophils C) Cholestatic reaction with increased bilirubin count D) Rapid onset of hepatitis and increased GGT, ALT, and bilirubin

Ans: A Feedback: Acetaminophen is commonly implicated in cases of direct hepatotoxicity, a situation that is characterized by hepatocellular necrosis and increased ALT and AST levels. An allergic-type reaction is associated with idiosyncratic drug reactions, and cholestatic reactions and development of hepatitis are not noted to be associated with acetaminophen.

15. A 60-year-old man has long managed his type 1 diabetes effectively with a combination of vigilant blood sugar monitoring, subcutaneous insulin administration, and conscientious eating habits. This morning, however, his wife has noted that he appears pale and clammy and appears to be in a stupor, though he is responsive. She suspects that he has made an error in his insulin administration and that he is experiencing a hypoglycemic episode. Which of the following actions should be the wife's first choice? A) Administration of 15 to 20 g of glucose in a concentrated carbohydrate source B) Administration of subcutaneous glucagon C) IV infusion of 50% dextrose and water solution D) Careful monitoring for level of consciousness and resolution of hypoglycemia

Ans: A Feedback: An insulin reaction necessitates intervention in addition to careful observation. The ideal response to an insulin reaction in a still, conscious client is the administration of 15 to 20 g of glucose in a concentrated carbohydrate source. Glucagon or D50 would be indicated if the client is unconscious or unable to swallow.

20. In the context of a workshop on rheumatoid arthritis, a clinical educator is teaching a group of nurses about autoimmune diseases. Which of the following statements by an attendee would the educator most likely want to follow up with further teaching? A) ìIntroduction of a foreign antigen can sometimes induce a cascade of immune response that is not self-limitingî B) ìOften the problem can be traced to antigens that sensitize T cells without the need for presentation.î C) ìIn some cases, the body attacks its own cells that are chemically similar to those of infectious organisms.î D) ìSometimes when the body's own cells are released after a long time, they are interpreted as being foreign.î

Ans: A Feedback: Autoimmune responses are not considered to be uncontrolled cascades of immune response that are catalyzed by introduction of an antigen. Superantigens are able to forego the normal antigen presentation process and directly stimulate T-cell response, resulting in overactivation of T cells. Molecular mimicry involves the misidentification of somatic cells as similar foreign cells, and self-antigens that have been sequestered for long periods can invoke an immune response.

2. The nurse knows which of the following statements listed below is accurate regarding the functions and nature of cytokines relative to a variety of pathologies? A) ìA particular cytokine can have varied effects on different systems, a fact that limits their therapeutic use.î B) ìCytokine production is constant over time, but effects are noted when serum levels cross a particular threshold.î C) ìMost cytokines are produced by granular leukocytes, and different cells are capable of producing the same cytokine.î D) ìCytokine actions are self-limiting, in that activation of one precludes activation of other cytokines with similar actions.î

Ans: A Feedback: Because cytokines can mediate diverse effects due to their pleiotropic properties, they can have significant side effects. Cytokine production is brief, not constant, and production does not normally take place in granulocytes. Activation of a cytokine does not necessarily limit other similar cytokines, and additive effects are not uncommon.

15. Chronic anxiety and stress contribute to ulcers. Which of the following effects of the sympathetic nervous system is most responsible for this effect? A) Inhibition of the actions of Brunner glands B) Overstimulation of the oxyntic glands C) Suppression of cholecystokinin D) Inflammation of the parotid glands

Ans: A Feedback: Brunner glands, which produce large amounts of alkaline mucus that protects the duodenum from acid and digestive enzymes, are strongly affected by sympathetic stimulation, which causes a marked decrease in mucus production. Where the stomach contents and secretions from the liver and pancreas enter the duodenum, deficiency of mucus can cause irritation and, potentially, ulcers. Overstimulation of the oxyntic glands is not produced by the SNS. The hormone cholecystokinin, which is thought in part to control gastric emptying, is released in response to the pH, osmolality, and fatty acid composition of the chyme. The two largest salivary glands are called parotid glands. One is located in each cheek over the jaw in front of the ears. Inflammation of one or more of these glands is called parotitis.

14. A 28-year-old man presents with complaints of diarrhea, fecal urgency, and weight loss. His stool is light colored and malodorous, and it tends to float and be difficult to flush. He has also noted tender, red bumps on his shins and complains of pain and stiffness in his elbows and knees. Sigmoidoscopy reveals discontinuous, granulomatous lesions; no blood is detected in his stool. Which of the following diagnoses would his care team first suspect? A) Crohn disease B) Ulcerative colitis C) Diverticulitis D) Colon cancer

Ans: A Feedback: Crohn disease, like ulcerative colitis, causes diarrhea, fecal urgency, weight loss, and systemic symptoms such as erythema nodosum and arthritis. Unlike ulcerative colitis, it also causes steatorrhea but is not as likely to cause blood in the stool. The granulomatous ìskipî lesions confirm the diagnosis of Crohn disease. Neither diverticulitis nor colon cancer would cause this combination of symptoms and signs.

9. Following focal seizures that have damaged the dominant hemisphere of a patient's auditory association cortex, the nurse may observe the patient displaying A) receptive aphasia. B) facial drooping. C) auditory hallucinations. D) delusions of grandeur.

Ans: A Feedback: Damage to the auditory association cortex, especially if bilateral, results in deficiencies of sound recognition and memory (auditory agnosia). If the damage is in the dominant hemisphere, speech recognition can be affected (sensory or receptive aphasia). The others are not caused by focal seizures.

4. Which of the following phenomena best accounts for the increased presence of leukocytes at the site of inflammation? A) Existing leukocytes stick to the epithelial cells and move along blood vessel walls. B) Increased numbers of leukocytes are released into circulation via cytokine stimulation. C) Leukocytes are osmotically drawn from circulation into the interstitial space as a result of swelling. D) Epithelium expresses leukocyte stimulation factors in response to cell injury.

Ans: A Feedback: During inflammation, leukocytes accumulate at the point of epithelial contact in the processes of margination, adhesion, and transmigration. This is not directly achieved by a way of increased leukocyte production or release, nor by osmotic pressure. The epithelium does not produce leukocyte stimulation factors.

3. A female neonate has been in respiratory distress since delivery and is unresponsive to oxygen therapy. Endoscopy has confirmed a diagnosis of esophageal atresia and tracheoesophageal fistulae (EA/TEF). Which of the following explanations should the care team provide to the infant's parents? A) ìWe will have to perform surgery to correct the hole in her throat to make sure that she is able to swallow and breathe normally.î B) ìThis problem will require respiratory therapy and supplementary feeding, but it will likely resolve itself over time.î C) ìThe biggest risk that your daughter will face until this is fixed is the danger of malnutrition and dehydration.î D) ìThe priority in our immediate treatment prior to her surgery will be pain management, as the contents of her stomach can burn her lungs.î

Ans: A Feedback: EA/TFE necessitate surgery and preclude both normal respiration and swallowing. Aspiration is the primary immediate risk and the priority for treatment. Although the infant will require respiratory therapy to assist with proper tube placement and ventilator maintenance, the only effective treatment is surgery. Aspiration of feeding (aspiration pneumonia) is a major complication that can occur immediately and can be life threatening. Maintaining an open airway and adequate gas exchange are the priority nursing diagnoses for this infant.

16. A number of clients on a geriatric subacute medical floor of a hospital have developed foul-smelling diarrhea over the last several days, and subsequent culture of stool samples has confirmed the presence of Clostridium difficile in each case. The care team in the unit would recognize that which of the following factors likely contributed to the health problem and would anticipate which of the following treatments? A) The use of broad-spectrum antibiotics likely played a role in the development of infections, and most clients would likely receive metronidazole as a treatment. B) Genetic predisposition and the presence of the bacterium in clients' normal flora likely contributed, and treatment would consist of broad-spectrum antibiotics. C) Poor hand washing practice on the part of care providers led to the outbreak, and treatment will consist of hydration and nutritional support. D) Ingestion of contaminated food probably contributed to the infections, and corticosteroids will be needed to treat them.

Ans: A Feedback: Elimination of the normal intestinal flora by broad-spectrum antibiotics commonly precedes infection by C. difficile, and metronidazole is the normal treatment. Neither genetic predisposition nor ingestion of contaminated food is a likely factor. While poor hand hygiene can spread the bacteria, treatment for C. difficile necessitates antibiotics and not simply hydration and nutritional support. The treatment of C. difficile is with antibiotics. Metronidazole is the drug of choice with vancomycin, being reserved for people who cannot tolerate metronidazole. Corticosteroids will not kill the bacterium.

7. A woman with a diagnosis of type 2 diabetes has been ordered a hemoglobin A1C test for the first time by her primary care provider. The woman states, ìI don't see why you want to test my blood cells when its sugar that's the problem.î What aspect of physiology will underlie the care provider's response to the client? A) The amount of glucose attached to A1C cells reflects the average blood glucose levels over the life of the cell. B) Hemoglobin synthesis by the bone marrow is inversely proportionate to blood glucose levels, with low A1C indicating hyperglycemia. C) The high metabolic needs of red cells and their affinity for free glucose indicate the amount of glucose that has been available over 6 to 12 weeks. D) Insulin is a glucose receptor agonist on the hemoglobin molecule, and high glucose suggests low insulin levels.

Ans: A Feedback: Glucose entry into red blood cells is not insulin dependent, and the rate at which glucose becomes attached to the hemoglobin molecule depends on blood glucose levels. A1C levels thus indirectly indicate glucose levels. Hemoglobin synthesis, the metabolic needs of hemoglobin, and an agonist role of insulin do not underlie the A1C test.

1. A female patient presented to her primary care physician with classic signs and symptoms of Cushing syndrome. Upon testing, it was discovered that the patient had vaginal small cell carcinoma. How can the health care providers explain her Cushing syndrome signs and symptoms to this patient? A) ìYour tumor in your vagina is secreting a hormone called adrenocorticotropic hormone (ACTH), which is responsible for these signs and symptoms.î B) ìWe are going to have to run some more tests. We think you might have a problem with your pituitary gland.î C) ìThere is no connection between the Cushing syndrome and the vaginal carcinoma. You have two very distinct problems occurring at the same time.î D) ìWe need to check your thyroid. Your Cushing syndrome may be caused by hypofunction of this gland.î

Ans: A Feedback: Hyperfunction is usually associated with excessive hormone production. This can result from excessive stimulation and hyperplasia of the endocrine gland or from a hormone-producing tumor. A clinical example of this phenomenon is evidenced by the case of a woman with vaginal small cell carcinoma who also presented with Cushing syndrome. After testing, it was determined that the tumor is secreting ACTH. In this situation, the cause is not related to a pituitary problem. There is a connection between Cushing syndrome and the carcinoma. The thyroid gland is not responsible for Cushing syndrome.

13. A pancreatitis patient is admitted with weight loss, nausea, and vomiting. To maintain nutrition, the physician orders parental nutrition to be started. Knowing that a major side effect of parenteral nutrition is a hyperosmolar hyperglycemic state, the nurse should assess the patient for which clinical manifestation (listed below)? A) Dry lips, excess urine output, and seizures B) Facial tics, shuffling gait, and stiff joints C) Fever, chills, and elevated BP of 170/101 D) Irritability, bradycardia, and wheezing noted on inspiration

Ans: A Feedback: Hyperosmolar hyperglycemic state is characterized by high blood glucose (>600 mg/dL), dehydration (dry lips), depression of sensorium, hemiparesis, seizures, and coma, and also weakness, polyuria, and excessive thirst. HHS may occur in various conditions, including type 2 diabetes, acute pancreatitis, severe infection, MI, and treatment with oral or parenteral nutrition solutions.

10. A nurse is providing care for a client who is immunocompromised following chemotherapy. The nurse knows which of the following characterizations of the adaptive immune system listed below is responsible for the client's disruption in his normal immune function? A) Epitopes on antigens are recognized by immunoglobulin receptors following presentation by accessory cells. B) Haptens combine to form epitopes that stimulate the response of regulatory and effector cells. C) Effector cells orchestrate the immune response of regulatory cells toward an antigen. D) Accessory cells such as macrophages are engulfed by regulatory cells, stimulating effector cells.

Ans: A Feedback: In the adaptive immune response, accessory cells present antigen epitopes to receptors, initiating the immune response of lymphocytes. Epitopes may combine to form haptens, and regulatory cells orchestrate effector cells. Regulatory cells do not engulf accessory cells.

18. A 10-year-old child with strep throat asks the nurse, ìwhy there are large bumps [lymph nodes] on my neck when my throat gets sore?î The nurse replies lymph nodes A) help your body fight off infections by allowing special cells (lymphocytes and macrophages) move through the lymph chain and engulf and destroy germs. B) bring in cells into the lymph node (your bump) to stop the germs from going anywhere else in the body. C) bring all kind of good cells to your throat so that they can wall the strep off and keep the germs from getting any food or water.î D) help your tonsils get bigger with cells that will bring immune cells into your throat to prevent any other infections.

Ans: A Feedback: Lymphocytes and macrophages move slowly through the lymph nodes so that they can have adequate time to engulf microorganisms and interact with circulating antigens. The lymph nodes do not bring cells into the node to stop germs; do not bring cells to the throat to wall off strep; and do not enlarge the tonsil with immune cells.

14. A 67-year-old patient diagnosed with myasthenia gravis will likely display which clinical manifestations as a result of autoantibodies ultimately blocking the action of acetylcholine, resulting in destruction of the receptors? A) Weakness of the eye muscles; difficulty in swallowing and slurred speech; impaired gait B) Tremor of hands/arms; rigidity of the arms; shuffling gait C) Short-term memory lapses; problems with orientation; a lack of drive or initiative D) Facial droop; slurred speech; weakness on one side of the body

Ans: A Feedback: Myasthenia gravis ultimately results in destruction of receptors in the neuromuscular junction leading to a decrease in neuromuscular function. Answer B relates to s/s of Parkinson disease; answer C relates to Huntington disease; and answer D relates to classic s/s of CVA.

5. A 73-year-old man presents to his family physician with complaints of recent urinary hesitation and is eventually diagnosed with benign prostatic hyperplasia (BPH). Which of the following clinical consequences would his care provider expect prior to the resolution of his health problem? A) Hydroureter and pain B) Development of renal calculi and renal cysts C) Unilateral hydronephrosis and pain D) Development of glomerulonephritis or nephrotic syndrome

Ans: A Feedback: Pain and the distention of the distal ureter would be expected manifestations of BPH. Renal calculi, cysts, glomerulonephritis, and nephrotic syndrome are unlikely to develop consequentially, and unilateral hydronephrosis is unlikely, given that the obstruction is below the level of the ureterovesical junction.

4. A stroke patient is having difficulty swallowing food and beverages. The patient complains that he feels like ìthe food is sticking to the back of his throat.î Given this information, the priority nursing interventions would be to A) make the patient ìnothing per osî (NPO) and call the physician. B) feed the patient while he is sitting in an upright position. C) add a thickening agent to all of the patient's beverages. D) warrant no action since this is a normal occurrence after a stroke.

Ans: A Feedback: People with dysphagia usually complain of choking, coughing, or an abnormal sensation of food sticking in the back of the throat or upper chest when they swallow. A neuromuscular cause involves lesions of the CNS, such as a stroke, which often involve the cranial nerves that control swallowing. Feeding in upright position is good once it is determined by swallowing evaluation that the patient can swallow food without it going into the lungs. Likewise, thickening agents help dysphagia patients after a swallow evaluation has been performed. No action could put the patient at risk for aspiration pneumonia.

19. A 1-year-old baby boy with renal dysplasia risks end-stage renal disease unless intervention occurs. Which of the following treatment options is his care team most likely to reject? A) Dietary restriction plus erythropoietin B) Continuous cyclic peritoneal dialysis C) Renal transplantation D) Continuous ambulatory peritoneal dialysis

Ans: A Feedback: Renal transplantation and dialysis are recommended for children; of these, transplantation is the preferred treatment. Conservative measures are inappropriate in this age group because of the importance of fostering proper bone growth, especially in the first 2 years, and appropriate cognitive development, which is at risk due to issues such as uremic encephalopathy and the effect of renal failure upon the central nervous system of the developing child.

12. Otitis media (OM), which can occur in any age group, is the most common diagnosis made by health care providers who care for children. Which bacterial pathogen causes the largest proportion of cases that result in sensorineural hearing loss? A) Streptococcus pneumoniae B) Acoustic neuromas C) Haemophilus influenzae D) Parainfluenza

Ans: A Feedback: S. pneumoniae is the most common cause of bacterial meningitis that results in sensorineural hearing loss after the neonatal period. Acoustic neuromas are cancers that cause impaired hearing. Parainfluenza and influenza viruses are common viral pathogens in OM.

11. Which of the following statements best captures the essence of a second messenger in the mechanisms of the endocrine system? A) Second messengers act as the intracellular signal that responds to the presence of a hormone. B) Endocrine-producing cells must release both a hormone and a second messenger in order to exert a distant effect. C) Second messengers act to supplement hormone effects on cell receptors when the desired hormonal effect must be either increased. D) Second messengers provide an alternative pathway for endocrine effects on a cell that bypass the normal receptor pathways.

Ans: A Feedback: Second messengers interact with hormones that cannot cross the cell membrane, and they mediate the ultimate effect on the cell. They are not produced by the hormone-producing cell, and they are necessary to bring about hormonal effects, not simply for increasing the intensity of the effect. They are not an alternative mechanism of effect but rather a prerequisite for certain hormonal effects on body cells.

10. A nurse is changing the wound dressing on the coccyx-region pressure ulcer of an immobilized patient. The existing dressing is saturated with both watery, clear discharge and foul, gray-colored liquid. Which of the following entries in the patient's chart best captures this? A) ìLarge amounts of suppurative and serous exudates notedî B) ìPurulent discharge and fibrinous exudates noted on existing dressingî C) ìAbscess activity noted to coccyx woundî D) ìPlasma proteins and membranous exudates present on existing dressingî

Ans: A Feedback: Serous discharge is clear and low in plasma proteins, while suppurative, or purulent, exudates are a mass of degraded cells. An abscess would be physically contained with no discharge, and the exudate is neither fibrinous nor membranous.

1. Which of the following statements best captures an aspect of the role of hormones in the body? A) Some chemical substances can function as hormones or be integrated with the central and peripheral nervous systems. B) Hormones directly initiate many of the processes that contribute to homeostasis. C) Control of body processes is ensured by the fact that a single hormone can only exert one effect on one specific system or tissue. D) Each hormone that exists in the body is produced by only one specific endocrine gland.

Ans: A Feedback: Some chemicals, such as epinephrine, can both function as a hormone and be closely integrated with the central and peripheral nervous systems as well as the immune systems, leading to current terminology such as ìneuroendocrine.î Hormones modulate, but do not initiate, changes in the body, and one hormone may exert multiple effects on multiple body systems. Hormones are produced by a variety of body tissues, not solely by endocrine glands.

3. A patient being seen in the clinic has just received his routine lab results. The patient has been told that his cholesterol level is extremely elevated. The physician plans to give the patient a prescription for medication to help control this condition. Which of the following medications should the nurse prepare to educate the patient on? A) Atorvastatin (Lipitor) B) Abilify (aripiprazole) C) Pancrecarb (Pancrelipase) D) Pancrelipase (Ultrase)

Ans: A Feedback: The HMG-CoA reductase inhibitors or statins are used to treat high cholesterol levels by inhibiting this step in cholesterol synthesis. Abilify is for schizophrenia; Pancrecarb and Pancrelipase are pancreatic enzymes used for pancreatitis.

10. A hospital client with a diagnosis of chronic renal failure has orders for measurement of her serum electrolyte levels three times per week. Which of the following statements best captures the relationship between renal failure and sodium regulation? A) Clients with advanced renal failure are prone to hyponatremia because of impaired tubular reabsorption. B) Renal clients often require a sodium-restricted diet to minimize the excretion load on remaining nephrons. C) Clients with renal failure often maintain high sodium levels because of decreased excretion. D) Restricting sodium intake helps to preserve nephron function and has the additional benefit of lowering blood pressure.

Ans: A Feedback: The compromised ability of the tubular nephrons to reabsorb sodium predisposes renal clients to low serum sodium levels. A sodium restriction is thus not normally indicated.

4. A 40-year-old male who has been HIV positive for 6 years is experiencing a new increase in his viral load along with a corresponding decrease in his CD4+ count. Which of the following aspects of his immune system is likely to remain most intact? A) Presentation of major histocompatibility molecules on body cells B) Orchestration of natural killer cells as part of cell-mediated immunity C) Activation of B lymphocytes D) Phagocytic function of monocytes and macrophages

Ans: A Feedback: The expression of MHC on various cells of the body is not noted to be directly influenced by HIV. However, infected CD4+ cells are compromised in their ability to guide the action of NK cells, to direct phagocytic function of macrophages, and to present antigens that activate B cells.

7. Which of the following best describes the half-life of a highly protein-bound drug such as thyroxine (99% protein bound)? The half-life would be A) much longer to reduce the concentration of the hormone by one half. B) shorter because only a little of the hormone has to be used up to reduce the concentration. C) dependent on which drugs were in the blood system holding on to the hormone. D) dependent on the liver to carry the hormone to its designated target organ.

Ans: A Feedback: The half-life of a hormoneóthe time it takes for the body to reduce the concentration of the hormone by one halfóis positively correlated with its percentage of protein binding. Thyroxine, which is more than 99% protein bound, has a half-life of 6 days, whereas aldosterone, 15% bound, has a half-life of only 25 minutes.

13. A nurse on a medical unit is providing care for a 37-year-old female patient who has a diagnosis of Graves disease. Which of the following treatments would the nurse most likely anticipate providing for the client? A) -Adrenergicñblocking medications to reduce sympathetic nervous stimulation B) Administration of levothyroxine to supplement thyroid function C) Calcium channel blocking medications to reduce heart rate and cardiac risks D) Administration of somatostatin analogs to inhibit GH production

Ans: A Feedback: The hyperthyroidism that constitutes Graves disease can often be mitigated by the administration of -adrenergicñblocking medications. Levothyroxine would be used to address hypothyroidism, and calcium channel blockers are not an identified treatment modality for Graves disease. Somatostatin analogs are used to treat GH excess.

5. When the sympathetic nervous system is stimulated, the interstitial cells of Cajal, pacemaker cells of the GI tract, react by A) decreasing amplitude or abolishing the slow waves that control the spontaneous oscillations in membrane potentials. B) increasing the peristaltic motion of the GI tract, thereby causing explosive diarrhea. C) increasing the amount of secretions being entered into each segment of the intestinal tract. D) signaling the vagus nerve to slow down motility and increase absorption of water from the large intestine.

Ans: A Feedback: The interstitial cells of Cajal that are found in groups between the layers of smooth muscle tissue are hypothesized to function as the pacemakers. These cells display rhythmic, spontaneous oscillations in the membrane potentials, called slow waves, ranging in frequency from approximately 3 per minute in the stomach to 12 per minute in the duodenum. The vagus nerve responds to parasympathetic innervation. GI motility is enhanced because of increased vagal activity that could cause diarrhea.

5. When explaining the final stages of the inflammatory response to pathogens, the nurse will educate the patient about A) how the body can kill the pathogen by generating toxic oxygen and nitrogen products producing such things as nitric oxide and hydrogen peroxide. B) margination, which is a process whereby white cells (leukocytes) stick to the endothelium and accumulate along the blood vessel. C) the increase in vascular permeability, which lets fluids leak into the extravascular tissues. D) the promotion of tissue regeneration whereby monocytes and macrophages produce potent prostaglandins and leukotrienes.

Ans: A Feedback: The latter stages of phagocytosis results in intracellular killing of pathogens accomplished by several mechanisms, including toxic oxygen and nitrogen products, lysozymes, proteases, and defensins. The metabolic burst pathways generate toxic oxygen and nitrogen products (i.e., nitric oxide, hydrogen peroxide, and hypochlorous acid). Margination is one of the early stages of the inflammatory response. Vascular changes occur with inflammation but are prior to the final stage. Macrophages arrive within hours at the inflammation site.

9. Neurotransmitters like catecholamines (e.g., dopamine and epinephrine) have a reaction time of A) milliseconds. B) less than 10 minutes. C) 24 to 36 hours. D) 4 to 7 days.

Ans: A Feedback: The neurotransmitters, which control the opening of ion channels, have a reaction time of milliseconds.

7. During a physical exam, the nurse practitioner notes that the patient's optic disk is very pale with a larger size/depth of the optic cup. At this point, they are thinking the patient may have A) glaucoma. B) diabetes retinopathy. C) macular degeneration. D) retinal tear.

Ans: A Feedback: The normal optic disk has a central depression called the optic cup. With progressive atrophy of axons caused by increased intraocular pressure, pallor of the optic disk develops, and the size and depth of the optic cup increase. Diabetes retinopathy, macular degeneration, nor retinal tear has these clinical manifestations.

16. An 87-year-old male resident of an assisted living facility has been consistently continent of urine until the last several weeks. Which of the following actions by the care providers at the facility is the most likely priority? A) Performing a physical examination and history to determine the exact cause and character of the incontinence B) Providing client education focusing on the fact that occasional incontinence is a normal, age-related change C) Teaching the resident about protective pads, collection devices, and medications that may be effective D) Showing the resident the correct technique for exercises to improve bladder, sphincter, and pelvic floor tone

Ans: A Feedback: The priority in the treatment of incontinence in the elderly is an acknowledgement that it is not an inevitability and that the exact causes should and most often can be identified. This identification by way of history-taking and examination would supersede teaching about protective devices or exercises.

9. A male patient has just been diagnosed with esophageal cancer. He knew that he was losing weight and fatigued most days, but he just attributed it to aging and working. The physician recommends chemotherapy and irradiation. However, the cancer has already metastasized. The patient asks the nurse what he can expect if he agrees to the treatments. The nurse responds, A) ìThe therapies may shrink the cancer.î B) ìThe doctor is prescribing treatment measures to help you swallow better.î C) ìThese therapies will most likely cure your cancer.î D) ìYou need to talk with your physician some more. I will page him for you.î

Ans: A Feedback: The prognosis for people with cancer of the esophagus, although poor, has improved. Even with modern forms of therapy, the long-term survival is limited because, in many cases, the disease has already metastasized by the time the diagnosis is made. These therapies may help with food consumption, but that is not their primary purpose. Nor, will these therapies cure the cancer.

18. A 38-year-old male has presented to a clinic for the treatment of severe dermatitis after contact with poison ivy on a camping trip. The client has been prescribed prednisone, a corticosteroid, for the treatment of his skin condition. The clientís care provider has emphasized that dosages of the drug will be gradually tapered off rather than stopped upon resolution of the symptoms. What is the most accurate rationale for this dosing protocol? A) The client's hypothalamicñpituitaryñadrenal (HPA) system will require recovery time before normal function is restored. B) Steroids can induce a dependency that it best addressed with a gradual withdrawal. C) HPA function is heightened during steroid administration and must return to normal levels before the drug is completely stopped. D) Abrupt cessation of the drug can contribute to symptoms similar to Cushing syndrome.

Ans: A Feedback: The suppression of the HPA system that accompanies steroid therapy requires time for a return to normal function. Dependency on the drug itself is not the rationale for tapering, and HPA function is suppressed, not heightened during therapy. Abrupt cessation can contribute to an Addison diseaseñlike response, not Cushing syndrome.

14. A 31-year-old female has been recently diagnosed with type 2 diabetes mellitus and is attending a diabetes education class. Which of the following statements by the woman demonstrates an accurate understanding of her health problem? A) ìI'll have to control my blood sugars, my blood pressure, and my cholesterol in order to make sure I don't develop sight problems.î B) ìI'm grieving the fact that I won't be able to get pregnant without causing permanent damage to my vision.î C) ìIt's surprising that sugar in my blood can accumulate on the lens of my eye and cause a loss of sight.î D) ìI want to avoid going through the treatments for sight restoration that I would need if my diabetes causes damage to my vision.î

Ans: A Feedback: The threat to vision that is posed by poorly controlled blood sugar levels is compounded by high blood pressure and/or cholesterol levels. Pregnant women with diabetes need additional care to monitor their sight, but they will not necessarily lose it. The damage caused by diabetes does not occur on the lens, and restoration of lost visual acuity is not normally possible.

20. A 48-year-old woman has been found to have nodules on her thyroid that must be biopsied to determine whether or not they are malignant. Which of the following imaging techniques will be most helpful to the surgeon in visualization of the nodes for fine needle aspiration? A) Ultrasound B) Magnetic resonance imaging C) Radioactive scanning using radioiodine D) Radioactive scanning using sestamibi

Ans: A Feedback: Thyroid ultrasound is recommended for managing thyroid nodules and can aid in visualization of the nodule for biopsy (fine needle aspiration [FNA]), which is necessary to help distinguish benign from malignant etiology. Magnetic resonance imaging is the preferred choice for pituitary and hypothalamic imaging. Isotopic imaging includes radioactive scanning of the thyroid (e.g., using radioiodine), parathyroids (e.g., using sestamibi), and adrenals (e.g., using metaiodobenzylguanidine [MIBG] to detect pheochromocytoma).

2. A 2-year-old girl has had repeated ear and upper respiratory infections since she was born. A pediatrician has determined a diagnosis of transient hypogammaglobulinemia of infancy. What is the physiological origin of the child's recurrent infections? A) The child's immune system is unable to synthesize adequate immunoglobulin on its own. B) The child had a congenital absence of IgG antibodies that her body is only slowly beginning to produce independently. C) The child was born with IgA and IgM antibodies, suggesting intrauterine infection. D) The child lacks the antigen-presenting cells integral to normal B-cell antibody production.

Ans: A Feedback: Transient hypogammaglobulinemia of infancy is characterized by inadequate communication between B and T cells. IgG would be inherited through placental blood, and an intrauterine infection is neither causative or nor synonymous with transient hypogammaglobulinemia of infancy. The diagnosis does not include a lack of antigen-presenting cells.

4. A 3-year-old girl has just been diagnosed with type 1A diabetes. Her parents are currently receiving education from the diabetes education nurse. How can the nurse best explain to the parents the etiology (cause) of their daughter's diabetes? A) ìThe problem that underlies her diabetes is that her own body has destroyed the cells in her pancreas that produce insulin.î B) ìIt's not known exactly why your daughter has completely stopped making insulin, and treatment will consist of your rigidly controlling her diet.î C) ìThis tendency to produce insufficient amounts of insulin is likely something that she inherited.î D) ìEnvironmental and lifestyle factors are known to play a part in the fact that her pancreas secretes and withholds insulin at the wrong times.î

Ans: A Feedback: Type 1A, or immune-mediated, diabetes involves the autoimmune destruction of pancreatic beta cells and a consequent absolute lack of insulin. Exogenous insulin required as dietary control alone is insufficient. The central problem is an absolute lack of insulin production rather than deranged release.

6. Following the diagnosis of acute renal failure, the nurse knows that one of the earliest manifestations of residual tubular damage is which of the following lab/diagnostic results? A) Elevated blood urea nitrogen (BUN) B) Serum creatinine elevation C) Inability to concentrate urine D) Reduced glomerular filtration rate

Ans: A Feedback: Urine tests that measure urine osmolality, urinary sodium concentration, and fractional excretion of sodium help differentiate prerenal azotemia, in which the reabsorptive capacity of the tubular cells is maintained, from tubular necrosis, in which these functions are lost. One of the earliest manifestations of tubular damage is the inability to concentrate urine. Conventional markers of serum creatinine and urea nitrogen, fractional secretion of sodium to assess glomerular filtration rate (GFR), and urine output do not manifest for 1 to 2 days after the acute renal failure has begun.

16. A public health nurse is teaching a health promotion class to a group of older adults at a seniors' center. A woman attending states that, ìMy husband has got dry macular degeneration, and I don't know what we're going to do when he goes blind from it.î How can the nurse best respond to the woman's statement? A) ìVitamins C and E as well as zinc and beta carotene may have some value in slowing the progression of his disease.î B) ìYou should talk to your doctor about the surgical options that might help preserve his sight.î C) ìIf your husband can lower his blood pressure and bad cholesterol, it can possibly slow the progression of his disease.î D) ìCataract surgery is showing promise as a way of replacing the part of the eye associated with macular degeneration.î

Ans: A Feedback: Vitamin E, vitamin C (ascorbic acid), zinc, and beta carotene have shown promise at slowing the progression of the age-related macular degeneration in persons with the disease. Surgical options are not normally available for the dry variety of macular degeneration. Blood pressure control, cholesterol control, and cataract surgery are unlikely to be directly useful.

13. The nurse should anticipate that a patient diagnosed with spastic bladder dysfunction may be prescribed which of the following medications that will help decrease detrusor muscle tone and increase bladder capacity? Select all that apply. A) Ditropan (Oxybutynin), an antimuscarinic drug B) Detrol LA (tolterodine tartrate), an antimuscarinic drug C) Uroxatral (alfuzosin), an -adrenergic antagonist D) Flomax (tamsulosin), an -blocker E) Bactrim (sulfamethoxazole and trimethoprim), antibiotics

Ans: A, B Feedback: Antimuscarinic drugs, such as oxybutynin, tolterodine, and propantheline, decrease detrusor muscle tone and increase bladder capacity in people with spastic bladder dysfunction. Answer choices C and D are medications prescribed for males with BPH.

17. Parents have brought their 7-year-old child into the emergency room with abdominal pain. Which of the following clinical manifestations would lead the health care team to suspect the child has appendicitis? Select all that apply. A) Tenderness in right lower quadrant with palpation B) Rebound tenderness in inguinal areas with palpation C) Redness and warmth over right lower quadrant D) Bloating and flatulence noticeable E) Urine has the smell of stool with brown coloring

Ans: A, B Feedback: Appendicitis usually has an abrupt onset. Palpation of the abdomen usually reveals a deep tenderness in the LRQ, which is confined to a small area approximately the size of the fingertip. It usually is located at approximately the site of the inflamed appendix. Rebound tenderness, which is pain that occurs when pressure is applied to the area and then released, and spasm of the overlying abdominal muscles are common. An abscess may display redness and warmth. Bloating and flatulence are usually seen with diverticular disease as is the development of fistula (vesicosigmoid), where stool is seen in the urine.

5. Testing for short stature growth hormone (GH) problems can be done by pharmacologic means. Which of the following medications can be utilized to test for a rise in GH? Select all that apply. A) Insulin B) Levodopa C) Persantine D) Dobutamine E) Sestamibi

Ans: A, B Feedback: Diagnostic procedures for short stature include tests to exclude nonendocrine causes. If the cause is hormonal, extensive hormonal testing procedures are initiated. Tests can be performed using insulin, CHRH, levodopa, and arginine, all of which stimulate GH secretion so that GH reserve can be evaluated. Persantine, dobutamine, and sestamibi are used in cardiac stress testing. Sestamibi is also used in the testing of the parathyroid.

9. During a lecture about medication therapy for type 2 diabetic patients, the pharmacology instructor discusses side effects to monitor following subcutaneous injection of exenatide (Byetta), a glucagon-like peptide-1 (GLP-1) agonist. Of the following list, which signs and symptoms should the students be assessing in this patient? Select all that apply. A) Nausea B) Weight loss C) Dizziness D) Tachycardia E) Dehydration

Ans: A, B Feedback: Exenatide is approved as an injectable monotherapy or combination adjunctive therapy for people with type 2 diabetes in association with diet and exercise. It is injected subcutaneously 60 minutes before a meal. The major side effects are nausea and weight loss. Dizziness and tachycardia are not listed as major side effects; however, they may be indicative of hypoglycemia related to other medications.

9. A patient diagnosed with a primary immunodeficiency disorder has asked his siblings to be tested as possible stem cell donors. When discussing this procedure with his family, the nurse emphasizes that stem cells can be harvested from: Select all that apply. A) bone marrow. B) peripheral blood. C) skin tissue harvesting. D) mouth swabs. E) tears.

Ans: A, B Feedback: Hematopoietic stem cells can be harvested from either the bone marrow or the peripheral blood and also from the umbilical cord blood. From sibling donors, the results are effective with improved survival in approximately 90% of people.

18. Following kidney transplantation, the patient is prescribed maintenance immunosuppressive therapy consisting of prednisone, azathioprine, and cyclosporine. Educating the patient about long-term maintenance on immunosuppressive therapy should include discussion of side effects that may include: (Select all that apply). A) cardiovascular complications. B) increased risk of developing cancer. C) nephrotoxicity of a newly transplanted kidney. D) development of moon face and buffalo hump. E) ringing or buzzing of the ears.

Ans: A, B Feedback: Mostly all immunosuppressive drugs have serious side effects such as CV complications, metabolic dysfunction, and cancer. Rarely do these medications harm the newly transplanted kidney. Distracter D lists classic s/s of Cushing syndrome. Ringing or buzzing of the ears can be associated with ibuprofen intake.

10. A patient comes into a clinic complaining of cough, fever, and shortness of breath. The patient informs the health care provider that he is HIV positive. Upon physical exam, the family nurse practitioner (FNP) may note which of the following clinical manifestations of suspected Pneumocystis jiroveci pneumonia (PCP)? Select all that apply. A) Interstitial infiltrates on chest x-ray B) Respiratory rate of 32 with normal breath sounds C) Stridor when taking a deep breath D) Use of abdominal muscles to breathe while sitting on the exam table E) Night sweats that require clothing changes frequently throughout the night

Ans: A, B Feedback: PCP is a common presenting manifestation of AIDS or people with compromised immune systems. The symptoms include cough, fever, shortness of breath, and weight loss. Physical exam demonstrates only fever and tachypnea (elevated respiratory rate) and normal breath sounds. Chest x-ray shows interstitial infiltrates. Night sweats are usually associated with tuberculosis infection.

17. A newly diagnosed HIV-positive adolescent has blood work drawn, which includes a CD8 T-cell count. The nurse knows which of the following functions of CD8 T cells listed below will assist the adolescent's immune system in fighting off the viral attack? Select all that apply. A) Release destructive enzymes B) Trigger intracellular programmed death C) Cause allergens to surround the virus D) Boost antigenñantibody response E) Remove foreign material from lymph before it enters the blood

Ans: A, B Feedback: The primary function of cytotoxic T (CD8) cells is to monitor the activity of all cells in the body and destroy any that threaten the integrity of the body. The CD8 cells destroy target cells by releasing cytolytic enzymes, toxic cytokines, and pore-forming molecules or by triggering membrane molecules and intercellular apoptosis. Removal of foreign material from lymph before it enters the blood is the function of lymph nodes.

12. Of the following list of nursing interventions, which would be considered priority when managing a patient with life-threatening myxedematous coma? Select all that apply. A) Administer 3% sodium IV solution to increase sodium levels. B) Administer 50% dextrose to raise glucose levels. C) Place on oxygen therapy to encourage deep breathing. D) Place on a warming bed to raise body temperature. E) Administer sedatives frequently to prevent seizures.

Ans: A, B, C Feedback: Myxedematous coma is a life-threatening, end-stage expression of hypothyroidism. It is characterized by coma, hypothermia, CV collapse, hypoventilation, and severe metabolic disorders that include low sodium, low glucose, and lactic acidosis. Treatment includes aggressive management of precipitating factors; supportive therapy such as management of CV status, hyponatremia, and hypoglycemia; and thyroid replacement therapy. If hypothermia is present, active rewarming is contraindicated because it may induce vasodilation and vascular collapse. Administering sedatives frequently could be harmful since the person is unable to metabolize sedatives, analgesics, and anesthetic drugs.

1. A pediatric unit will be receiving an 8-day-old infant with a suspected congenital renal disorder. Which of the following renal abnormalities could be the possible cause? Select all that apply. A) One of the infant's kidneys may have failed to develop normally. B) The kidneys may be misshapen and have cysts present. C) The upper or lower poles of the two kidneys may be fused. D) Renal cell carcinoma may be present. E) Urine-filled dilation of renal pelvis associated with atrophy of the kidney may be present.

Ans: A, B, C Feedback: Renal hypoplasia, cystic dysplasia, and horseshoe kidney are more common diagnoses in infants. Renal cell carcinoma is not a congenital condition or one that often manifests in infancy. Urine-filled dilation of renal pelvis associated with atrophy of the kidney is a description of hydronephrosis.

20. The nurse knows that a diabetic patient admitted for a lower limb infection likely is experiencing which of the following pathophysiologic principles listed below? Select all that apply. A) Many diabetic patients have sensory deficits and ignore minor trauma just because they can't feel it in their feet. B) Patients with chronic diabetes may have vascular problems that impair circulation, and therefore cells needed for adequate inflammatory response cannot reach the site. C) Hyperglycemia may provide an environment that enhances the growth of microorganisms. D) Infections happen to all age groups and people, and the diabetic patient has the same risk for developing an infection as other patients without diabetes. E) Any exercise that creates diaphoresis in the feet can cause a fungal infection in diabetic patients.

Ans: A, B, C Feedback: Suboptimal response to infection in a person with diabetes is caused by the presence of chronic complications, such as vascular disease and neuropathies, and by the presence of hyperglycemia and altered neutrophil function. Certain types of infections occur with increased frequency in people with diabetes: soft tissue infections of the extremities, osteomyelitis, UTIs and pyelonephritis, candidal infections of the skin and mucous surfaces, dental caries, and periodontal disease.

9. Which of the following clinical manifestations would lead the nurse to suspect the renal failure patient is developing uremia? Select all that apply. A) Weakness and fatigue B) Lethargy and confusion C) Extreme itching D) Blood in urine E) Urine smell in the stool

Ans: A, B, C Feedback: Uremia affects all body systems. The symptoms at the onset include weakness, fatigue, nausea, and apathy. These are subtle signs. More severe symptoms include extreme weakness, frequent vomiting, lethargy, and confusion.

3. A nurse who works in the office of an endocrinologist is orienting a new staff member. Which of the following teaching points is the nurse justified in including in the orientation? Select all that apply. A) ìA bodily process can be the result of the combined effect of several different hormones from different sources.î B) ìA single hormone can act not only on one process or organ but often on several different locations or processes.î C) ìIt's common for production of hormones to be far removed from the tissue where they ultimately exert their effect.î D) ìSometimes hormones act locally on the area where they were produced, like in the case of paracrine and autocrine actions.î E) ìThe regulation in homeostasis requires that hormones be absent from the body when their effect is not needed.î

Ans: A, B, C, D Feedback: A single hormone can exert various effects in different tissues or, conversely, a single function can be regulated by several different hormones. Hormones act both distant from their source and more locally, as in the case of autocrine and paracrine actions. Hormones are normally present at all times.

16. While a travel during a vacation via car, the mother notes that her 14-year-old child is getting sick. The mother suspects motion sickness. Which of the following clinical manifestations would confirm this diagnosis? Select all that apply. A) Rapid breathing B) Feeling faint C) Red, flushed face D) Rapid pulse rate E) Severe balance problems

Ans: A, B, D Feedback: Autonomic signs (of motion sickness) including lowered BP, tachycardia, and excessive sweating may occur. Hyperventilation produces changes in blood volume and pooling of blood in the lower extremities, leading to postural hypotension and sometimes syncope. Red, flushed face is usually associated with elevated temperature. Severe balance problems are usually associated with irritation or damage of the vestibular end organs.

11. Which of the following individuals would be considered at high risk for developing cataracts? Select all that apply. A) An 88-year-old female with osteoporosis and congestive heart failure B) A 51-year-old female whose rheumatoid arthritis is controlled with oral corticosteroids C) A 50-year-old male who takes nebulized bronchodilators four times daily for the management of his emphysema D) A 39-year-old woman with a history of open-angle glaucoma and poorly controlled diabetes E) A 29-year-old artist who spends long hours in sunlight painting landscapes

Ans: A, B, D, E Feedback: Advanced age, steroid use, and sunlight exposure are all significant risk factors for the development of cataracts. Metabolically induced cataracts are caused by disorders of carbohydrate metabolism (diabetes). Use of bronchodilators is not noted to be strongly associated with cataracts.

14. For several years, a 39-year-old female has been averaging two to three bottles of wine each night after her children go to sleep and has included several ounces of brandy in recent years as well. Despite negative consequences to her career and the dissolution of her marriage, her drinking has culminated in a diagnosis of cirrhosis. Which of the following physical manifestations of the health problem would her care team anticipate? Select all that apply. A) Ascites B) Anorexia C) Fever D) Bleeding tendencies E) Epigastric pain F) Obesity

Ans: A, B, D, E Feedback: Ascites, anorexia, bleeding tendencies due to neurological effects, and epigastric pain are common accompaniments to cirrhosis. Ascites occurs when the amount of fluid in the peritoneal cavity is increased and is a late-stage manifestation of cirrhosis and portal hypertension. Epigastric pain is caused by liver enlargement or peritonitis. The peritoneal fluid is seeded with bacteria from the blood or lymph or from passage of bacteria through the bowel wall. Because factors V, VII, IX, and X, prothrombin, and fibrinogen are synthesized by the liver, their decline in liver disease contributes to bleeding tendencies. Fever and obesity would be less likely to exist, as the pathology is not infectious in nature and malnutrition and impaired food metabolism are common.

12. A female client with suspected glomerular disease has been referred to a nephrologist. The nurse knows that which of the following clinical manifestations may be present with the diagnosis of acute nephritic syndrome? Select all that apply. A) Sudden onset of hematuria B) Proteinuria C) Flank pain D) Excess urine output E) Edema

Ans: A, B, E Feedback: In its most dramatic form, the acute nephritic syndrome is characterized by sudden onset of hematuria, variable degrees of proteinuria, diminished GFR, oliguria, and signs of impaired renal function. Inflammatory processes damage the capillary wall. This damage to the capillary wall allows RBCs to escape into the urine and produce a decrease in GFR. Extracellular fluid accumulation, hypertension, and edema develop because of the decreased GFR. Flank pain is usually associated with kidney stones. Oliguria occurs rather than excess urine output.

12. In the ICU, a patient has been diagnosed with sepsis due to a bacterial invasion. The human body usually responds to infections by developing an uncontrolled inflammatory response with large production and release of inflammatory cytokines such as IL-1 and TNF-a. The nurse will note which of the following clinical manifestations in this septic patient as a result of the activation of these cytokines? Select all that apply. A) Excessive interstitial edema related to increased vascular permeability B) Decreased cardiac output resulting from myocardial depression C) Increased respiratory rate with crackles heard throughout all lung fields D) Excessive bleeding from bowels and bladder E) Lower blood pressure due to intravascular fluid loss

Ans: A, B, E Feedback: SIRS (systemic inflammatory response syndrome) can develop as a result of large quantities of microbes entering the blood, resulting in the release of enormous quantities of inflammatory cytokines. They cause generalized vasodilation (hypotension), increased vascular permeability (fluid loss into the tissues), intravascular fluid loss (dehydration with low urine output and low BP), myocardial depression (decreased cardiac output), and circulatory shock.

9. Of the following patient conditions, which patients would be at risk for experiencing a thyroid problem due to a decrease in thyroxine-binding globulin (TBG)? Select all that apply. A) A 55-year-old male with cirrhosis due to alcohol abuse B) A 47-year-old female experiencing hot flashes and excess diaphoresis related to menopause C) A 75-year-old man receiving chronic glucocorticoid therapy to treat his severe chronic obstructive pulmonary disease (COPD) D) A 18-year-old female anorexia nervosa patient weighing 78 lb and has consumed no protein for the past 3 years

Ans: A, C, D Feedback: A number of disease conditions and pharmacologic agents can decrease the amount of binding protein in the plasma or influence the binding of hormone. Glucocorticoid medications and systemic disease conditions such as protein malnutrition, nephritic syndrome, and cirrhosis decrease TBG concentrations.

17. A 54-year-old male who lives alone and has poorly controlled type 1 diabetes has been admitted to hospital for treatment of sepsis resulting from a chronic foot ulcer. Which of the other findings from the man's admission assessment and history would his care team likely to attribute to his diabetes? Select all that apply. A) The client complains of bloating after eating a meal and states that he is unable to eat much food at any one sitting. B) The man's resting heart rate is noted to be irregular. C) He states that his bowel movements are almost always loose. D) The client states that he has been unable to maintain an erection in recent years. E) Crackles are audible in his lower lung fields bilaterally. F) The man's temperature is 38.2∞C (100.8∞F) orally.

Ans: A, C, D Feedback: Gastroparesis, diarrhea, and erectile dysfunction are frequent manifestations of diabetes, while cardiac dysrhythmias, pulmonary edema, and fever are not.

20. When educating the patient about possible treatments following surgery for bladder cancer, the nurse might include which of the following chemotherapy options? Select all that apply. A) Intravesical chemotherapy with doxorubicin (Adriamycin) B) Intravenous chemotherapy with at least three agents C) Bacillus Calmette-GuÈrin (BCG) vaccine D) Endocan, a tumor angiogenesis inhibitor

Ans: A, C, D Feedback: No chemotherapeutic regimens for bladder cancer have been established. Instillation of chemotherapeutic drugs into the bladder is currently done using thiotepa, mitomycin C, and doxorubicin. BCG vaccine causes a significant reduction in the rate of relapse and prolongs relapse-free intervals in people with cancer in situ. Inhibitors of tumor angiogenesis and inhibitors of EGF drugs are proving effective with bladder cancer.

13. Which of the following patients would be considered high risk for developing papilledema? Select all that apply. A) A 2-year-old patient who has a shunt placed following delivery where he was diagnosed with hydrocephalus B) A 55-year-old male with substernal chest pain radiating down both arms and experiencing nausea C) A 43-year-old male with diabetes, renal insufficiency, and BP 200/107 D) A 25-year-old motorcyclist who was in an accident and has a potential subdural hematoma E) An 18-year-old female complaining of severe cramps with her menstrual bleeding

Ans: A, C, D Feedback: The most common conditions causing increased intracranial pressure include cerebral tumors, subdural hematoma, hydrocephalus, and malignant hypertension. Possible MI is not one of the causes of increased intracranial pressure.

8. Tumor necrosis factor-a and IL-1 are major cytokines that mediate inflammation. If the patient is developing a systemic response to an infection, the nurse will likely assess which of the following clinical manifestations? Select all that apply. A) Elevated temperature B) Hypertension C) Tachycardia D) Decrease in urine output E) Anorexia

Ans: A, C, E Feedback: IL-1 and TNF-a are mediators of the acute-phase responses associated with infection or injury. Features of these systemic responses include fever (elevated temperature), hypotension, tachycardia (increased heart rate), anorexia, increase in neutrophil count, and increased levels of corticosteroid hormones.

2. Which of the following clients would be considered to have a significant risk of developing the prerenal form of acute renal failure? Select all that apply. A) A 22-year-old male who has lost large amounts of blood following a workplace injury B) A 41-year-old female who is admitted for intravenous antibiotic treatment of pyelonephritis C) A 79-year-old male with diagnoses of poorly controlled diabetes mellitus and heart failure D) A 20-year-old male who is admitted for treatment of an overdose of a nephrotoxic drug E) A 68-year-old male with a diagnosis of benign prostatic hyperplasia (BPH) F) An 80-year-old female who has been admitted for the treatment of dehydration and malnutrition

Ans: A, C, F Feedback: Hemorrhage, heart failure, and dehydration (hypovolemia) are all noted contributors to prerenal failure. Pyelonephritis and damage from nephrotoxic drugs would more likely result in intrinsic renal failure, while BPH is postrenal in nature.

8. A client has been diagnosed with having calcium oxalate kidney stones following intravenous pyelography. Which of the following teaching points about the treatment of the health problem are justifiable? Select all that apply. A) ìYou may need to cut out cocoa, chocolate, and some nuts from your diet.î B) ìIt's important that you avoid high-calcium foods like milk, cheese, and yogurt.î C) ìWe will come up with a plan to safely limit your fluid intake over the next few weeks.î D) ìExtracorporeal shock-wave lithotripsy treatment may be used to fragment larger stones.î E) ìMost likely your stones can be dissolved by medications over the next several days.î

Ans: A, D Feedback: Individuals with calcium oxalate stones often need to avoid high-oxalate foods like nuts, cocoa, and chocolate. Extracorporeal shock-wave lithotripsy treatment may be used to fragment larger renal calculi. It would not be necessary to avoid calcium intake, and fluid intake should encouraged, not curbed. Medications can reduce the potential for stone formation but are not a common treatment modality.

3. While explaining to a group of nursing students what the function of the first mucosal layer of the lower two thirds of the esophagus, the pathophysiology instructor mentions which of the following functions? Select all that apply. A) Secretion of mucus to lubricate and protect the inner surface of the alimentary canal B) Smooth muscle cells that facilitate movement of contents of the GI tract C) Holding the organs in place and storage of fats D) Barrier to prevent the entry of pathogenic organisms E) A cushioning to protect against injury from sports or car accidents

Ans: A, D Feedback: The first layer performs numerous functions. These include production of mucus that lubricates and protects the inner surface of the alimentary canal; secretion of digestive enzymes and substances that break down food; absorption of the breakdown products of digestion; and maintenance of a barrier to prevent the entry of noxious substances and pathogenic organism. The facilitation of movement of contents of the GI tract occurs in the third layer while holding the organs in place, and storage of fats occurs in the fourth layer.

4. A 4-year-old boy has had otitis media with effusion (OME) for several weeks, and his condition has recently progressed to acute otitis media (AOM). Which of the following factors could have contributed to his AOM? Select all that apply. A) Reflux of fluid from the boy's nose into his middle ear B) A deficiency in immunoglobulin M C) Accumulation of cerumen in the external acoustic meatus D) Sensorineural deficits in the auditory control apparatus E) Exposure to respiratory virus

Ans: A, E Feedback: Reflux via the eustachian tubes, IgG deficiency, and exposure to RSV have all been implicated in the development of AOM. Cerumen accumulation in the outer ear, deficient IgM, and sensorineural deficits are unlikely to contribute to AOM.

20. Which of the following clinical findings among older adults is most unlikely to warrant further investigation and possible treatment? A) An 81-year-old male's serum creatinine level has increased sharply since his last blood work. B) A 78-year-old female's GFR has been steadily declining over several years. C) A 90-year-old female's blood urea nitrogen (BUN) is rising. D) An 80-year-old male whose urine dipstick reveals protein is present.

Ans: B Feedback: A gradual decrease in GFR is considered a normal age-related change. Increased creatinine or BUN would warrant follow-up, as would the presence of protein in a client's urine.

12. A patient has been admitted with diabetic ketoacidosis. The emergency department starts an IV to improve circulatory volume. If there is a sudden change in extracellular fluid osmolality that results in a too rapid blood glucose lowering, the nurse will likely observe which of the following clinical manifestations? A) Chills, profuse sweating, weakness B) Headaches, dizziness, change in level of consciousness C) Increase urine output, thirst, vomiting D) Rapid, deep respirations, palpitations, low BP

Ans: B Feedback: A sudden change in the osmolality of extracellular fluid can also occur when blood glucose levels are lowered too rapidly, and this can cause cerebral edema, more common in children than in adults. Answers A, B, and C all have some component of signs and symptoms of DKA rather than cerebral edema signs and symptoms (answer B).

17. Which of the following statements best captures an aspect of the function of the hypothalamicñpituitaryñadrenal (HPA) system? A) Adrenocorticotropic hormone (ACTH) released by the hypothalamus controls to release of cortisol. B) The pituitary gland communicates with the adrenal cortex through the release of ACTH. C) The adrenal cortex receives corticotrophin-releasing hormone (CRH) and in turn releases cortisol. D) The pituitary gland causes a release of CRH from the hypothalamus, which promotes hormone release from the adrenal cortex.

Ans: B Feedback: ACTH mediates between the anterior pituitary gland and the adrenal cortex in the HPA system. ACTH is released by the pituitary, not the hypothalamus, and CRH acts on the pituitary, not the adrenal cortex. CRH flows from the hypothalamus to the pituitary, not vice versa.

7. A 28-year-old male who is 611 tall has a diagnosis of acromegaly. The man is explaining to a curious but sympathetic coworker exactly what accounts for his extraordinary height. Which of the following explanations demonstrates a sound understanding of his health problem? A) ìMy pituitary gland produced a much higher than normal amount of growth hormone when I was a child.î B) ìA tumor in my brain threw off my hormone levels after I was finished adolescence.î C) ìMy liver is malfunctioning and produces too many of the hormones that ultimately cause growth.î D) ìThe high sugar levels that go along with my diabetes made my pituitary gland overproduce the hormones that cause you to grow.î

Ans: B Feedback: Acromegaly is associated with adult onset and nearly always involves an adenoma. Increased GH as a child and liver dysfunction are not noted contributors to acromegaly. High levels of GH can cause overproduction of insulin and eventual diabetes, but diabetes does not itself lead to acromegaly.

12. A 37-year-old male has survived a logging accident in which the severing of his femoral artery and consequent blood loss resulted in cardiogenic shock. On recovery, one of the deficits that he finds most frustrating is a significant loss of visual acuity. Which is the most likely rationale for his vision damage? A) Decreased cerebral perfusion results in progressive damage to the optic nerve. B) Circulatory collapse causes rapid death of retinal neurons. C) Lack of oxygen results in a distortion of the fovea. D) The visual cortex is susceptible to hypoxic necrosis.

Ans: B Feedback: Acute decreased circulation can result in sight damage from edema and death of retinal neurons. Damage to the optic nerve, the fovea, and the visual center are not likely to be contributing factors.

10. A 43-year-old male client has presented to the emergency department with vomiting that he claims is of a sudden onset. The client also states that the emesis has often contained frank blood in the hours prior to admission. His vital signs are stable with temperature 98.3∞F, pulse 88, BP 140/87, and respiratory rate 18. Which of the following potential contributing factors would the health care team suspect first? A) Overuse of antacids B) Alcohol consumption C) Staphylococcal enterotoxins D) Effects of Helicobacter pylori

Ans: B Feedback: Acute gastritis associated with alcohol use is characterized by intermittent vomiting and the possibility of hematemesis. Aspirin and H. pylori do not normally cause such an acute symptom onset, and infectious organisms do not normally cause bleeding of the stomach lining. A combination of calcium carbonate and magnesium is commonly found in antacids. Overdose of antacids can result in irregular heartbeat, poor balance, shallow, rapid breathing and stupor (lack of alertness).

20. Which of the following statements best captures an aspect of the process of fat digestion and absorption? A) Ingested triglycerides are broken down into absorbable form by gastric lipase. B) Triglycerides are digested with the aid of bile salts. C) Long-chain fatty acids are absorbed directly into the portal blood. D) Stool is not excreted until all fat is absorbed.

Ans: B Feedback: After breakdown, by pancreatic lipase, triglycerides are absorbed primarily in the upper jejunum. Long-chain fatty acids are absorbed less easily than medium-chain triglycerides, while stool often contains a certain amount of fat.

11. A male elementary school student has a severe allergy to peanuts and is displaying the signs of anaphylactic shock after inadvertently eating a peanut-containing candy bar. Which of the following statements best captures the boy's current status and preferred treatment? A) He is experiencing shortness of breath caused by potent vasoconstriction that can be relieved by epinephrine injection. B) He is approaching vascular shock and developing edema due to actions of IgE antibodies, situations that can be reversed by administration of epinephrine. C) His mast cells and basophils have been sensitized, but systemic effects can be mitigated by administration of bronchodilators. D) He is likely in a primary- or initial-stage allergic response that can be relieved by antihistamine administration.

Ans: B Feedback: Anaphylactic reactions are often accompanied by vascular shock and edema, and the normal treatment is epinephrine injection. Symptoms are caused by vasodilation, not vasoconstriction; bronchodilators are not the ideal treatment.

9. While working at the triage desk in the local emergency department, which of the following patients is likely having a medical emergency and needs to be seen first? A) A 17-year-old high school student who has a red, itchy eye B) A 55-year-old truck driver complaining of sudden onset of ocular pain and blurred vision C) A 45-year-old school teacher complaining of a red eye that is draining yellow secretions D) An infant with red eyes who is irritable and refusing to eat

Ans: B Feedback: Attacks of increased intraocular pressure are manifested by ocular pain and blurred vision caused by corneal edema. Acute angle-closure glaucoma is an ophthalmic emergency. Treatment is directed at reducing the intraocular pressure, usually with pharmacologic agents.

11. Which of the following clients' diagnostic blood work is most suggestive of chronic kidney disease (CKD)? A) A client with high pH; low levels of calcium; and low levels of phosphate B) A client with low vitamin D levels; low calcitriol levels; and elevated parathyroid hormone (PTH) levels C) A client with low bone density; low levels of calcium; and low levels of phosphate D) A client with low potassium levels; low calcitriol levels; and increased PTH levels

Ans: B Feedback: CKD is associated with low vitamin D and calcitriol levels, which induces increased PTH production. CKD is also associated with acidosis (low pH), high levels of phosphate, and hyperkalemia.

19. The mother of a 19-week-old infant has brought her baby in for assessment to a pediatrician because of the baby's persistent weight loss and diarrhea. An intestinal biopsy has confirmed a diagnosis of celiac disease, and the child's mother is anxious to know what caused the disease. Which of the following aspects of the etiology of celiac disease would underlie the explanation that the physician provides? A) Bacterial or chemical invasion of the peritoneum leads to decreased nutrient absorption and transport. B) An inappropriate T-cellñmediated response results in increased levels of antibodies and an inflammatory response. C) Neurogenic or muscular inhibition of peristalsis results in inappropriate motility of ingested food in the lower small intestine and the colon. D) Inability to process or absorb the fat content of breast milk results in malnutrition and deficiency of fat-soluble vitamins.

Ans: B Feedback: Celiac disease is rooted in an inappropriate immune response that initiates an inflammatory response, resulting in loss of absorptive villi. Bacterial or chemical invasion of the peritoneum is associated with peritonitis, while inhibition of peristalsis is associated with obstructions. An inability to process or absorb fat is associated with malabsorption syndrome.

15. Which of the following individuals is most likely to be diagnosed with a central vestibular disorder? A) A man who got up quickly from his bed and sustained an injury after he ìblacked outî B) A woman who has ongoing difficulty in balancing herself when walking C) A woman who suffered a loss of consciousness after being struck on the head during a soccer game D) A man who states that he feels car sick whenever he rides in the back seat of a vehicle

Ans: B Feedback: Central vestibular disorders are marked by a sensation of motion that interferes with balance but that is mild and constant and chronic in duration. It should be differentiated from postural hypotension, loss of balance from a head injury, or motion sickness.

7. Which of the following pain descriptions would lead the nurse to suspect the client is experiencing ureteral colic? A) Right upper quadrant pain that worsens with deep breaths and palpation B) Excruciating pain in the flank and upper outer quadrant of the abdomen that radiates to the bladder area C) Pain described as ìfire poking in their side,î pulsating with every heart beat but decreases when in fetal position D) Perineal pain that increases when urinating and then lessens until the time to urinate again

Ans: B Feedback: Classic ureteral colic is manifested by acute, intermittent, and excruciating pain in the flank and upper outer quadrant of the abdomen on the affected side. The pain may radiate to the lower abdominal quadrant, bladder area, perineum, or scrotum in the man.

2. The father of a third grade girl has brought his daughter to a walk-in clinic because he believes the girl has pink eye, which has been going around the students in her class. The nurse at the clinic concurs with the father's suspicion of conjunctivitis. Which follow-up explanation by the nurse is most accurate? A) ìThe insides of her eyelids have become infected. This often produces severe discomfort.î B) ìThe surfaces of her eyes have bacteria or a virus established, and it's important to maintain good hand hygiene until it goes away.î C) ìAn antibiotic ointment will likely resolve her infection, but pain control will be necessary in the mean time.î D) ìIt's important to aggressively treat this in children, since damage to her sight can result if it's not treated.î

Ans: B Feedback: Conjunctivitis often spontaneously resolves. The pain associated with conjunctivitis usually produces only mild discomfort compared with severe discomfort associated with corneal lesions or deep and severe pain associated with acute glaucoma. Conjunctivitis may spread to other family members. The corneal surface is not primarily involved, and pain that is severe suggests corneal involvement rather than conjunctivitis. Sight damage is not likely to result.

16. Following the identification of low blood levels of cortisol and low 24-hour urinary free cortisol, a 51-year-old female client has been diagnosed with a primary adrenal cortical insufficiency. Which of the following health consequences would be attributable to her low levels of cortisol? A) Visible exophthalmos B) Impaired immunological and inflammatory response C) Diminished secondary sex characteristics D) Insufficient regulation of serum potassium and sodium levels

Ans: B Feedback: Cortisol plays a central role in the normal functioning of the immune response and inflammation. Exophthalmos is associated with Graves disease, and secondary sex characteristics are functions of adrenal androgens. Potassium and sodium are regulated by mineralocorticoids.

10. Following a serious bout of bacterial meningitis, the parents of a 14-month-old has noted the child is not responding to verbal commands. The nurse will explain the pathophysiologic principle behind this by educating the patients by which of the following statements? A) ìThis could be caused by the same organism that caused the meningitis, infecting the child's tympanic membrane.î B) ìThis may be due to a loss of hair cells and damage to the auditory nerve.î C) ìThe ear and the lining of the brain that was infected are all connected together.î D) ìIt is common for meningitis to use up all the natural killer cells and therefore increase the risk of having brain tumors develop.î

Ans: B Feedback: Deafness or some degree of hearing impairment is the most common serious complication of bacterial meningitis in infants and children. The mechanism causing hearing impairment seems to be suppurative labyrinthitis or neuritis resulting in the loss of hair cells and damage to the auditory nerve. There is no direct connection between the meninges of the brain and the tympanic membrane. Bacterial meningitis is not associated with an increased risk of developing a brain tumor.

4. Which of the following patients is most likely to benefit from transplantation of thymic tissue or major histocompatibility complex (MHC)-compatible bone marrow? A) A 12-year-old girl with a history of epilepsy and low IgG levels secondary to phenytoin use B) A 7-year-old boy whose blood work indicates decreased IgA and IgG with increased IgM C) A 6-year-old boy whose pre-B cells are incapable of translation to normal B cells D) A 9-year-old girl who has a diagnosis of IgA deficiency

Ans: B Feedback: Decreased IgA, IgE, and IgG with increased IgM levels are characteristic of X-linked immunodeficiency with hyper-IgM, a primary cell-mediated immunodeficiency that would respond to thymic tissue transplantation and MHC-compatible bone marrow. Low IgG levels secondary to phenytoin use, X-linked hypogammaglobulinemia, and selective IgA deficiency are unlikely to be treated with the T-cellñfocused treatments like thymic tissue transplantation and MHC-compatible bone marrow.

13. A 24-year-old woman presents with fever and painful, swollen cervical lymph nodes. Her blood work indicates neutrophilia with a shift to the left. She most likely has A) a mild parasitic infection. B) a severe bacterial infection. C) a mild viral infection. D) a severe fungal infection.

Ans: B Feedback: Fever and painful, palpable lymph nodes are nonspecific inflammatory conditions; leukocytosis is also common but is a particular hallmark of bacterial infection. Neutrophilia also indicates a bacterial infection, whereas increased levels of other leukocytes would indicate other etiologies. The shift to the leftññthe presence of many immature neutrophilsññindicates that the infection is severe, because the demand for neutrophils exceeds the supply of mature cells.

11. A patient has recently been diagnosed with H. pylori gastritis. The nurse knows that this form of gastritis is usually treated with a combination of an antibiotic and A) antianxiety medications. B) proton pump inhibitors. C) lactulose, to reduce the blood ammonia levels. D) calcium carbonate, an antacid.

Ans: B Feedback: H. pylori is associated with an increased risk of gastric adenocarcinoma, gastric atrophy, and peptic ulcer. It is less likely to contribute to IBD, esophagitis, or diverticular disease. Eradication of H. pylori is difficult. Treatment requires a combination therapy that includes the use of antibiotics and a proton pump inhibitor. The proton pump inhibitors have direct antimicrobial properties against H. pylori. Antianxiety medications will not kill the bacteria. H. pylori is not associated with elevated blood ammonia levels. Calcium carbonate is usually given to relieve heartburn caused by GERD.

7. A nurse has just learned that her child has a life-threatening complement disorder known as hereditary angioneurotic edema (HAE). Due to deficiency in C1-INH, the nurse needs to be prepared for which possible life-threatening clinical manifestation? A) Bulging eyeballs B) Swelling of the airway C) Compressed carotid arteries D) Compression of brachial nerves

Ans: B Feedback: HAE is a rare, life-threatening complement disorder that results from deficiency of C1-inhibitor (HAE-C1-INH). It is an inherited autosomal dominant trait that causes mutation in the 11th chromosome. Deficiencies in C1-INH result in uncontrolled release of vasoactive substances that promote vascular permeability. The net result is the development of swelling in the subcutaneous tissues of the extremities, face/torso, or upper airway and GI tract. Laryngeal edema is a life-threatening manifestation that can lead to complete airway obstruction and death without interventions.

10. A male patient comes to the clinic asking to speak to a health care provider privately. He reveals that he had shared a needle/syringe with a prostitute (shooting up cocaine) and then had unprotected sex. Upon questioning, it was revealed that the patient had not had any immunization for hepatitis B. Which of the following medications would the nurse anticipate administering today to this patient? A) Tenofovir disoproxil fumarate plus emtricitabine B) Hepatitis B immunoglobulin (HBIG) C) Hepatitis C immunoglobulin (HCIG) D) Hepatitis A vaccine

Ans: B Feedback: HBIG is used as an adjunct to hepatitis B vaccine for postexposure immunoprophylaxis to prevent HBV infections in high-risk populations. Tenofovir disoproxil fumarate plus emtricitabine is for HIV preexposure coverage. There is no hepatitis C vaccine available on the market yet. Hepatitis A vaccine is for HAV, which is usually spread from fecalñoral routes.

1. While teaching about HIV/AIDS to a group of high school seniors, the school health nurse will begin by explaining the basic facts that will likely include which of the following information? A) Like all viruses, HIV is a genetic material made from DNA with long molecules that carry genetic information. B) HIV is different from other viruses since it is a retrovirus that selectively attacks the body's immune cells. C) There are two types of HIV, but the one that is endemic to the United States is HIV type 2. D) HIV type 1 for some reason rarely develops into full-blown AIDS.

Ans: B Feedback: HIV is a retrovirus that selectively attacks the CD4+ T lymphocytes, the immune cells responsible for orchestrating and coordinating the immune response to infection. It must change from RNA to DNA through a series of stages in order to get in a cell and begin replication. HIV type 2 is endemic in West Africa but is rarely seen in other parts of the world. People with HIV-2 tend not to develop AIDS.

16. A student states, ìIt seems like helper T cells do a lot more than just 'help' the cellular immunity process.î Which of the following responses listed below best conveys an aspect of the role of CD4+ helper T cells in immunity? A) ìWithout helper T cells, no antigens would be presented.î B) ìHelper T cells play a major role in stimulating and regulating the whole process.î C) ìWithout helper T cells, the wrong antibodies would end up being produced.î D) ìHelper T cells are key to the hematopoiesis that produces all the components of the immune system.î

Ans: B Feedback: Helper T cells are central to the regulation, proliferation, and stimulation of the immune system. They do not play a central role in antigen presentation or early hematopoiesis, however. Their absence would not result in incorrect antibody production, but rather insufficient or absent immune response.

4. A 30-year-old woman has sought care because of her recurrent photophobia, tearing, and eye irritation. During assessment, her care provider asks about any history of cold sores or genital herpes. What is the rationale for the care providerís line of questioning? A) Herpes simplex virus (HSV) conjunctivitis indicated a need for antiviral rather than antibacterial treatment. B) HSV infection of the cornea is a common cause of corneal ulceration and blindness. C) Chronic viral infection of the eyes can result in HSV autoinoculation of the mouth and labia. D) A history of HSV with eye irritation is suggestive of glaucoma.

Ans: B Feedback: Herpes simplex virus (HSV) keratitis (not conjunctivitis) with stromal scarring is the most common cause of corneal ulceration and blindness in the Western world. Autoinoculation from the eyes to other sites is not common, and glaucoma is not noted to be a consequence or symptom of HSV infection.

13. Which of the following clients is most clearly displaying the signs and symptoms of irritable bowel disease (IBD)? A) A 32-year-old mother who complains of intermittent abdominal pain that is worse during her menstrual period B) A 51-year-old male who states that his stomach pain is in his lower abdomen, ìcomes and goes,î and ìfeels more like a cramp than a dull acheî C) A 44-year-old man who works the evening shift at a factory and who states that his lower abdominal pain is much worse at night than during the day D) A 24-year-old man who has a stressful job but whose diarrhea and cramping do not worsen during periods of high stress

Ans: B Feedback: IBD is commonly manifested as intermittent lower abdominal pain that feels like cramping. Defecation normally relieves the pain, and symptoms are normally not present at night or during sleep. Stress commonly exacerbates symptoms.

14. When counseling a male patient with suspected HIV, the nurse informs him that if the enzyme-linked immunosorbent assay (ELISA) comes back positive, then A) no further testing is required since this confirms HIV infection. B) a second test known as the Western blot assay will be ordered to confirm positive HIV status. C) he will be sent to an infectious disease physician for a tissue biopsy to confirm infection. D) if the second test, the Western blot, returns negative, he has not developed a case of full-blown AIDS.

Ans: B Feedback: If ELISA is positive, his blood sample is then sent for Western blot assay. If the Western blot is positive, diagnosis of HIV is confirmed. If the Western blot is negative, then the person is not infected with HIV.

10. If the anteriorñposterior dimension of the eyeball is too long, the focus point for an infinitely distant target is anterior to the retina. This patient would be diagnosed as having A) hyperopia. B) myopia. C) cycloplegia. D) presbyopia.

Ans: B Feedback: If the anteriorñposterior dimension of the eyeball is too long, the focus point for an infinitely distant target is anterior to the retina. This condition is called myopia or nearsightedness. People with myopia can see close objects without problems. Hyperopia is farsightedness. Cycloplegia is paralysis of the ciliary muscle, with loss of accommodation. Presbyopia refers to a decrease in accommodation that occurs because of aging.

8. A nurse is collecting a urine specimen prior to measuring the albumin level in a clientís urine. A colleague questions the rationale for the test, stating, ìI thought albumin was related to liver function, not kidney function.î How can the nurse best respond to this statement? A) ìUrine should normally be free of any proteins, and albumin is one of the more common proteins to be excreted in chronic renal failure.î B) ìUrine albumin levels are useful for diagnosing diabetic kidney disease.î C) ìA urine dipstick test will tell us exactly how much albumin is being spilled by the client's kidneys.î D) ìA urine test for albumin allows us to estimate the client's GFR quite accurately.î

Ans: B Feedback: In clients with diabetes, albumin tests are useful adjunctive test of nephron injury and repair. Urine is not normally completely free of proteins, and a urine dipstick does not allow for the quantification of how much albumin is in a sample. Albumin tests do not allow for an accurate indirect indication of GFR.

17. During the follicular stage of menstruation, increased estradiol production causes an increase in FSH production. This increase in FSH production by the anterior pituitary gland will have what effect on the follicle? A) The follicle will continue to grow until it can no longer stay in its membrane. B) The follicle will die, which results in a fall of FSH. C) The follicle will continue to grow and produce estradiol. D) The follicle will secrete additional hormones to attract swimming sperm.

Ans: B Feedback: In positive feedback control, rising levels of a hormone cause another gland to release a hormone that is stimulating to the first. Increased estradiol production during the follicular stage of the menstrual cycle causes increased FSH production by the anterior pituitary gland. This stimulates further increases in estradiol levels until the demise of the follicle.

2. Following destruction of the pituitary gland, ACTH stimulation stops. Without ACTH to stimulate the adrenal glands, the adrenals' production of cortisol drops. This is an example of which type of endocrine disorder? A) Primary B) Secondary C) Tertiary D) Somatic

Ans: B Feedback: In secondary disorders of endocrine function, the target gland is essentially normal, but defective levels of stimulating hormones or releasing factors from the pituitary system alter its function.

7. Which of the following patients would be considered to be in the latent period of HIV infection? A) A 16-year-old prostitute who has open sores on her labia that drain purulent secretions B) A 33-year-old heroin drug abuser who has numerous enlarged lymph nodes in his axilla and cervical neck region for the past 4 months C) A 45-year-old alcohol abuser who is complaining of excessive vomiting of blood that started 2 weeks ago D) A 24-year-old college student who has developed a chronic cough that will not go away, even after taking two courses of antibiotics.

Ans: B Feedback: In the latent period, which can last up to 10 years, the CD4+ count falls gradually to approximately 200 cells/µL. Some people experience swollen lymph nodes that are chronically swollen for more than 3 months in at least two locations, not including the groin. The lymph nodes may be sore or visible externally.

14. Three days ago, a mother delivered her full-term infant who had been identified as having an in utero infection. The infant is receiving antibiotic and phototherapy, and the mother is breast-feeding. Which of the following types of immunoglobulins could most reasonably be expected to predominate in the infant's immune system? A) IgA, IgM, IgD B) IgG, IgA, IgM C) IgE, IgG, IgD D) IgM, IgD, Igm

Ans: B Feedback: Infants are born with IgG from transfer across the placenta, while IgA is found in colostrum. IgM is indicative of an in utero infection.

18. A 22-year-old student has developed a fever and diarrhea while on a backpacking trip in Southeast Asia. His oral temperature is 101.4∞F. The diarrhea is bloody, frequent, and small in volume. These clinical manifestations are sufficiently distressing that he is visiting a local medical clinic in the area. Which of the following diagnoses best characterizes this health problem? A) Noninflammatory diarrhea B) Inflammatory diarrhea C) Factitious diarrhea D) Secretory diarrhea

Ans: B Feedback: Inflammatory diarrhea is often characterized by small-volume diarrhea that is bloody and accompanied by a fever. Noninflammatory diarrhea is normally larger in volume and not bloody. Factitious diarrhea is normally attributable to laxative use, and secretory diarrhea is associated with increased secretory processes of the bowel; neither is likely to produce bloody stool.

17. A female dental assistant has developed signs and symptoms of a latex sensitivity and is undergoing allergy testing as well as blood work. Which of the following components of the assistant's blood work would most likely be the focus of her health care provider's analysis? A) Analysis of class II MHC antigens B) Serum IgE immunoassays C) Serum B-lymphocyte levels D) Serum CD8+ levels

Ans: B Feedback: Latex sensitivity can be either a type I or type IV reaction. Though Th1 levels are relevant in a type IV reaction, IgE analysis is the most common component of relevant blood work. MHC and CD8+ levels are unlikely to be considered.

11. A 40-year-old man who uses heroin intravenously was diagnosed with hepatitis C (HCV) 1 year ago and is now considered to have chronic viral hepatitis. Which of the following statements by the client to his care provider would warrant correction? A) ìI know the medications to treat this aren't fantastic, but at least there are some options for controlling the virus.î B) ìIt's at least a bit reassuring that my liver isn't undergoing damage when I'm not experiencing symptoms.î C) ìEven though I'm sick, at least I won't feel sick most of the time.î D) ìI'm not looking forward to all the side effects of the drug treatments for my HCV, but I hope I don't end up needing a liver transplant.î

Ans: B Feedback: Liver damage persists both during symptomatic and asymptomatic periods of acute viral hepatitis. Medications do exist for treatment of the disease, and asymptomatic periods are more common than symptomatic episodes. Side effects of drug treatment are common, and transplant is a potential end-stage treatment option.

3. Following a long history of fatigue, weakness, and poor appetite, a 39-year-old male has been diagnosed with hypopituitarism. Which of the following clinical findings would most likely cause his care team to suspect that the man has an additional endocrine disorder from a different source? A) The man has a low sperm count and has been unable to have children. B) The man has a chronic platelet deficiency and is occasionally anemic. C) The client is 52 tall and was consistently short for his age as a child. D) The man displays the signs and symptoms of hypothyroidism.

Ans: B Feedback: Low platelets and low hemoglobin are unlikely to be a manifestation of hypopituitarism. A low sperm count, small stature, and hypothyroidism are all noted manifestations of pituitary hypofunction.

5. The nurse knows which of the following components listed below is needed for long-lasting immunity in a client with a diagnosis of sepsis without the causative agent identified? A) Neutrophils B) Lymphocytes C) Colony-stimulating factors D) Natural killer cells

Ans: B Feedback: Lymphocytes provide lifelong immunity and an antigen-specific response to harmful microorganisms. Neutrophils, macrophages, and natural killer cells do not provide this.

8. A 51-year-old woman diagnosed with a cerebrovascular accident (CVA) 5 months prior is distressed that she has had several recent episodes of urinary incontinence. She has asked her nurse practitioner why this is the case. Which of the following statements best captures the fact that would underlie the nurse's response to the client? A) Neurological diseases like MS often result in flaccid bladder dysfunction. B) She may be unable to sense her bladder filling as a result of her MS. C) Lesions to the basal ganglia or extrapyramidal tract associated with MS inhibit detrusor contraction. D) Pathological reductions in bladder volume brought on my MS necessitate frequent micturition.

Ans: B Feedback: MS may result in neurogenic bladder characterized by an inability to sense filling and consequent incontinence. She is not demonstrating the signs of a flaccid bladder, and lesions to the basal ganglia or extrapyramidal tract are associated with Parkinson disease, not MS. Her disease is unlikely to directly reduce bladder volume.

17. A nurse in an acute medical unit of a hospital has admitted a 62-year-old female from the emergency department who has been diagnosed with acute pyelonephritis. Which of the following statements most accurately conveys an aspect of the knowledge base that the nurse needs to perform adequate care and teaching? A) Most cases of acute pyelonephritis are attributable to poorly controlled hypertension. B) Flank pain, dysuria, and nausea and vomiting are likely assessment findings. C) The infection in the kidney is most likely a manifestation of a systemic infection. D) Imaging tests are likely to reveal scarring and deformation of the renal calices and pelvis.

Ans: B Feedback: Manifestations of acute pyelonephritis include pain, frequency, urgency, dysuria, nausea, and vomiting. Chronic rather than acute pyelonephritis is often caused by hypertension, while most cases are caused by ascending bacteria, not systemic infections. Scarring is more commonly a result of chronic pyelonephritis.

12. A client with a new diagnosis of an endocrine disorder is unclear how the body can control the levels of different hormones over time. Which of the following statements most accurately underlies the dominant regulation process of hormone levels in the body? A) A positive feedback cycle ensures that stable levels of hormones exist in the body over time. B) With input from various sensors, hormone production and release are adjusted based on existing hormone levels. C) The hypothalamus ensures that hormone levels correspond accurately to the diurnal cycle. D) The pituitary gland is genetically programmed to stimulate and inhibit hormone production and/or release based on the needs at different points in the life cycle.

Ans: B Feedback: Most hormone levels are controlled by way of a negative feedback cycle, in which low levels stimulate production and/or release. A positive feedback cycle would not achieve this effect. While some hormones are released on a diurnal schedule, the dominant form of hormone regulation in the body is that of negative feedback. Hormone release is not predetermined by the pituitary gland.

19. A 15-year-old who has just been diagnosed with type 1 diabetes says she read on the Internet that diabetes is the leading cause of acquired blindness among Americans. She asks you if she will lose her sight. In addition to explaining that new treatment technologies are being worked on every day, which of the following would be the most appropriate response? A) Reassure her that only type 2 diabetes is a risk factor for blindness. B) Explain that almost all people with type 1 diabetes do experience some degree of vision loss. C) Tell her to expect that she will begin to lose her eyesight by the time she is 25. D) Tell her there is about a 50%chance that she will suffer some diabetes-related sight loss by the time she is 50.

Ans: B Feedback: Nearly all people with type 1 diabetes and more than 60% of people with type 2 diabetes have some degree of retinopathy. Pregnancy, puberty, and cataract surgery can accelerate these changes.

17. A 70-year-old woman with a diagnosis of benign paroxysmal positional vertigo (BPPV) is receiving teaching from her physician about her diagnosis. The client is eager to avoid future episodes of vertigo and has asked the physician what she can do to prevent future episodes. How can the physician best respond? A) ìUnfortunately there aren't any proven treatments for your condition.î B) ìThere are some exercises that I'll teach you to help reorient your inner ear and prevent vertigo.î C) ìAlthough they involve some risks, there are some options for ear surgery that can prevent future vertigo.î D) ìWe usually don't actively treat BPPV unless it starts to affect your hearing.î

Ans: B Feedback: Nondrug therapies for BPPV using habituation exercises and canalith repositioning are successful in many people. Canalith repositioning involves a series of maneuvers in which the head is moved to different positions in an effort to reposition the free-floating debris in the endolymph of the semicircular canals. Surgery is not a noted treatment option, and even in the absence of hearing loss, treatment is warranted.

19. A mom asks her neighbor, a nurse, why every time she takes her daughter (10 years old) out for ice cream she comes home with a stomachache and then experiences a bout of diarrhea. The nurse is thinking that this girl is experiencing A) anxiety about increasing too many calories. B) a deficiency of lactase. C) gallbladder disease. D) premature peptic ulcer formation.

Ans: B Feedback: People with a deficiency of lactase, the enzyme that breaks down lactose, experience diarrhea when they drink milk or eat dairy products. Doubtful the child is anxious about the calories in ice cream. Gallbladder disease s/s usually occur in relation to high saturated fat intake. Peptic ulcer s/s include bloating, vomiting blood, foul coffee ground stools, etc.

14. Which of the following patients would have a very poor response related to tissue regeneration of his or her injured area? A) A 21-day-old infant undergoing a diaphragmatic hernia repair B) A 54-year-old male who had a massive MI 4 days ago and came to the ED today for treatment C) A 73-year-old female who is undergoing lithotripsy for kidney stones D) A 33-year-old athlete undergoing surgery to repair a torn MCL in his right knee

Ans: B Feedback: Permanent or fixed cells cannot undergo mitotic division. The fixed cells include nerve cells, skeletal muscle cells, and cardiac muscle cells.

12. Which of the following medications would the nurse anticipate being prescribed for the renal failure patient who has hyperphosphatemia? A) Vitamin D (calcitriol) B) Calcium carbonate C) Levothyroxine (Synthroid) D) Sensipar (Cinacalcet)

Ans: B Feedback: Phosphate-binding antacids (aluminum salts, calcium carbonate, or calcium acetate) may be prescribed to decrease the absorption of phosphate from the GI tract.

19. A patient exhibiting problems with his or her thyroid has been scheduled for a radioactive scan. From the following list of patients, what would the nurse question as to whether this would be a safe procedure for this patient? A) An adult patient having an episode of wheezing from allergies B) A young female patient who has been trying to get pregnant C) A middle-aged male patient with uncontrolled type 2 diabetes mellitus D) An elderly patient who has a history of aortic stenosis

Ans: B Feedback: Radioactive iodine therapy is contraindicated in pregnant women because 131I crosses the placenta and can adversely affect the fetal thyroid gland. The other clients would have no contraindication to the substance.

17. A patient who is recovering from burn injuries is discussing his prognosis with a physician. Which of the following teaching points about expectations for healing should the physician include? A) ìOnce your healing is complete, your skin will be just as strong as before your accident.î B) ìYou may find that the scar is a bit smaller than the area of the wound.î C) ìYou'll find that your new tissue is more elastic and fragile than the rest of your skin.î D) ìThe final remodeling phase of healing may last up to 3 months in your case.î

Ans: B Feedback: Scars are often smaller than the original area of the wound. There is nearly always an accompanying loss of strength and elasticity, and remodeling can take over 6 months.

15. Following a spider bite she received while camping, a 20-year-old female presented to the emergency department with rash, edema, and fever and was subsequently diagnosed with serum sickness. Which of the following statements best conveys the physiological rationale for the broad systemic effects of this event? A) The woman is experiencing diffuse tissue necrosis as a consequence of an Arthus reaction. B) Antigenñantibody complexes have been deposited in a variety of locations throughout the body. C) Antibody binding to specific target cell receptors is bringing about a change in cell function. D) Deposited antibodies are activating her complement system.

Ans: B Feedback: Serum sickness is characterized by the deposition of antigenñantibody complexes in blood vessels, joints, heart, and kidney tissue. The deposited complexes activate complement, increase vascular permeability, and recruit phagocytic cells, all of which can promote focal tissue damage and edema. Serum sickness is not synonymous with an Arthus reaction. Antibody binding to specific target cell receptors causing a change in cell function is characteristic of antibody-mediated cellular dysfunction. Serum sickness is not associated with the activation of the complement system.

8. A 68-year-old African American man who has smoked for at least 50 years reports that lately he feels as though food is getting stuck in his throat. At first, this was a problem just with dry food, but now his morning oatmeal is getting ìstuck.î On questioning, he reports drinking at least three alcoholic beverages nearly every day. His problem is most likely A) achalasia. B) squamous cell carcinoma of the esophagus. C) dysphagia secondary to scleroderma. D) gastrointestinal reflux disease.

Ans: B Feedback: Squamous cell carcinoma of the esophagus is the seventh leading cause of cancer death among men, particularly black men; mean age at diagnosis is 67 years. Alcohol and tobacco use are the main risk factors for this cancer, and dysphagia is a common presenting complaint. An esophageal motility disorder involves the smooth muscle layer of the esophagus and the lower esophageal splincter (LES). Achalasia is characterized by difficulty swallowing and regurgitation. GERD (gastroesophageal reflux disease) is a condition that causes the esophagus to become irritated and inflamed. Clients with GERD usually feel a burning in the chest or throat called heartburn. Sometimes, they taste stomach fluid in the back of the mouth.

15. Which of the following statements best captures the role of the adrenal cortex in maintaining homeostasis? A) The adrenal cortex is responsible for the production of epinephrine and norepinephrine that are part of the sympathetic nervous system. B) The adrenal cortical hormones are primarily steroids and sex hormones. C) Redundant, secondary production of adrenal cortical hormones can compensate for the loss of the adrenal glands. D) Normal sexual function is dependent on adequate adrenal cortical function.

Ans: B Feedback: The adrenal cortex is responsible for secreting three types of hormones: the glucocorticoids, the mineralocorticoids, and the adrenal androgens. The adrenal medulla produces epinephrine and norepinephrine, and there are no alternate production sites for adrenal cortical hormones. The adrenal androgens are least responsible for normal sexual function.

6. A gastroenterologist is teaching a group of medical students about the enteric nervous system in preparation for a consult on client who has suffered a spinal cord injury. Which of the physician's teaching points is most accurate? A) ìThe myenteric plexus is responsible for controlling the function of each segment of the intestinal tract.î B) ìThe enteric nervous system is made up of the myenteric and submucosal plexuses; these are located in the wall of the GI tract.î C) ìSympathetic innervation of much of the GI tract occurs by way of the vagus nerve.î D) ìParasympathetic stimulation blocks the release of the excitatory neuromediators and inhibits GI motility.î

Ans: B Feedback: The enteric nervous system consists of the myenteric and submucosal plexuses, which are located within the wall of the gastrointestinal tract. The myenteric plexus is responsible for controlling overall function along the entire length of the gut, while the vagus nerve provides parasympathetic, not sympathetic, innervation. Sympathetic simulation lessens excitatory neuromediators and inhibits GI motility.

1. Which of the following data would a clinician consider as most indicative of acute renal failure? A) Alterations in blood pH; peripheral edema B) Increased nitrogenous waste levels; decreased glomerular filtration rate (GFR) C) Decreased serum creatinine and blood urea nitrogen (BUN); decreased potassium and calcium levels D) Decreased urine output; hematuria; increased GFR

Ans: B Feedback: The hallmark of acute renal injury is azotemia, an accumulation of nitrogenous wastes such as creatinine, urea nitrogen, and uric acid, plus a decrease in the GFR of the kidneys. While pH alterations, edema, electrolyte imbalances, and decreased urine output may accompany acute renal failure, they are all potentially attributable to other pathologies. Creatinine, GFR, and BUN would unlikely rise during renal failure.

12. A 48-year-old man who has been HIV positive for 6 years has just learned that he has been diagnosed with Kaposi sarcoma (KS). Which of the following facts most accurately conveys an aspect of his diagnosis? A) An opportunistic Epstein-Barr virus underlies the man's KS. B) He is likely to have lesions on his skin, mouth, or GI tract. C) Intense pain was probably his first manifestation of KS. D) Heterosexual contact most likely underlies his HIV and subsequent KS.

Ans: B Feedback: The lesions of KS can be found on the skin and in the oral cavity, gastrointestinal tract, and the lungs. More than 50% of people with skin lesions also have gastrointestinal lesions. It is linked with a herpes virus and can often be painless, especially in early stages. Men who have sex with men are at a higher risk of developing KS.

14. Following a meal, a woman's blood glucose level has increased. In addition, her pancreas has increased the amount of insulin produced and released. Which of the following phenomena has occurred? A) Increased hormone level according to a negative feedback mechanism B) Adjustment according to the level of the substance a hormone regulates C) Hormone production and release via the positive feedback cycle D) Hypothalamicñpituitary control of hormone levels

Ans: B Feedback: The level of some hormones is adjusted according to the amount of the substance that they control. In this case, insulin controls glucose levels and would increase in response to the increase in serum glucose that follows a meal. This differs from a negative feedback cycle in which a simple decrease in a hormone level stimulates production and/or release of that hormone. Positive feedback and hypothalamicñpituitary control are not evident in this situation.

14. A 55-year-old man has made an appointment to see his family physician because he has been awakening three to four times nightly to void and often has a sudden need to void with little warning during the day. What is the man's most likely diagnosis and possible underlying pathophysiological problem? A) Stress incontinence due to damage to CNS inhibitory pathways B) Overactive bladder that may result from both neurogenic and myogenic sources C) Overactive bladder due to intravesical pressure exceeding urethral pressure D) Overflow incontinence that can result from displacement of the angle between the bladder and the posterior proximal urethra

Ans: B Feedback: The man's complaints are typical of overactive bladder, a condition that can result from the interaction of both the nervous control of bladder emptying and the muscles of the bladder itself. His symptoms are not characteristic of stress incontinence, and when intravesical pressure exceeds, urethral pressure overflow incontinence results. The angle between the bladder and the posterior proximal urethra is more commonly a factor in the continence of females.

5. An 8-week-old boy has been recently diagnosed with a severe combined immunodeficiency (SCID). His parents have performed a significant amount of research on the Internet and have brought a large amount of material to discuss with their care provider. Which of the following statements best reflects an accurate understanding of their son's health situation? A) ìWe read that gene therapy could cure our son; we'd like you to look into that option.î B) ìOur son likely has a deficiency of B lymphocytes and can't produce antibodies.î C) ìWe feel guilty, because dietary and environmental factors have been shown to contribute to SCIDî D) ìThe antibodies that our son produces are mismatched to the infections that he was born with and encounters.î

Ans: B Feedback: The pathophysiology of SCID involves normal B cells but a lack of antibody production because of inadequate T-cell help. Gene therapy is not yet a realistic treatment option, and the disease has a genetic basis. Antibodies are not incorrect but rather inadequate in number.

9. A 53-year-old woman with a history of chronic alcohol abuse but without visible jaundice comes to the clinic complaining of nausea and weakness. She admits to taking acetaminophen for persistent headaches but denies exceeding the recommended daily dose; she has not taken any other medications. She is suspected of having acetaminophen toxicity. Which of the following diagnostic test findings would implicate a different cause of her symptoms? A) Normal serum acetaminophen level B) Elevated serum HBsAg level C) Evidence of steatosis on liver biopsy tissue sample D) Hypoglycemia

Ans: B Feedback: The presence of HBsAg would suggest that this woman is in the prodromal phase of hepatitis B infection. A normal serum acetaminophen level does not preclude toxicity if the drug is taken over a period of time. Steatosis is fatty infiltration of the liver. Steatosis is often but not exclusively an early histological feature of alcoholic liver disease (therefore, with chronic alcohol abuse, the nurse should expect the patient may have steatosis). Drinking heavily without eating can block your liver from releasing stored glucose into your bloodstream, causing hypoglycemia.

2. As part of her prenatal education, a 29-year-old woman who is pregnant with her first child is receiving teaching from her primary care provider. Which of the following statements by the woman reflects an accurate understanding of HIV transmission? A) ìI know my baby is safe from HIV while in the womb, but the delivery will place him or her at real risk.î B) ìIt's discouraging to know that my breast milk can pass on HIV to my baby.î C) ìI know it's possible, but it's comforting that the chances of my child contracting my HIV are actually very low.î D) ìI'm relieved to learn that a caesarean delivery will protect my baby from being born HIV positive.î

Ans: B Feedback: Transmission from mother to infant is the most common way that children become infected with HIV. HIV may be transmitted from infected women to their offspring in utero, during labor and delivery, or through breast-feeding. Ninety percent of infected children acquired the virus from their mother. The risk of transmission of HIV from mother to infant is approximately 25%, with estimates ranging from 15% to 45%, depending on what country they live in.

1. A care aide at a long-term care facility has informed a resident physician that an 80-year-old woman's eyes appear to be inflamed and that her eyelids are caked with sticky secretions. The woman has been subsequently diagnosed with posterior blepharitis. Which of the following treatments is the physician likely to initiate? A) Surgical repair of the woman's blocked meibomian glands B) Warm compresses to be applied regularly to her eyes in addition to oral antibiotics C) Regularly scheduled cleansing of the woman's eyes with normal saline D) Intravenous steroids coupled with topical antibiotic ointment

Ans: B Feedback: Treatment of posterior blepharitis is determined by associated conjunctival and corneal changes. Initial therapies can include warm compressing of the lids and use of flaxseed or fish oil tablets to provide omega-3 fatty acid benefits to meibomian oil secretions. Long-term, low-dose systemic antibiotic therapy guided by results of bacterial cultures along with short-term topical steroids may also be needed.

5. A 40-year-old man who is morbidly obese and leads a sedentary lifestyle has recently been diagnosed with type 2 diabetes. Which of the following aspects of the man's obesity likely contributed to his new health problem? A) The low metabolic needs of adipose tissue mimic a hypoglycemic state and suppress insulin secretion. B) Free fatty acids contribute to problems such as beta cell dysfunction and insulin resistance. C) Fat tissue initiates glycogenolysis and reliance on glycogen release rather than metabolism of free glucose. D) Triglyceride deposits in the pancreas result in damage to beta cells.

Ans: B Feedback: Type 2 diabetes in obese people is thought to link to the actions of free fatty acids, which include beta cell dysfunction (lipotoxicity), insulin resistance, glucose underutilization, and the accumulation of FFAs and triglycerides reduce hepatic insulin sensitivity

4. An 82-year-old resident of a long-term care facility with a recent history of repeated urinary tract infections and restlessness is suspected of having urinary retention. Which of the following actions by the care team is most appropriate? A) Uroflowmetry to determine the rate of the client's urine flow B) Ultrasound bladder scanning to determine the residual volume of urine after voiding C) Renal ultrasound aimed at identifying acute or chronic kidney disease D) Urinalysis focusing on the presence of or absence of microorganisms, blood, or white cells in the man's urine

Ans: B Feedback: Ultrasound bladder scanning yields a fast and noninvasive indication of whether or not an individual is adequately emptying his or her bladder with each void. Uroflowmetry would be less indicative of whether the man is retaining, and renal ultrasound would address deficits in urine production rather than bladder emptying. Urinalysis would be useful in the diagnosis of infections and/or renal issues more than deficiencies in bladder emptying.

11. Because the associated nephropathy is an important cause of end-stage renal failure in children and adolescents, a toddler who has had an uncomplicated bout of urinary tract infection (UTI) should still be evaluated for A) urethrovesical reflux. B) vesicoureteral reflux. C) neurogenic bladder. D) detrusor muscle instability.

Ans: B Feedback: Urethrovesical reflux occurs when urine from the urethra moves into the bladder. Vesicoureteral reflux occurs when urine moves from the bladder into the ureter. Vesicoureteral reflux is the most commonly associated abnormality in UTIs. Reflux nephropathy is an important cause of end-stage renal disease in children and adolescents; children with a relatively uncomplicated first UTI may turn out to have significant reflux. Therefore, even a single documented UTI in a child requires careful diagnosis. Functional obstructions include neurogenic bladder, infrequent voiding, detrusor (bladder) muscle instability, and constipation.

16. Which of the following processes would most likely be considered an anomaly during the cellular phase of inflammation? A) Platelet aggregation B) Vasoconstriction C) Migration of phagocytic white cells D) Macrophage activity

Ans: B Feedback: While vasoconstriction is a component of the immediate inflammatory response, the later cellular phase of inflammation is accompanied by vasodilation. Platelet aggregation, vasoconstriction, migration of phagocytic white cells, and macrophage activity are all associated with the cellular phase.

19. A frantic mother brings her young child into the emergency department. She states that during the evening bath, she noticed a large mass in her child's abdomen. After diagnostic testing, the pediatrician tells the parents that their child has Wilms tumor, stage IV. After the doctor leaves the room, the parents ask the nurse, ìWhat does this mean?î The nurse will respond, ìYour child ('s) A) ìhas cancer in his stomach.î B) ìhas cancer in the kidney that has spread most likely to his lungs.î C) ìwill need to undergo surgery to remove both kidneys and then go on dialysis.î D) ìtumor can be easily treated with chemotherapy. We will start this soon.î

Ans: B Feedback: Wilms tumor usually is a solitary mass that occurs in any part of the kidney. It usually is sharply demarcated and variably encapsulated. Stage IV means the tumors have undergone hematogenous metastasis, most commonly involving the lung. Treatment involves surgery, chemotherapy, and sometimes radiation therapy. Long-term survival rates are good (90%) for stages I to III.

6. Following an oral glucose tolerance, a 36-year-old mother of four has been diagnosed with gestational diabetes mellitus (GDM), a problem that was not present in any of her previous pregnancies. What should her primary care provider tell her about this new health problem? A) ìThis diabetes is unlikely to persist after you give birth, but the main risk is that your baby will likely be born with diabetes.î B) ìYour baby could become too large or have low blood sugars if we're not vigilant about controlling your sugars.î C) ìWe'll monitor this closely and begin insulin therapy as soon as possible.î D) ìThis is likely a result of your liver releasing too much fat, rather than your pancreas not secreting insulin.î

Ans: B Feedback: Women with GDM are at higher risk for complications of pregnancy, mortality, and fetal abnormalities. Fetal abnormalities include macrosomia, hypoglycemia, hypocalcemia, polycythemia, and hyperbilirubinemia. GDM often persists as type 2 diabetes after delivery, and the baby does not face a significantly higher risk of developing diabetes. Nutrition therapy would precede insulin therapy, and GDM involves a pancreatic etiology.

10. An infant born with congenital hypothyroidism and has not sought care from any health care provider is likely to develop which of the following complications? Select all that apply. A) Deformed joints and bone spurs B) Impaired physical growth C) Mental retardation D) Loss of fine motor control and arthritis E) Down syndrome

Ans: B, C Feedback: Thyroid hormone is essential for normal growth and brain development, almost half of which occurs during the first 6 months of life. If untreated, congenital hypothyroidism causes mental retardation and impairs physical growth. Down syndrome is a congenital birth defect and not caused by hypothyroidism.

1. After years of going to different physicians with vague symptoms, a 55-year-old client with a history of Hodgkin disease has been diagnosed with a secondary immunodeficiency syndrome. The client asks the nurse what this means. The nurse knows from the following list of characteristics that secondary immunodeficiency disorders: Select all that apply. A) may be inherited as a sex-linked trait. B) usually develop later in life. C) may be a result of chemotherapy being used to treat a cancer. D) can result from frequent recurring Staphylococcus aureus infections. E) can occur in a chronic obstructive pulmonary disease patient taking corticosteroids daily.

Ans: B, C, E Feedback: Secondary immunodeficiency disorders develop later in life as a result of other pathophysiologic states such as malnutrition, disseminated cancers, infection of the cells of the immune system, and treatment with immunosuppressive drugs, such as chemotherapeutic agents. Primary disorders may be congenital or inherited as sex-linked, autosomal dominant, or autosomal recessive traits. Humoral (B-cell) immunodeficiencies are primarily associated with recurrent infections like Staphylococcus aureus.

16. Which of the following individuals would most likely be placed on highly active antiretroviral therapy (HAART) if he or she were not yet receiving the treatment? Select all that apply. A) A 35-year-old female sex trade worker who is HIV negative but who has a documented history of sharing needles for heroin use B) A 46-year-old male with long-standing HIV and a CD4+ count of 125 cells/mL C) A 16-year-old female who was diagnosed with HIV 2 days prior and is asymptomatic with normal CD4+ levels D) A 38-year-old woman who has a CD4+ count of 250 cells/mL and is keen to begin HAART E) Prophylactically to a health care worker who incurred a laceration from a scalpel used in surgery but has no abnormal lab results

Ans: B, D Feedback: All symptomatic patients should be treated with antiretroviral therapy. If the individual is asymptomatic, therapy is recommended for CD4+ cell counts less than or equal to 160/mL. For those who have a CD4+ cell count greater than 350 cells/mL, antiretroviral therapy is generally not recommended. For those whose CD4+ cell count is 160 to 350 cells/mL, then antiretroviral therapy should be considered, and a decision individualized to the patient should be made. HAART is not begun prophylactically in the absence of HIV.

14. A 9-year-old boy has been diagnosed with the nephrotic syndrome. Place the following stages in the development of his health problem in ascending order. Use all the options. A) Hypoalbuminemia B) Increased glomerular membrane permeability C) Decreased colloidal osmotic pressure D) Proteins escape from the plasma to the glomerular filtrate E) Accumulation of fluid in the interstitial tissue (edema)

Ans: B, D, A, C, E Feedback: The pathophysiology of the nephrotic syndrome involves damaged glomeruli becoming increasingly permeable to protein, allowing more protein into the glomerular filtrate. Massive proteinuria results, leading to hypoalbuminemia. Generalized edema, which is the hallmark of nephrotic syndrome, results from the loss of colloidal osmotic pressure of the blood with subsequent accumulation of fluid in the interstitial tissues.

6. Stepping out of a mall and into the sunshine has caused a man's pupils to constrict. Place the following anatomical components of the man's pupillary reflex in the ascending chronological order that they responded to the light. Use all the options. A) Oculomotor nuclei B) Retinal ganglionic cells C) Preganglionic neurons D) Pretectal nuclei

Ans: B, D, C, A Feedback: Pretectal areas on each side of the brain are connected, explaining the binocular aspect of the light reflex. The afferent stimuli for pupillary constriction arise in the ganglionic cells of the retina and are transmitted to the pretectal nuclei at the junction of the thalamus and the midbrain and from there to preganglionic neurons in the oculomotor (CN III) nuclei via the pretectooculomotor tract.

6. Which of the following individuals are likely to display identified risk factors for the development of lower urinary tract obstruction? Select all that apply. A) A 32-year-old woman who had a healthy delivery of her third child 4 months ago B) A 68-year-old man who has been diagnosed with benign prostatic hyperplasia (BPH) C) A 55-year-old man with diabetes who is receiving diuretic medications for the treatment of hypertension D) A 30-year-old woman who has been diagnosed with gonorrhea E) A 74-year-old woman who has developed a lower bowel obstruction following several weeks of chronic constipation F) A 20-year-old man who has spina bifida and consequent impaired mobility.

Ans: B, D, E, F Feedback: BPH frequently obstructs the urethra, while sexually transmitted diseases, bowel obstructions, and spina bifida are also associated with physical blockages of the lower urinary tract. Postpartum women and individuals receiving diuretics would be more likely to be at risk for incontinence rather than urinary retention.

4. A nurse has noted the high incidence of urinary tract obstructions of a variety of etiologies. Which of the following individuals are at risk of developing urinary obstructions? Select all that apply. A) A 43-year-old male with an acidñbase imbalance secondary to malnutrition B) A 29-year-old female, pregnant for the first time C) A 69-year-old female with anemia secondary to insufficient erythropoietin production D) A 70-year-old male with benign prostatic hyperplasia (BPH) E) A 58-year-old male with renal calculi F) A 28-year-old male with a neurogenic bladder secondary to spinal cord injury

Ans: B, D, E, F Feedback: Pregnancy, BPH, renal calculi, and neurogenic bladder are all identified contributors to urinary obstructions. Acidñbase imbalances and impaired erythropoietin production are health problems with renal involvement but are less likely to contribute to urinary obstruction.

15. Because they strengthen the pelvic floor muscles, Kegel exercises are most likely to help A) overflow incontinence. B) urge incontinence. C) stress incontinence. D) mixed incontinence.

Ans: C Feedback: : Stress incontinence is commonly caused by weak pelvic floor muscles, which allow the angle between the bladder and the posterior proximal urethra to change so that the bladder and urethra are positioned for voiding when some activity increases intra-abdominal pressure. Overflow incontinence results when the bladder becomes distended and detrusor activity is absent. Urge incontinence is probably related to CNS control of bladder sensation and emptying or to the smooth muscle of the bladder. Mixed incontinence, a combination of stress and urge incontinence, probably has more than one cause.

2. An example of a single hormone that can exert effects in different tissues, erythropoietin, made in the kidney stimulates the bone marrow to produce A) platelets. B) natural killer cells. C) red blood cells. D) mast cells.

Ans: C Feedback: A characteristic of hormones is that a single hormone can exert various effects in different tissues. For example, erythropoietin, a traditional circulating hormone, is made in the kidney and stimulates erythropoiesis in the bone marrow.

20. A 51-year-old woman has been diagnosed with Cushing syndrome after a diagnostic workup that reveals cortisol hypersecretion. The nurse knows which of the following assessment findings would be inconsistent with her diagnosis? A) Increased blood pressure and decreased potassium levels B) A protruding abdomen and a ìbuffalo humpî on the back C) Poor stress management and hyperpigmentation D) A ìmoon faceî and muscle weakness

Ans: C Feedback: A low tolerance for stress and hyperpigmentation is associated with Addison disease and its consequent elevated levels of ACTH. High blood pressure, hypokalemia, buffalo hump, and moon face are all characteristics of the elevated steroid levels that denote Cushing syndrome.

13. A 21-year-old female is suspected of having inadequate function of her hypothalamicñpituitaryñthyroid system. Her care provider is planning to inject thyrotropin-releasing hormone (TRH) and then measure her levels of TSH. Which of the following diagnostic tests is being performed? A) Suppression test B) Radioimmunoassay (RIA) test C) Stimulation test D) Metabolite excretion test

Ans: C Feedback: A stimulation test involves the introduction of an element that stimulates the production of another factor or hormone followed by measurement of that hormone. This is not the case in a suppression test, RIA test, or metabolite excretion test.

3. The clinical nurse educator on a nephrology unit of a large, urban hospital is orientating recent nursing graduates to the unit. Which of the following teaching points about acute tubular necrosis (ATN) should the educator include in the orientation session? A) ìThe cardinal signs of ATN are oliguria and retention of potassium, creatinine, and sulfates.î B) ìUreteral and bladder outlet obstructions are often contributors to ATN.î C) ìTrauma, burns, and major surgery are common precursors to ATN.î D) ìTubular epithelial cells are sensitive to ischemia and toxins, and damage is irreversible.î

Ans: C Feedback: ATN is often preceded by major surgery, burns, or trauma. Many cases of ATN are nonoliguric, and obstructions that are postrenal in nature are not common causes of ATN. Damage to tubular epithelial cells is not necessarily irreversible.

5. Which of the following preoperative teaching points related to corneal transplantation is most justified? A) ìYou should know that there is a significant risk that your body will reject the transplant.î B) ìThe cornea is highly vascular, and therefore you will be at risk for hemorrhage.î C) ìYour new cornea would come from someone who has recently died.î D) ìYou run a risk of developing a major inflammatory response post-op and will need frequent follow-up appointments.î

Ans: C Feedback: Advances in ophthalmologic surgery permit corneal transplantation using a cadaver cornea. The low rejection rate is due to several factors: the cornea is avascular, which limits perfusion by immune elements; major histocompatibility complexes are virtually absent in the cornea; antigen-presenting cells are not present in great numbers; the cornea secretes immunosuppressive factors; and corneal cells secrete substances that protect against apoptosis, thereby minimizing inflammation.

6. When educating the parents of a child who has just had tympanostomy tube insertion, the nurse should provide further teaching if the parents say which of the following statements? A) ìI will call the physician if I see that the tube has come out.î B) ìI will be looking for any drainage coming from the ears.î C) ìI'm so glad that we can take the child swimming next week when we go on vacation.î D) ìI'm so glad we had the child tested for allergies prior to having these tubes placed.î

Ans: C Feedback: After tube insertion, the ears of children with tubes must be kept out of water. All of the other distracters are normal post-op teaching for this procedure. Anytime a device comes out after surgery, the physician should be notified. Routine post-op education includes looking for infection, which in this case could be fever, increase in drainage from the ear, or restlessness. Prior to surgery, most children with recurrent otitis media have allergy testing performed.

8. A male infant is brought into the clinic because of colic-like symptoms. The mother states he acts like something is hurting. After eating, he vomits most of the feeding and then assumes a fetal position. He is also not gaining weight. The nurse practitioner is thinking that he is displaying clinical manifestations of obstruction and may have which of the following medical diagnoses? A) Duodenal ulceration B) Constipation C) Pyloric stenosis D) Erosive esophagitis

Ans: C Feedback: An example of obstruction is hypertrophic pyloric stenosis, which can occur in infants with an abnormally thick muscularis layer in the terminal pylorus. A defect in the lining of the first part of the small intestine (duodenal ulcer) is usually caused by an infection with a bacterium (germ) called H. pylori. When food is ingested and digested but not excreted, it forms a blockage in the colon. Regular bowel movements are needed in order for this not to occur. When bowel movements are irregular, constipation may result. This infant appears to be vomiting his stomach content; therefore, no BM is occurring since no food is being digested in the small intestine. Gastrin provides the major stimulus for gastric acid production. Its action on the lower esophageal sphincter protects the esophageal mucosa when gastric acid levels are elevated. If stomach acids reflux into the esophagus, acid irritation and inflammation cause extensive injuries to the esophagus.

1. A 23-year-old man has received a recent diagnosis of appendicitis following 24 hours of acute abdominal pain. The nurse providing care for the man is explaining that while unpleasant, the inflammation of his appendix is playing a role in his body's fight against the underlying infectious process. Which of the following teaching points should the nurse eliminate from his teaching to the patient? A) ìInflammation can help to remove the body tissue cells that have been damaged by infection.î B) ìInflammation will start your body on the path to growing new, healthy tissue at the site of infection.î C) ìInflammation helps your body to produce the right antibodies to fight the infection.î D) ìInflammation ultimately aids in eliminating the initial cause of the cell injury in your appendix.î

Ans: C Feedback: Antibody production is not a noted component of the inflammatory response. Removing damaged cells, generating new tissue, and eliminating the cause of cell injury are all documented components of the inflammatory response.

18. While undergoing a kidney transplant from a nonfamily member, the patient's transplanted kidney has just had the arterial clamps removed. The OR staff notice that the organ is turning purple with no urine output. When explaining to the family why they had to remove the donor kidney, the nurse will anticipate that the surgeon would likely include which statement? A) Obviously, there has been a mismatch during the human leukocyte antigen (HLA) testing. B) The circulating B and T lymphocytes are just doing their job. C) Hyperacute rejection occurs because antibodies against HLA antigens are deposited in vessels causing necrosis. D) Previous exposure to the HLA antigens is responsible for the high titers of complement fixing antibodies that cause the rejection.

Ans: C Feedback: Antibody-mediated rejection can be hyperacute, which occurs almost immediately after vascular reperfusion to graft tissue occurs. Performed antibodies against HLA antigens are deposited in the tissue endothelium and microvasculature where they activate the classic complement pathway causing tissue necrosis and graft injury.

2. A 60-year-old man has presented to his family physician following an earache that has become progressively more painful in recent days. Following a history and examination with an otoscope, the man has been diagnosed with otitis externa. Which of the physician's following statements to the man is most accurate? A) ìYou'll need to avoid getting any water in your ear until you finish your course of antibiotic pills.î B) ìI'm going to instill some warm water into your ear to flush out debris and bacteria.î C) ìI'll prescribe some ear drops for you, and in the mean time, it's important not to use ear swabs.î D) ìThis likely happened because your ears aren't draining like they should, but antibiotics that you'll put in your ears will resolve this.î

Ans: C Feedback: Antimicrobial ear drops are the standard treatment for otitis externa, and the ears must be protected from trauma during infection. Oral antibiotics are not commonly used, and flushing the ears is not indicated for the condition. A lack of normal ear drainage is not part of the etiology of otitis externa.

10. When explaining what is occurring when their child has an acute bronchial asthma attack, the nurse will emphasize that which mediator is primarily responsible for the bronchial constriction? A) Tree pollen B) Mold dust C) Histamine D) T-lymphocyte proliferation

Ans: C Feedback: Asthma response begins within 5 to 30 minutes of exposure to an allergen. It is mediated by mast cell degranulation and the release of preformed and/or enzymatically activated mediators. These mediators include histamine, serotonin, and acetylcholine. Histamine is the most recognized mediator of type I hypersensitivity reactions and ultimately results in bronchial constriction.

16. A 38-year-old woman takes clomiphene, an infertility drug that works by competing with, and thereby blocking, cellular receptors for estrogen. Which of the following statements is most likely to be true of this client? A) Receptors for all other steroid hormones will also be blocked. B) Up-regulation will increase the number of estrogen receptors on each target cell. C) Estrogen will continue to pass freely through the cellular membranes. D) Laboratory tests will reveal an increase in cyclic adenosine monophosphate (cAMP) levels.

Ans: C Feedback: Because estrogen is a steroid hormone, its receptors in target cells are located inside the cell membrane, and their blockage does not affect the movement of the hormone into and out of the cell. Receptors are specific for each hormone, so no hormones other than estrogen will be blocked. Up-regulation occurs when hormone levels are decreased, and in this case, the estrogen level will increase. Second messengers, such as cAMP, are only activated by peptide hormones and catecholamines.

1. During a procedure to remove impacted cerumen, the nurse should be assessing the patient for which of the following most critical complications? A) Excessive bleeding from the ear B) Pain related to instillation of warm fluids to soften the cerumen C) Symptomatic bradycardia from vagal nerve innervation D) Respiratory distress related to fluid entering bronchiole tubes

Ans: C Feedback: Because the external auditory canal is innervated by the auricular branch of the vagus nerve, coughing or even cardiac deceleration can result from stimulation of the canal by cerumen impactions or removal attempts. Since this just involves irrigation using a bulb syringe and warm tap water, bleeding should not occur. Pain may occur but is not a critical complication. Since the fluid is going into the ear canal, there should be no fluid entering the respiratory system.

15. A middle school student is scheduled to receive booster immunizations, and the father asks the nurse why the booster is necessary. What characteristic of the adaptive immune system listed below would provide the rationale for the nurseís response? A) Some antibodies require a repeat of the primary immune response. B) Some antibodies have a duration measured in months rather than years. C) A secondary response causes a sharp rise in antibody levels. D) Antigen receptors on CD4+ cells require multiple exposures separated by time.

Ans: C Feedback: Booster immunizations take advantage of the increase in antibodies that accompanies a repeat exposure. The primary immune response cannot be repeated, and antibodies survive beyond several months. Antigen receptors on CD4+ cells do not require multiple exposures.

18. A male client complaining of chronic cramping, bloating, and diarrhea has been determined to have a deficiency in brush border enzymes within his small intestine. Which of the following meals or snacks high in carbohydrates and protein will likely exacerbate the client's signs and symptoms? A) Grapefruit and prunes B) Tossed salad with an oil and vinegar dressing C) Roast beef and a baked potato D) Tortilla chips and guacamole

Ans: C Feedback: Brush border enzymes are primarily responsible for the metabolism of carbohydrates and proteins, substances best exemplified by a baked potato and roast beef, respectively. Of the distracters, choice C has both high protein and high carbohydrate content. Distracters A, B, and D are not high in both carbohydrates and proteins.

16. Following a 14-day course of broad-spectrum antibiotics for the treatment of sepsis, a 60-year-old woman has developed watery diarrhea. Her care team attributes this to likely elimination of normal intestinal flora by the antibiotics. What other phenomena is most likely accompanying her low levels of normal flora? A) Decreased mineral and nutrient absorption; decreased carbohydrate metabolism B) Decreased pH of the stomach; increased pH of the lower GI tract C) Decreased fermentation of undigestible dietary residue; decreased vitamin absorption D) Proliferation of vitamin K; lower GI bleeding

Ans: C Feedback: Central among the functions of normal intestinal flora are the fermentation of dietary components that are not digestible and the facilitation of vitamin absorption. Mineral and nutrient absorption as well as carbohydrate metabolism are less likely to be affected, while vitamin K production would likely decrease somewhat. pH is unlikely to be affected.

8. A 4-year-old boy presents with a chronic cough and swollen lymph nodes. His records show that he has been given antibiotics several times in the past year with limited success, most recently for a liver abscess, and that he also has a recurring fungal skin condition. Which of the following is his most likely diagnosis? A) Selective IgA deficiency B) A deficiency in IgG2 subclass antibodies C) Chronic granulomatous disease D) Ataxiañtelangiectasia

Ans: C Feedback: Chronic granulomatous disease, because it affects phagocytic function, increases susceptibility to soft tissue infections, particularly of the skin, lungs, lymph nodes, and liver. Selective IgA deficiency and deficient IgG2 subclass antibodies can predispose people to infection, but those infections respond readily to antibiotic treatment. Ataxiañtelangiectasia can cause skin and liver problems, but its primary manifestations are ataxia and telangiectasia.

6. Utilizing the World Health Organization (WHO) framework of clinical categories for persons with acquired immunodeficiency syndrome (AIDS) over 15 years of age, a visitor to the United States goes to a city clinic complaining of diarrhea, weight loss of 20 lb, and feeling like he is running a temperature. These manifestations have been occurring for the past 5 weeks. The nurse would identify this patient to be in which clinical stage? A) Stage 1 B) Stage 2 C) Stage 3 D) Stage 4

Ans: C Feedback: Clinical stage 3 includes unexplained chronic diarrhea for greater than 1 month, persistent oral candidiasis, oral hairy leukoplakia, TB, neutropenia, anemia, and thrombocytopenia.

15. A patient with pancreatic cancer is admitted for portal hypertension in which he is symptomatic with ascites. Following paracentesis and removal of 7.5 L of ascitic fluid, the nurse should anticipate that the physician will order which of the following medications to assist in maintaining an effective circulating fluid volume? A) Bumetanide (Bumex) B) Furosemide (Lasix) C) Albumin (human) 5% D) Epogen (epoetin alfa)

Ans: C Feedback: Complications of portal hypertension include ascites, splenomegaly, and hepatic encephalopathy. Following paracentesis, to remove ascitic fluid, a volume expander such as albumin is usually administered to maintain the effective circulating volume. Lasix and Bumex are diuretics that decrease circulating fluid volume. Epogen stimulates red blood cell production and ultimately increases O2-carrying capacity.

7. A family physician is providing care for a 61-year-old obese male who has a history of diabetes and hypertension. Blood work has indicated that the man has a GFR of 51 mL/minute with elevated serum creatinine levels. Which of the following statements will the physician most likely provide the client in light of these results? A) ìWe will regularly monitor your kidney function, but most likely your kidneys will be able to compensate on their own and intervention is not required.î B) ìYou likely have chronic kidney disease, and there may be urine in your blood until it is controlled.î C) ìYour chronic kidney disease has likely been caused by your diabetes and high blood pressure.î D) ìYou're in kidney failure, and I'll be starting dialysis treatment immediately.î

Ans: C Feedback: Diabetes and hypertension are conditions that can cause chronic kidney disease (CKD). While the kidneys do have a remarkable ability to compensate for impaired function, this fact does not mean that treatment would not be undertaken. Hematuria is not a common manifestation of CKD, and the client's GFR of 51 mL/minute does not indicate kidney failure or the need for dialysis.

19. Which of the following patients is most likely to have impairments to the wound-healing process? A patient with A) chronic obstructive pulmonary disease. B) a diagnosis of multiple sclerosis and consequent impaired mobility. C) poorly controlled blood sugars with small blood vessel disease. D) congenital heart defects and anemia.

Ans: C Feedback: Diabetes mellitus is strongly associated with impaired wound healing. The other noted pathologies are less causative of deficiencies in the healing process.

20. As part of the diagnostic workup for a client's long-standing vertigo, a clinician wants to gauge the eye movements that occur in the client. Which of the following tests is the clinician most likely to utilize? A) Romberg test B) Rotational tests C) Electronystagmography (ENG) D) Caloric stimulation

Ans: C Feedback: ENG is an examination that records eye movements in response to vestibular, visual, cervical (vertigo triggered by somatosensory input from head and neck movements), rotational, and positional stimulation. With ENG, the velocity, frequency, and amplitude of spontaneous or induced nystagmus and the changes in these measurements brought by a loss of fixation, with the eyes open or closed, can be quantified. The Romberg test, rotational tests, and caloric stimulation do not allow for these data.

17. A patient has sought medical attention because of a loss of different half-fields in the two eyes. Knowing the potential causes of this complaint, the nurse anticipates that the physician will order tests looking for A) metal fragments in the eyes. B) hemorrhages in the capillaries of both eyes. C) an enlarging pituitary tumor. D) subarachnoid hemorrhage.

Ans: C Feedback: Enlarging pituitary tumors can produce longitudinal damage through the optic chiasm with loss of medial fibers of the optic nerve representing both nasal retinas and both temporal visual half-fields. The loss of different half-fields in the two eyes is called a heteronymous loss. Metal fragments can get in the eye from welding. Hemorrhages in both eyes have numerous causes like uncontrolled hypertension and diabetes. Subarachnoid hemorrhages are usually trauma related.

5. A nurse practitioner is providing care for a male client with a long-standing hiatal hernia. Which of the following statements most accurately captures an aspect of the pathophysiology of hiatal hernias? A) Paraesophageal hiatal hernias are common and are normally not treated if the client is asymptomatic. B) The root causes of hiatal hernias are normally treatable with medication. C) If esophageal acid clearance is impaired, esophagitis can result. D) An incompetent pyloric sphincter and high-fat diet are commonly implicated in the development of hiatal hernias.

Ans: C Feedback: Erosive esophagitis can be a complication of hiatal hernias if esophageal acid clearance is significantly impaired. Paraesophageal hiatal hernias are more serious than the sliding variety and require treatment. The root cause of hiatal hernias, herniation of the stomach through the diaphragm, is not normally amenable to treatment with medication. The pyloric sphincter is not associated with hiatal hernias.

7. Parents of a 20-month-old infant report that he refuses food or eats poorly and that he grimaces when he swallows. He also is irritable and cries a lot. The mother is worried that he ate something inappropriate this morning, because he vomited something that looked like coffee grounds. Which of the following health problems would the care team first suspect? A) Rotavirus infection B) Appendicitis C) Esophagitis from gastrointestinal reflux D) Hirschsprung disease

Ans: C Feedback: Esophagitis secondary to reflux can cause feeding problems, early satiety, and hematemesis. Infants may demonstrate signs of pain when swallowing and may be irritable and cry frequently. Rotavirus causes diarrhea and vomiting, but not the other symptoms. Appendicitis is inflammation of the appendix. Appendicitis usually starts with the main symptom of pain around the navel that moves to the lower right abdomen. Hirschsprung disease is a blockage of the large intestine due to improper muscle movement in the bowel. It is a congenital condition, which means it is present from birth. In Hirschsprung disease, the nerves are missing from a part of the bowel. One primary s/s is a failure to pass meconium shortly after birth.

12. Following a history of gastric pain and an endoscopy, a client has been diagnosed with a duodenal peptic ulcer. Which of the following teaching points should his caregiver provide? A) ìWhile your diet most certainly contributed to this problem, the good news is that changing your diet can help solve it.î B) ìUlcers like yours do not penetrate all layers of the stomach or duodenum, so you don't have to worry about losing too much blood.î C) ìYour family history, your smoking history, and NSAID use may all have contributed to this problem.î D) ìWhile there aren't really any effective medications for these ulcers, changes in lifestyle can keep them well controlled.î

Ans: C Feedback: Family history, NSAID use, and smoking have all been identified as contributing factors in the development of peptic ulcers. Diet therapy has not been shown to be effective, and duodenal peptic ulcers are more common than the gastric variant. Perforation occurs when an ulcer erodes through all layers of the stomach or duodenum wall. When perforation occurs in older adults, their mortality is significantly increased. Effective medication regimens are available with antacids, H2-receptor antagonists or proton pump inhibitors being the most common medications used.

9. A 63-year-old woman has begun a diet that purports to minimize the quantity and effects of free radicals in her body. What physiological processes could best underlie her care provider's teaching about her new diet? A) Free radicals act as direct mediators in the inflammatory process. B) Free radicals inhibit the inflammatory response, limiting preadaptive response to infection. C) Free radicals increase cytokine expression and adhesion molecule levels, resulting in increased inflammation. D) Free radicals contribute to atherosclerosis and decreased immune response.

Ans: C Feedback: Free radicals are thought to bring about an inappropriate inflammatory response by increasing cytokines and numbers of adhesion molecule. They are not direct mediators of inflammation and are not associated with decreased immune response but rather inappropriate inflammation. Free radicals are not associated with inhibition of the inflammatory response.

18. Following several days of intermittent upper right quadrant pain, a 29-year-old obese, Native American woman has been diagnosed with cholelithiasis. The nurse at the clinic has taught the client about the pathophysiology and contributing factors to her health problem, as well as some of the likely treatment options. Which of the following statements by the client demonstrates a sound understanding of her diagnosis? A) ìAll in all, I guess this is a result of the fact that I've been eating a diet too high in cholesterol for too long.î B) ìSeveral factors like my genetics and gender may have contributed to this, but I'm glad that medications can cure it.î C) ìThis explains why my skin was yellow-tinged lately and why I had those pains that spread to my upper back and right shoulder.î D) ìI suppose the fever and vomiting I had this week was probably a sign of my gallstones too.î

Ans: C Feedback: Gallstones can be caused by abnormalities in the composition of bile (increased cholesterol) and stasis of bile. The formation of cholesterol stones is associated with obesity and occurs more frequently in women. These factors cause the liver to excrete more cholesterol into the bile. Estrogen reduces the synthesis of bile acid in women. Cholesterol stones are extremely common in Native Americans. Jaundice and pain that radiates to the upper back and right shoulder are noted signs and symptoms of cholelithiasis. While cholesterol is a key element in the formation of gallstones, the particular amount ingested in the diet is not central to the development of the problem. Surgery, not medication, is the normal treatment modality, and fever and nausea are more closely associated with cholecystitis rather than cholelithiasis.

1. A 55-year-old man has been diagnosed with a gastroesophageal reflux disease (GERD), in which the function of his lower esophageal sphincter is compromised. Which of the following consequences of this condition is most likely to occur? A) Decreased absorption of ingested foods and fluids B) Impaired control of the gastric emptying rate C) Protrusion of the stomach or regurgitation of stomach contents into the esophagus D) Inappropriate release of gastric enzymes

Ans: C Feedback: Given that the role of the lower esophageal sphincter is to control the exchange of foods and fluids, a deficit is likely to allow the stomach contents, or the stomach itself, to protrude into the esophagus. Absorption is unlikely to be directly affected, and the sphincter is not responsible for controlling gastric emptying or enzyme secretion.

18. A 34-year-old man has been taking up to 2400 mg of ibuprofen per day following a motor vehicle accident several months ago and consequent chronic pain. He has recently been diagnosed with chronic analgesic nephritis as a result of his high analgesic intake. The man is surprised at the diagnosis stating, ìI thought that taking too many drugs hurts your liver if anything, not your kidneys.î What is the most appropriate response to the man's statement? A) ìYour liver does perform most of the detoxification in your body, but your kidneys can perform this role if the liver is unable to.î B) ìHigh drug intake can cause your kidneys to be very vulnerable to infections, which is likely what happened in your case.î C) ìYour kidneys are vulnerable to damage because of how much blood flows through them and the fact that they break down many drugs.î D) ìIt is very rare for someone as young as yourself to have kidney damage like this; usually only older people are vulnerable to kidney damage from drugs.î

Ans: C Feedback: High flow and pressure combined with the metabolic transformative of drugs make the kidneys vulnerable to drug toxicity. They do not play a backup role to the liver in the metabolism of drugs, and while structural and functional damage may occur with drug overuse, infection is a less likely consequence. The elderly are particularly vulnerable to drug-related nephritis and nephropathies, but it would be incorrect to conclude that younger people are thus immune.

15. A 25-year-old Asian American man arrives in the emergency room in a panic. Except for a bout with bronchitis a week earlier, he has been healthy his entire life; today he has blood in his urine. What disease has likely caused of his hematuria and how should it be treated? A) Goodpasture syndrome and will be treated with plasmapheresis and immunosuppressive therapy B) Membranous glomerulonephritis and should be treated with corticosteroids C) Immunoglobulin A nephropathy and may be advised to use omega-3 fatty acids to delay progression of disease D) Kimmelstiel-Wilson syndrome and should be treated with medication to control high blood pressure

Ans: C Feedback: Immunoglobulin A (IgA) nephropathy (Buerger disease) is a primary glomerulonephritis characterized by the presence of glomerular IgA immune complex deposits. It can occur at any age, but most commonly occurs with clinical onset in the second and third decades of life. It is more common in males than in females and is the most common cause of glomerular nephritis in Asians. There is no satisfactory treatment for IgA nephropathy. Goodpasture syndrome is a form of glomerulonephritis; treatment includes plasmapheresis to remove circulating anti-GBM antibodies and immunosuppressive therapy to inhibit antibody production. Membranous glomerulonephritis is the most common cause of primary nephrosis in adults, most commonly those in their sixth or seventh decade. It is treated with corticosteroids. In nodular glomerulosclerosis, also known as Kimmelstiel-Wilson syndrome, there is nodular deposition of hyaline in the mesangial portion of the glomerulus. As the sclerotic process progresses in the diffuse and nodular forms of glomerulosclerosis in many cases, early changes in glomerular function can be reversed by careful control of blood glucose levels. Control of high blood pressure and smoking cessation are recommended as primary and secondary prevention strategies in persons with diabetes.

14. A 30-year-old male who manages his type 1 diabetes with glyburide presents at the emergency room complaining of headache, confusion, and tachycardia. He has come from a party at which he drank two beers to celebrate running his first half-marathon. Which of the following is likely to be the cause of his complaints? A) Diabetic ketoacidosis B) Hyperosmolar hyperglycemic state C) Hypoglycemia D) Somogyi effect

Ans: C Feedback: In hypoglycemia, headache, difficulty in problem solving, disturbed or altered behavior, coma, and seizures may occur. At the onset, activation of the parasympathetic nervous system often causes hunger, and the initial parasympathetic response is followed by activation of the sympathetic nervous system; this causes anxiety, tachycardia, sweating, and constriction of the skin vessels (i.e., the skin is cool and clammy). In diabetic ketoacidosis, the client typically has a history of 1 or 2 days of polyuria, polydipsia, nausea, vomiting, and marked fatigue. Abdominal pain and tenderness may be experienced without abdominal disease, and the breath has a characteristic fruity smell. The most prominent manifestations of hyperosmolar hyperglycemic state are dehydration and neurologic signs, including grand mal seizures, hemiparesis, Babinski reflexes, aphasia, muscle fasciculations, hyperthermia, hemianopia, nystagmus, and visual hallucinations; the client will also experience excessive thirst. The Somogyi effect describes a cycle of insulin-induced posthypoglycemic hyperglycemic episodes.

6. The mother of 6-year-old male and female fraternal twins has brought her son to see a pediatrician because he is nearly 4 inches shorter than his sister. Which of the following phenomena would the physician most likely suspect as contributing factor to the boy's short stature? A) Genetic short stature B) Lack of IGF receptors in epiphyseal long bones C) A shortage of hypothalamic GHRH production D) Excess insulin production resulting in chronically low blood glucose levels

Ans: C Feedback: Inadequate levels of hypothalamic GHRH will result in adequate production but inadequate release of GH by the pituitary. Genetic short stature is less likely given the disparity between his height and his twin's, and a shortage of IGF receptors is not a noted pathology. While poorly controlled diabetes can contribute to short stature, excess insulin production is not a likely factor.

1. When explaining the role of liver Kupffer cells to a group of nursing students, which of the following statements about the function of these cells is most accurate? A) The primary function of Kupffer cells is to secrete bile. B) These cells are the functional unit of the liver and are responsible for all liver secretions. C) The cells are capable of removing and phagocytizing old and defective blood cells. D) The role of the Kupffer cells is to provide at least 50% of cardiac output each minute to each lobular of the liver.

Ans: C Feedback: Kupffer cells are reticuloendothelial cells that are capable of removing and phagocytizing old and defective blood cells, bacteria, and other foreign material from the portal blood as it flows through the sinusoid. This phagocytic action removes enteric bacilli and other harmful substances that filter into the blood from the intestine. Small tubular channels in the liver secrete bile. The functional unit of the liver is lobule. Approximately 25% of blood per minute enters the liver through the hepatic artery.

3. A 61-year-old woman who has had an upper respiratory infection for several weeks has presented to her family physician with complaints of a recent onset of urinary retention. She reveals to her physician that she has been taking nonprescription cold medications over and above the suggested dose for the past 2 weeks. Which of the following phenomena will her physician most likely suspect is contributing to her urinary retention? A) Cholinergic actions of the cold medicine are triggering internal and external sphincter contraction. B) Antihistamine effects inhibit communication between the pons and the thoracolumbar cord. C) The anticholinergic effects of the medication are impairing normal bladder function. D) Over-the-counter medications such as cold medicine stimulate the parasympathetic nervous system and inhibit bladder emptying.

Ans: C Feedback: Many over-the-counter cold medications have an anticholinergic effect that interferes with normal bladder emptying. These effects on micturition are not a result of cholinergic actions or miscommunication between the pontine micturition center and the spinal cord. Stimulation of the parasympathetic nervous system would tend to increase rather than decrease bladder emptying.

10. A 24-year-old college student has presented to the campus medical clinic with complaints of frequent, burning urination and has, subsequent to urinalysis, been diagnosed with an acute lower urinary tract infection (UTI) caused by E. coli. What teaching will the clinician most likely provide to the student? A) ìThis should likely resolve itself if you drink a lot of water and especially cranberry or blueberry juice.î B) ìUnfortunately, the bacteria causing your infection is no longer responsive to antibiotics, but there are alternative treatments that we can use.î C) ìMany of these bacteria are now resistant to some antibiotics, but I will take that into account when I choose which antibiotic to prescribe.î D) ìThis likely shows that you have some sort of obstruction in your urinary system, so when that is treated your UTI will likely resolve as well.î

Ans: C Feedback: Microbial resistance to TMP-SMX antibiotics is now common in the United States; however, other specific antibiotic options do exist. Cranberry and blueberry juice are more appropriate as preventative rather than curative measures (reduces bacterial adherence to the epithelial lining of the urinary tract), and the majority of uncomplicated UTIs in young women are not the result of urinary obstructions.

7. A speech therapist is performing a swallowing assessment on a 72-year-old man who has suffered a stroke 3 weeks ago. The man has been NPO (nothing by mouth) since his stroke, and the health care team is considering the introduction of oral food. The speech therapist is cueing the client to swallow to preclude either aspiration of food or pocketing of food in the sides of his mouth. The client most likely to have conscious control over which of the following processes listed below involved in swallowing? A) Initiation of primary peristalsis B) Moving the epiglottis back to cover the larynx C) Moving a bolus to the posterior wall of the pharynx D) Moving the bolus backward in the esophagus

Ans: C Feedback: Moving a bolus to the posterior wall of the pharynx is a component of the oral, or voluntary, phase of swallowing. Initiation of primary peristalsis, moving the epiglottis back to cover the larynx, and moving the bolus backward in the esophagus are all involuntary components of the pharyngeal and esophageal phases of swallowing.

5. A 35-year-old female ultramarathon runner is admitted to the hospital following a day-long, 50-mile race because her urinary volume is drastically decreased and her urine is dark red. Tests indicate that she is in the initiating phase of acute tubular necrosis. Why is her urine red? A) Hematuria B) Hemoglobinuria C) Myoglobinuria D) Kidney bleeding

Ans: C Feedback: Myoglobinuria, which can cause acute tubular necrosis via intratubular obstruction, involves the leaching of myoglobin from skeletal muscle into the urine, bypassing the usual filtration by the glomerulus. Excess exercise and muscle trauma can contribute to this. While both hemoglobinuria and myoglobinuria discolor the urine, hemoglobinuria results from hemolysis following a reaction to a blood transfusion, whereas myoglobinuria involves muscle damage.

7. A client who has a diagnosis of an autoimmune disease asks his nurse why it is that his immune system does not attack all of the cells that make up his body. Which of the following aspects of pathogen recognition in the innate immune system listed below would underlie the nurse's response? A) Normal host cells excrete inhibitory proteins that are detected by natural killer cells. B) Intraepithelial lymphocytes and natural killer cells possess specific, highly diverse receptors. C) Pattern recognition receptors (PRRs) ensure that cells are correctly identified. D) Leukocytes possess pathogen-associated molecular patterns (PAMPs)

Ans: C Feedback: PRRs recognize the structure of invaders and thus prevent activation by healthy somatic cells, though neither intraepithelial lymphocytes nor natural killer cells possess the high level of specification and diversity of receptors associated with the adaptive immune system. Host cells do not excrete inhibitory proteins, and PAMPs exist on pathogens, not on leukocytes.

15. The nurse assessing a renal failure patient for encephalopathy caused by high uremic levels may observe which of the following clinical manifestations? A) Severe chest pain with pericardial friction rub on auscultation B) Stiff immobile joints and contractures C) Loss of recent memory and inattention D) Pruritus with yellow hue to skin tone

Ans: C Feedback: Reductions in alertness and awareness are the earliest and most significant indications of uremic encephalopathy. These often are followed by an inability to fix attention, loss of recent memory, and perceptual errors in identifying people and objects. Answer choice A refers to pericarditis; answer choice B refers to arthritis; answer choice D refers to liver involvement.

12. A science teacher is talking to a group of fifth graders about the role of ìspit.î During the course of the discussion, the teacher asks the students which of the following are functions of saliva? A) Good protection device if someone is being attacked B) Will have more saliva production when anxious, such as right before a test C) Has antibacterial action to help keep the mouth clean D) Secretes acid to chemically break down fatty foods like French fries E) Secretes the enzyme pepsin

Ans: C Feedback: Saliva has three functions. The first is to protect and lubricate. The second is to provide antimicrobial protection. The third is to initiate digestion of starches by secreting enzymes. If anxious, SNS activity causes dry mouth. Saliva begins breaking down starches (carbohydrates), not fatty foods. The chief cells secrete pepsinogen, which is converted into pepsin, the enzyme that breaks down protein. This occurs in the stomach.

16. A new nursing student is taking a tuberculin (TB) skin test as part of her preparation for beginning clinical placement in the hospital. The student is unclear of the rationale or physiology involved in this test. Which of the following is the correct explanation? A) The cell-mediated hypersensitivity associated with Mycobacterium tuberculosis remains detectable for several years. B) Formation of contact dermatitis lesions confirms prior TB contact. C) Previous TB exposure forms sensitized Th1 cells that are long-lived. D) This type of delayed-type hypersensitivity (DTH) is a response to latent Mycobacterium tuberculosis bacteria.

Ans: C Feedback: Sensitized Th1 cells form DTH response to introduced antigens. This is not an example of cell-mediated hypersensitivity or contact dermatitis, and it is not a response to latent Mycobacterium tuberculosis.

19. A 23-year-old HIV-positive woman in the United States with routinely low viral loads and robust CD4+ cell counts is planning to get pregnant. Which precaution would her care giver eliminate from her care? A) Offer her HAART that includes zidovudine B) Counsel her not to breast-feed C) Give her single-dose perinatal nevirapine D) Give the infant trimethoprimñsulfamethoxazole, starting at 4 to 6 weeks of age

Ans: C Feedback: Single-dose nevirapine is an appropriate alternative when zidovudine is not available. However, HAART-containing zidovudine is readily available in the United States. Avoiding breast-feeding will reduce the client's chances of transmitting HIV to her infant. Because the risk of transmission is not zero, prophylaxis with trimethoprimñsulfamethoxazole will protect her infant from PCP until its serostatus is known.

8. Since steroid hormones are bound to protein carriers for transport, this means A) they are water soluble and circulate freely in the blood. B) they are degraded by enzymes in the blood. C) they are inactive in the bound state. D) they will be converted into a useable form by enzymes in the blood.

Ans: C Feedback: Steroid hormones are bound to protein carriers for transport and are inactive in the bound state. Their activity depends on the availability of transport carriers.

5. Which of the following hormones are derivatives of cholesterol? A) Epinephrine and norepinephrine B) Insulin and glucagon C) Aldosterone and testosterone D) Eicosanoids and retinoids

Ans: C Feedback: Steroids such as aldosterone and testosterone are a classification of hormones that are derived from cholesterol. Epinephrine and norepinephrine are both amino acids, while insulin and glucagon are classified among peptides, polypeptides, proteins, and glycoproteins. Eicosanoids and retinoids consist of fatty acid compounds.

6. A 51-year-old woman has been experiencing signs and symptoms of perimenopause and has sought help from her family physician. A deficiency in estrogen levels has been determined to be a contributing factor. Which of the following phenomena could potentially underlie the woman's health problem? A) Sufficient synthesis of estrogen but inadequate vesicle-mediated release B) Inadequate synthesis in the rough endoplasmic reticulum of her ovarian cells C) Insufficient estrogen production within the smooth endoplasmic reticulum of the relevant cells D) A lack of prohormone precursors needed for estrogen synthesis and release

Ans: C Feedback: Steroids such as estrogen are produced in the smooth endoplasmic reticulum. Synthesis and release are not separate processes as in the case of peptide hormones, and prohormones are associated with peptide, polypeptide, and protein hormones.

11. The nurse knows the cells primarily programmed to remove the invading organisms and remember the antigen to respond rapidly during the next exposure are A) CD4 and CD8 cells. B) natural killer (NK) cells and macrophages. C) T and B lymphocytes. D) white blood cells and platelets.

Ans: C Feedback: T and B lymphocytes are unique in that they are the only cells in the body capable of recognizing specific antigens present on the surfaces of microbial agents and other pathogens.

1. When explaining about the passage of urine to a group of nursing students, the clinic nurse asks them which muscle is primarily responsible for micturition? Their correct reply is the A) urinary vesicle. B) trigone. C) detrusor. D) external sphincter.

Ans: C Feedback: The detrusor muscle is the muscle of micturition. When it contracts, urine is expelled from the bladder. External sphincter is a circular muscle that surrounds the urethra distal to the base of the bladder and can stop micturition when it is occurring. Trigone is a smooth triangular area that is bounded by the ureters and the urethra. Urinary vesicle is another name for the bladder.

20. A 71-year-old male has been recently diagnosed with a stage III tumor of colorectal cancer and is attempting to increase his knowledge base of his diagnosis. Which of the following statements about colorectal cancer demonstrates a sound understanding of the disease? A) ìIf accurate screening test for this type of cancer existed, it could likely have been caught earlier.î B) ìThe NSAIDs and aspirin that I've been taking for many years probably contributing to me getting cancer.î C) ìWhile diet is thought to play a role in the development of colorectal cancer, the ultimate causes are largely unknown.î D) ìA large majority of patients who have my type of colon cancer survive to live many more years.î

Ans: C Feedback: The etiology of cancer of the colon and rectum remains largely unidentified, though dietary factors are thought to exist. The prognosis, especially with stage III tumors, is poor. Simple and accurate screening tests do exist for colorectal cancer, while drugs are not implicated in the etiology.

4. The instructor asks a group of nursing students to explain the function of the omentum. The students will respond based on which pathophysiologic principle? A) It holds organs in place. B) It attaches the jejunum and ileum to the abdominal wall. C) It has lots of mobility and moves around in the peritoneal cavity with peristaltic movements. D) It is mainly there to prevent any noxious substance from inner into the gut.

Ans: C Feedback: The greater omentum has considerable mobility and moves around in the peritoneal cavity with the peristaltic movements of the intestines. It also cushions the abdominal organs against injury and provides insulation against the loss of body heat. The mesentery holds the organs in place and attaches the jejunum and ileum to the abdominal wall. The mucosal layer acts as a barrier to prevent the entry of noxious substances and pathogenic organisms.

9. A patient who has suffered a spinal cord injury at C4 is experiencing a sudden change in condition. His BP is 186/101; heart rate is 45; and he is profusely sweating and complaining of ìnot feeling right.î The nurse should A) call a ìCode Blue.î B) page physician stat. and ask for an antihypertensive medication. C) palpate his bladder for overdistention. D) place his bed flat and elevate the foot of the bed.

Ans: C Feedback: The most common causes of spastic bladder dysfunction are spinal cord lesions such as spinal cord injury, herniated intervertebral disk, vascular lesions, tumors, and myelitis. Because the injury interrupts CNS control of sympathetic reflexes in the spinal cord, severe hypertension, bradycardia, and sweating can be triggered by insertion of a catheter or mild overdistention of the bladder. The patient does not qualify for a Code Blue since he still has a pulse and is breathing. Antihypertensive medication is not necessary if the bladder is emptied. Placing him flat with the foot of the bed elevated will not help this situation.

15. A college junior calls his mother, a nurse, complaining of ìnot being able to see.î When questioned further, he describes, ìA gray curtain just went down my right visual field. I don't know what to do.î The nurse should recognize this symptom as which of the following conditions and have her teenager go to the emergency department immediately. A) Glaucoma B) Strabismus C) Retinal detachment D) Macular degeneration

Ans: C Feedback: The primary symptom of retinal detachment consists of painless changes in vision. Commonly, flashing lights or sparks, followed by small floaters or spots in the field of vision, occur as the vitreous pulls away from the posterior pole of the eye. As detachment progresses, the person perceives a shadow or dark curtain progressing across the visual field.

3. A potential donor is angry at the personal nature of the questions about HIV risk factors that he is required to answer at a blood collection center and states that simple blood testing should suffice. How can the nurse at the center best respond? A) ìThere are some very uncommon subtypes of the HIV virus that are not detectable by current testing methods.î B) ìThere's a chance that persons who are asymptomatic, but HIV positive can have their antibodies missed by serum testing.î C) ìThere's a period shortly after someone is infected with HIV when blood tests might still be negative.î D) ìEven though blood tests are completely accurate, the high stakes of blood donation and transfusion mean that double measures are appropriate.î

Ans: C Feedback: The time after infection and before seroconversion is known as the window period, during which HIV antibody screening may be negative. Potential donors are thus screened to identify potential risk factors. Undetectable subtypes of HIV do not exist, and individuals who are asymptomatic are still able to be accurately tested.

1. The nurse walks into a room and finds the patient forcefully expelling stomach contents into a wash basin. When documenting this occurrence, the nurse will use the term A) nauseous. B) retching. C) vomiting. D) expatriate.

Ans: C Feedback: Vomiting or emesis is the sudden and forceful oral expulsion of the contents of the stomach. It is usually preceded by nausea. Nausea is a subjective and unpleasant sensation. Retching consists of rhythmic spasmodic movements of the diaphragm, chest wall, and abdominal muscles. It usually precedes or alternates with periods of vomiting. Expatriate means to banish or withdraw.

14. A nurse educator is performing client education with a 51-year-old man who has been recently diagnosed with chronic kidney disease. Which of the following statements by the client would the nurse most likely want to correct or clarify? A) ìI'll be prone to anemia, since I'm not producing as much of the hormone that causes my bones to produce red blood cells.î B) ìMy heart rate might go up because of my kidney disease, and my blood might be a lot thinner than it should be.î C) ìMy kidney problems increase my chance of developing high blood pressure or diabetes.î D) ìI'll have a risk of either bleeding too easily or possibly clotting too quickly, though dialysis can help minimize these effects.î

Ans: C Feedback: While high blood pressure can be causative of, or consequent to, renal failure, diabetes is not normally a result of existing CKD. Persons with renal failure are indeed prone to anemia, increased heart rate, decreased blood viscosity, and coagulopathies. The risk of bleeding and thrombotic disorders can be partially mitigated by dialysis.

11. A woman with poorly controlled type 1 diabetes has been admitted to a hospital unit for the treatment of ketoacidosis. Place the following events in the pathophysiology of ketoacidosis in the correct chronological order. Use all the options. A) Decrease in pH B) Breakdown of triglycerides C) Low serum insulin levels D) Ketone production by the liver E) Production of fatty acids and glycerol

Ans: C, B, E, D, A Feedback: During ketoacidotic episodes, the lack of insulin leads to mobilization of fatty acids from adipose tissue because of the unsuppressed adipose cell lipase activity that breaks down triglycerides into fatty acids and glycerol. The increase in fatty acid levels leads to ketone production by the liver and ultimately metabolic acidosis.

20. While volunteering in an HIV clinic in a big city, the nurse notices a new mom and her baby (a 6-month-old male) in the waiting room. Upon assessing the infant for possible HIV infection, the nurse will be assessing for which of the following clinical manifestations of HIV infection? Select all that apply. A) Weighing him to determine if he is gaining 1.5 to 2 lb/month B) Observing to see if he can roll over from back to stomach C) Lack of coordination to play with toys/stuffed animals D) History of repeated episodes of bacterial pneumonia and ear infections E) Listlessness and poor eye contact

Ans: C, D, E Feedback: Children differ as to their clinical presentation of HIV infection when compared to adults. Failure to thrive (gain weight/height), CNS abnormalities (listlessness), and developmental delays are the most prominent primary manifestation of HIV infection in children. Answers A and B are normal growth and developmental tasks of a 6-month-old.

2. A 30-year-old man with a diagnosis of type 1 diabetes is aware of the multiple effects that insulin has on his metabolism. Which of the following physiological processes are actions of insulin? Select all that apply. A) Increasing the metabolic needs of body cells B) Promoting the breakdown of stored triglycerides C) Facilitating triglyceride synthesis from glucose in fat cells D) Inhibiting protein breakdown E) Promoting glucose uptake by target cells

Ans: C, D, E Feedback: The actions of insulin are threefold: (1) it promotes glucose uptake by target cells and provides for glucose storage as glycogen; (2) it prevents fat and glycogen breakdown; and (3) it inhibits gluconeogenesis and increases protein synthesis. Insulin acts to promote fat storage by increasing the transport of glucose into fat cells. It also facilitates triglyceride synthesis from glucose in fat cells and inhibits the intracellular breakdown of stored triglycerides. Insulin also inhibits protein breakdown. It does not directly influence the metabolic needs of body cells.

5. A male infant who is 48 hours postpartum is undergoing phototherapy for the treatment of jaundice and accompanying high levels of bilirubin. Place the following components of the production of bilirubin in the correct chronological order. Use all the options. A) Conjugated bilirubin B) Urobilinogen C) Red blood cells D) Biliverdin E) Free bilirubin

Ans: C, D, E, A, B Feedback: Hemoglobin from the red blood cell is broken down to form biliverdin, which is rapidly converted to free bilirubin. Inside the hepatocytes, free bilirubin is converted to conjugated bilirubin. Conjugated bilirubin is secreted as a constituent of bile, and in this form, it passes through the bile ducts into the small intestine. In the intestine, approximately one half of the bilirubin is converted into a highly soluble substance called urobilinogen by the intestinal flora.

19. A patient is admitted to the hospital in adrenal crisis 1 month after a diagnosis of Addison disease. The nurse knows which of the following clinical manifestations would support this diagnosis? A) Hyperactive deep tendon reflexes and slow, shallow breathing B) Cerebral spinal fluid leakage and impaired swallowing C) Irregular heart rate and decreased temperature D) Change in the level of consciousness and profound hypotension

Ans: D Feedback: Acute adrenal crisis is a life-threatening situation. Exposure to even a minor illness or stress can cause a client with Addison disease to develop nausea, vomiting, muscular weakness, hypotension, dehydration, and vascular collapse (which causes a change in LOC). Hemorrhage (low BP) can be caused by septicemia, adrenal trauma, anticoagulant therapy, adrenal vein thrombosis, or adrenal metastases. A hyperactive reflex may indicate disease of the pyramidal tract above the level of the reflex arc being tested. Generalized hyperactivity of DTRs may be caused by hyperthyroidism. Any tear or hole in the membrane that surrounds the brain and spinal cord (dura) can allow the fluid that surrounds those organs to leak. This fluid is called the cerebrospinal fluid (CSF). When it leaks out, the pressure around the brain and spinal cord drops. Causes of leakage through the dura include certain head, brain, or spinal surgeries; head injury; placement of tubes for epidural anesthesia or pain medications; or lumbar puncture. Irregular heart rates (arrhythmias) may be caused by many different factors, including coronary artery disease; electrolyte imbalances in your blood (such as sodium or potassium); changes in your heart muscle; or injury from a heart attack.

17. A patient asks the nurse what it means when the doctor said that he had adenocarcinoma of the bladder. Reviewing the pathophysiologic principles behind this type of cancer, the nurse knows A) it is a low-grade tumor that is readily cured with bladder surgery. B) after resection of the cancer, the prognosis is excellent with this type of cancer cell. C) that these types of cancer cells are very invasive to the tissue; therefore, the entire bladder must be removed. D) this is a rare but highly metastatic tumor that has a very poor prognosis.

Ans: D Feedback: Adenocarcinoma is rare and highly metastatic. Answer choices A and B relate to urothelial carcinoma; answer choice C relates to squamous cell carcinoma.

19. While on tour, a 32-year-old male musician has presented to the emergency department of a hospital after a concert complaining of severe and sudden abdominal pain. He admits to a history of copious alcohol use in recent years, and his vital signs include temperature 46.8∞C (101.8∞F), blood pressure 89/48 mm Hg, and heart rate 116 beats/minute. Blood work indicates that his serum levels of C-reactive protein, amylase, and lipase are all elevated. Which of the following diagnoses would the care team suspect first? A) Hepatitis C B) Cholecystitis C) Liver cirrhosis D) Acute pancreatitis

Ans: D Feedback: Alcohol use, fever, hypotension, and tachycardia are often associated with pancreatitis, as are elevated serum amylase and lipase levels. These enzymes would unlikely rise in cases of hepatitis, cholecystitis, or cirrhosis. The precise mechanisms whereby alcohol exerts its action are largely unknown. The capacity for oxidative and nonoxidative metabolism of ethanol by the pancreas and the harmful by-products that result have been related to the disease process. Hepatitis C has an incubation period. Most adults who acquire the infection usually are asymptomatic. Jaundice is uncommon. Direct measurement of HCV in the serum remains the most accurate test for infection. Cirrhosis represents the end stage of chronic liver disease. The end result is liver failure that affects many organs. The patients usually have anemia, thrombocytopenia, endocrine disorders, skin lesions, azotemia and renal failure, and hepatic encephalopathy.

11. Following the identification of low levels of T3 and T4 coupled with the presence of a goiter, a 28-year-old female has been diagnosed with Hashimoto thyroiditis. In light of this diagnosis, which of the following assessment results would constitute an unexpected finding? A) The presence of myxedema in the woman's face and extremities B) Recent weight gain despite a loss of appetite and chronic fatigue C) Coarse, dry skin and hair with decreased sweat production D) Increased white cell count and audible crackles on chest auscultation

Ans: D Feedback: An increased white cell count and the presence of adventitious fluid in the lungs are not classic findings associated with hypothyroidism. Myxedema, weight gain, lethargy, and dry skin and nails are commonly associated with low levels of thyroid hormones.

1. Which of the following statements best describes an aspect of the normal process of glucose metabolism? A) Blood glucose levels are primarily a result of the timing, quantity, and character of food intake. B) Ingested glucose that is not needed for cellular metabolism circulates in the blood until it is taken up to meet cellular needs. C) Blood glucose levels are kept in a steady state by selective excretion and reuptake by the kidneys. D) Glucose that exceeds metabolic needs is converted and stored by the liver.

Ans: D Feedback: Approximately two thirds of the glucose that is ingested with a meal is removed from the blood and stored in the liver as glycogen. Between meals, the liver releases glucose as a means of maintaining blood glucose within its normal range. Normal glucose metabolism does not involve large variations in blood glucose levels in response to food intake, and excess glucose does not normally remain in circulation. Glucose levels are not primarily maintained by the kidneys.

3. The nurse knows that a drug in a category identified as a colony-stimulating factor (CSF) helps A) cells engulf and digest microbes that want to attach to cell membranes and destroy normal cell function. B) produce cells that will be the first responder cells to protect against cancer formation. C) stimulate the person's immune system so that he or she can kill his or her own cancer cells. D) stimulate bone marrow to produce large numbers of mature cells such as platelets and erythrocytes.

Ans: D Feedback: CSFs participate in hematopoiesis by stimulating bone marrow pluripotent stem and progenitor or precursor cells to produce large numbers of mature platelets, erythrocytes, lymphocytes, neutrophils, and monocytes.

6. A 1-year-old child who has experienced low platelet counts and bacterial susceptibility has been admitted to a pediatric medical unit of a hospital for treatment of Wiskott-Aldrich syndrome. The nurse who has admitted the child to the unit would anticipate which of the following short-term and longer-term treatment plans? A) Transfusion of clotting factors XII and XIII and serum albumin; splenectomy B) Neutropenic precautions; fresh frozen plasma transfusions; treatment of gastrointestinal symptoms C) Intravenous immunoglobulin (IVIG) treatment; thyroidectomy D) Treatment of eczema; management of bleeding; bone marrow transplant

Ans: D Feedback: Common interventions for Wiskott-Aldrich syndrome involve controlling eczema, managing bleeding due to low platelets, and ultimately bone marrow transplant. The other noted interventions are not associated with the treatment of Wiskott-Aldrich syndrome.

8. Distraught at the persistent ringing in his ears and his inability to alleviate it, a 50-year-old man has visited his health care provider. After diagnostic testing, no objective cause (like impacted cerumen or vascular abnormality) was found. Given these testing results, which of the following teaching points by the care provider is most appropriate? A) ìThis is most often the result of a psychological disturbance, and therapy is often useful in relieving tinnitus.î B) ìThere are many drugs such as blood pressure pills, relaxants, heart medications, and antihistamines that can cause tinnitus.î C) ìA specialist can listen with a sensitive microphone to determine whether you are actually hearing these sounds.î D) ìThere are some treatments like tinnitus retraining therapy, which includes the extended use of low-noise generators, which has shown good success.î

Ans: D Feedback: Current treatment modalities for tinnitus address the symptoms of the problem rather than curing the underlying etiology. While therapy can be of some use, it is inaccurate to characterize tinnitus as a psychological disturbance. Medications, including antihistamines, anticonvulsant drugs, calcium channel blockers, benzodiazepines, and antidepressants, have been used for tinnitus alleviation; they are not implicated as a cause. While listening to differentiate between objective and subjective tinnitus is possible, the absence of objective sounds does not mean that tinnitus does not exist, rather that it is subjective. The use of tinnitus retraining therapy, which includes directive counseling and extended use of low-noise generators to facilitate auditory adaptation to the tinnitus, has met with considerable success.

11. A diabetes education nurse is teaching a group of recently diagnosed diabetics about the potential genitourinary complications of diabetes and the consequent importance of vigilant blood glucose control. Which of the following teaching points best conveys an aspect of bladder dysfunction and diabetes mellitus? A) ìPeople with diabetes are highly susceptible to urethral obstructions, and these can heal more slowly and cause more damage than in people without diabetes.î B) ìHigh blood sugar results in a high glucose level in your urine, and this can make your bladder muscle less able to fully empty the bladder.î C) ìMany people with diabetes find it necessary to live with an indwelling catheter to ensure their bladders do not become too full.î D) ìIt's important for you to empty your bladder frequently because diabetes carries risks of kidney damage that can be exacerbated by incomplete bladder emptying.î

Ans: D Feedback: Diabetics are vulnerable to peripheral neuropathies that can be somewhat mitigated by regular voiding; they are also especially vulnerable to renal damage from high blood sugars, a situation that is worsened when accompanied by incomplete bladder emptying. Urethral obstructions are not a noted complication of diabetes, and indwelling catheter placement is not normally necessary. High blood sugars do not necessarily yield high-glucose urine, and the bladder deficits associated with diabetes are neurological in nature rather than a result of particular urine chemistry.

7. When educating a patient with a wound that is not healing, the nurse should stress which of the following dietary modifications to ward off some of the negative manifestations that can occur with inflammation? A) Increase the amount of calcium in the diet, especially drinking milk and eating cheese. B) This is the one time whereby you should eat more fat (both polyunsaturated and saturated), so you can absorb more fat soluble vitamins. C) Since there is a loss of plasma proteins, you should increase your intake of organ meats like liver. D) Increase your intake of oily fish and fish oil so that you will increase absorption of omega-3 polyunsaturated fatty acids.

Ans: D Feedback: Dietary modification of the inflammatory response through the use of omega-3 polyunsaturated fatty acids, specifically eicosapentaenoic acid and docosahexaenoic acid, which are present in oily fish and fish oil, may be effective in preventing some negative manifestations of inflammation.

8. An endocrinologist is providing care for a 30-year-old male who has lived with the effects of increased levels of growth hormone (GH). Which of the following teaching points about the client's future health risks is most accurate? A) ìIt's not unusual for high GH levels to cause damage to your hypothalamus.î B) ìGH excess inhibits your pancreas from producing enough insulin.î C) ìThe high levels of GH that circulate in your body can result in damage to your liver.î D) ìWhen your pituitary gland is enlarged, there's a real risk that you'll develop some sight deficiencies.î

Ans: D Feedback: GH excess is associated with tumor formation and consequent compression of cranial nerves responsible for vision. Damage to the hypothalamus and liver is not common sequelae. While the beta cells of the pancreas can ìburn out,î the primary effect of excess GH is to increase insulin secretion.

15. A hospital patient has a large, superficial wound on her elbow that was the result of shearing action when she was moved up in her bed. The patient's husband mentions that the wound looks infected and irritated because the wound bed is completely red. Which of the following responses would be inappropriate? A) ìEven though it is red, it doesn't mean that the wound is infected.î B) ìThe red areas show that there is enough circulation to facilitate healing.î C) ìThose are fresh blood vessels that are a sign of healthy healing.î D) ìA thin sheet of blood clotting is actually desirable and not a sign that your wife's wound is infected.î

Ans: D Feedback: Granulation tissue indicates sufficient circulation and angiogenesis associated with healthy wound healing. Granulation tissue consists of new blood vessels, not clotted cellular components.

4. When explaining about structural classifications to a group of students, the instructor discusses the peptides and proteins. They talk about small hormones and hormones as large and complex as growth hormone (GH), which has approximately how many amino acids involved? A) 50 amino acids B) 100 amino acids C) 150 amino acids D) 200 amino acids

Ans: D Feedback: Growth hormone is a very large and complex protein that has approximately 200 amino acids.

17. A 42-year-old male has been diagnosed with renal failure secondary to diabetes mellitus and is scheduled to begin dialysis soon. Which of the following statements by the client reflects an accurate understanding of the process of hemodialysis? A) ìIt's stressful knowing that committing to dialysis means I can't qualify for a kidney transplant.î B) ìI know I'll have to go to a hospital or dialysis center for treatment.î C) ìChanging my schedule to accommodate 3 or 4 hours of hemodialysis each day will be difficult.î D) ìI won't be able to go about my normal routine during treatment.î

Ans: D Feedback: Hemodialysis requires the client to remain connected to dialysis machinery, whereas peritoneal dialysis allows for activity during treatment. Dialysis does not disqualify an individual from receiving a transplant. Dialysis does not require attendance at a dialysis center, and patients can be taught to perform the dialysis in their home with a family member in attendance. Hemodialysis is normally conducted three times weekly, not once per day.

5. A 19-year-old intravenous drug user was exposed to the HIV 3 weeks ago and is experiencing a rapid proliferation in viral load. Which of the following statements best captures an aspect of the process of HIV replication that underlies this proliferation? A) Free HIV RNA is able to attach to the cell coat of CD4+ cells. B) The cytoplasm of CD4+ cells provides a protected environment for the replication of RNA by HIV. C) Expression of reverse transcriptase by CD4+ cells allows replication of HIV cells rather than new lymphocytes. D) HIV is able to change its RNA into DNA to allow for replication by CD4+ cells.

Ans: D Feedback: In order for the HIV to reproduce, it must change its RNA into DNA. It does this by using the reverse transcriptase enzyme. Reverse transcriptase makes a copy of the viral RNA and then in reverse makes another mirror-image copy. The result is double-stranded DNA that carries instructions for viral replication. HIV RNA does not directly attach to CD4+ cells, and RNA is not replicated by HIV itself in the CD4+ cytoplasm. Reverse transcriptase is not produced by CD4+ cells, and CD4+ cells do not directly produce new lymphocytes.

13. A 24-year-old woman has gone to the OB-GYN clinic for her first visit since she found out she was pregnant. The clinician tested her blood type along with the usual prenatal testing. On a follow-up visit, the woman was told that she is Rh negative. When asked what that means for her baby, the nurse explains that Rh-negative women lack RhD antigens on their erythrocytes but produce anti-D antibodies. As a result of this blood type, A) ìIf you and your baby have mismatched blood, it can invoke anaphylaxis in the baby.î B) ìIf the types are incompatible, severe antibody-mediated inflammation occurs.î C) ìIf blood types do not match, the baby's liver will produce extra cells to replace RBCs needed to oxygenate organs.î D) ìIf the fetus is Rh positive, maternal anti-D antibodies can coat fetal RBCs resulting in severe anemia.î

Ans: D Feedback: In utero, the development of erythroblastosis fetalis (Rh incompatibility) results when Rh-negative women produce anti-D antibodies. In Rh-positive fetus, maternal anti-D antibodies will coat fetal RBCs containing RhD, allowing them to be removed from the fetal circulation by macrophage- and monocyte-mediated phagocytosis (destroying RBCs). RBCs are produced in the bone marrow, not the liver; therefore, answer C is incorrect.

2. A patient presented to the emergency department of the hospital with a swollen, reddened, painful leg wound and has been diagnosed with methicillin-resistant Staphylococcus aureus (MRSA) cellulitis. The patient's physician has ordered a complete blood count and white cell differential. Which of the following blood components would the physician most likely anticipate to be elevated? A) Basophils B) Eosinophils C) Platelets D) Neutrophils

Ans: D Feedback: Increased neutrophils are associated with inflammation in general and bacterial infections in particular. Platelets play a role in inflammation, but their levels would not rise to the same extent as would neutrophils'. Eosinophils are not strongly associated with bacterial infection, and basophils would not increase to the same degree as neutrophils.

9. A 79-year-old male resident of a long-term care facility has contracted Clostridium difficile and is experiencing consequent diarrhea. Auscultation of the man's abdomen indicates hyperactive bowel sounds. What process in the man's small intestine is most likely accompanying his current status? A) Pathogenic microorganisms are causing dilation of his small intestine, increasing motility. B) Segmentation waves have become more frequent as a result of his infection. C) Intestinal stasis brought on by infection is preventing his small intestine from sufficiently slowing the rate of motility. D) Inflammation is accompanied by an increase in peristaltic movements of his small intestine.

Ans: D Feedback: Inflammation of the small intestine is accompanied by an increase in motility, an effect that is the result of increased peristaltic waves. Segmentation waves are responsible primarily for mixing rather than moving food. Neither dilation nor inadequate slowing of passage contents is responsible for the increase in motility.

8. A 60-year-old male client with an acute viral infection is receiving interferon therapy. The physician is teaching the family of the client about the diverse actions of the treatment and the ways that it differs from other anti-infective therapies. Which of the following teaching points listed below should the physician least likely include? A) ìInterferon can help your father's unaffected cells adjacent to his infected cells produce antiviral proteins that can stop the spread of the infection.î B) ìInterferon can help limit the replication of the virus that's affecting your father.î C) ìInterferon helps your father's body recognize infected cells more quickly.î D) ìInterferon can bolster your father's immune system through the stimulation of natural killer cells that attack viruses.î

Ans: D Feedback: Interferons can activate macrophages in the fight against viral invaders, but they are not noted to stimulate the action of natural killer cells. Answers A, B, and C all capture elements of the action of interferons.

10. When educating a patient about glargine (Lantus), the nurse should explain that this medication A) has a rapid onset and peaks in about 5 minutes after injection, so he will need to eat food immediately after injection. B) will have a peak effect within 30 minutes, so it can be taken after a meal. C) is a combination with short-acting and intermediate-acting insulin, so it is safe to take anytime throughout the day. D) has a prolonged absorption rate and provides a relatively constant concentration for 12 to 24 hours.

Ans: D Feedback: Lantus is long-acting insulin that has a slow, prolonged absorption rate and provides a relatively constant concentration over 12 to 24 hours. Answer choice A relates to rapid-acting insulin; answer choice B relates to short-acting insulin; answer choice C relates to intermediate-acting insulin.

3. While reviewing the role of glucagon in regard to regulation of blood glucose, the nurse knows which of the following situations could lead to an inhibition of glucagon release? A) A sharp decrease in blood glucose concentration B) Recent strenuous physical activity C) Recent intake of large amounts of protein-rich food. D) An increase in glucose levels.

Ans: D Feedback: Low blood sugar, intake of protein, and strenuous physical activity are associated with glucagon release. Lowered cellular metabolic needs and/or increased glucose levels would inhibit glucagon release.

14. After receiving change-of-shift report about the following four patients, which patient should the nurse assess first? A) A 22-year-old admitted with SIADH who has a serum sodium level of 130 mEq/L B) A 31-year-old who has iatrogenic Cushing syndrome with a capillary blood glucose level of 204 mg/dL C) A 53-year-old who has Addison disease and is due for a scheduled dose of hydrocortisone (Solu-Cortef) D) A 70-year-old returning from PACU following partial thyroidectomy who is extremely agitated, has an irregular pulse rate of 134, and has an elevated temperature of 103.2∞F

Ans: D Feedback: Manipulation of a hyperactive thyroid gland during thyroidectomy can cause thyroid storm. It is manifested by very high fever, extreme cardiovascular effects (tachycardia, HF, angina), and severe CNS effects (agitation, restlessness, and delirium). Answer choice A refers to normal sodium levels. Answer choice B refers to high blood glucose level but not critical level. Answer choice C refers to lower priority. It is always preferred to give medications in a timely manner; however, thyroid storm signs and symptoms are the priority for this group of patients.

18. A 30-year-old woman has presented to her family doctor complaining of three distressing episodes over the last several months during which she got extremely dizzy, had loud ringing in her ears, and felt like her ears were full of fluid. She states that her hearing diminishes, and she feels nauseous during these episodes. What diagnosis is the physician most likely to first suspect? A) Acute otitis media B) Acute vestibular neuronitis C) Benign paroxysmal positional vertigo (BPPV) D) MÈniËre disease

Ans: D Feedback: MÈniËre disease is characterized by fluctuating episodes of tinnitus, feelings of ear fullness, and violent rotary vertigo that often renders the person unable to sit or walk. There is a need to lie quietly with the head fixed in a comfortable position, avoiding all head movements that aggravate the vertigo. Symptoms referable to the autonomic nervous system, including pallor, sweating, nausea, and vomiting, usually are present. The more severe the attack, the more prominent are the autonomic manifestations. A fluctuating hearing loss occurs with a return to normal after the episode subsides. Her symptomatology is not characteristic of AOM, acute vestibular neuronitis, or BPPV.

14. A 77-year-old woman has been admitted to hospital following several weeks of increasing fatigue. On observation, she is pale, and blood work indicates she has low hemoglobin and red cell counts. Stool tests for occult blood are positive, and following endoscopy, she has been diagnosed with an upper GI bleed that has been shown to originate in her stomach. She admits to regularly exceeding the recommended doses of nonsteroidal anti-inflammatory drugs (NSAIDs) in an effort to control her rheumatoid arthritis. Which of the following phenomena is most likely responsible for her present health problems? A) NSAIDs increase the gastric production of gastrin, increasing gastric secretions and lowering stomach pH. B) Drugs such as NSAIDs increase the H+ levels and thus decrease gastric pH, resulting in insult to the stomach lining. C) NSAIDs, aspirin, and other drugs increase prostaglandin synthesis, resulting in disruption of cellular structures lining the stomach. D) NSAIDs can disrupt the permeability of the gastric mucosa, causing hydrogen ions to accumulate in the mucosal cells of the lining.

Ans: D Feedback: NSAIDs damage the mucosal barrier, allowing hydrogen ions to damage cells of the stomach lining. They do not directly increase gastrin production or H+ levels, and decreased, not increased, prostaglandin synthesis would potentially compromise the gastric surfaces.

5. Which of the following would be considered an abnormal finding when the nurse practitioner uses an otoscope to look at a toddler's ear? The tympanic membrane is described as A) ìtransparent.î B) ìa shallow, oval cone pointing inward toward apex.î C) ìsmall, whitish cord seen traversing the middle ear from back to front.î D) ìyellow, amber discoloration noted.î

Ans: D Feedback: Otoscopic signs of middle ear effusion will be seen in the tympanic membrane as a yellow, amber discoloration. All of the other distracters are normal findings.

19. Which of the following situations can best be characterized as an example of passive immunity? A) A 6-month-old infant receives his scheduled immunization against measles, mumps, and rubella. B) A 9-year-old boy is immune to chicken pox after enduring the infection before 1 year. C) An 8-year-old girl recovers from a respiratory infection after intravenous antibiotic treatment. D) A 6-week-old infant receives antibodies from his mother's breast milk.

Ans: D Feedback: Passive immunity involves the transfer of antibodies from an outside source, such as those from breast milk. Immunization and recovery from illness involve active immunity.

8. A 46-year-old man who is obese has received news that he has type 2 diabetes. He is in the process of determining a plan of care with an interdisciplinary team at a hospital-based diabetes clinic. The nurse knows that the most likely treatment plan for the man will include A) injectable insulin and nutrition management. B) weight loss, glucose monitoring, and lifestyle modification. C) continuous subcutaneous insulin infusion (CSII) and nutrition management. D) oral antihyperglycemic medications and weight loss measures.

Ans: D Feedback: Persons with type 2 diabetes would unlikely require insulin initially, and oral medications are likely to be of benefit as an addition to lifestyle modifications and weight loss.

6. After several months of persistent heartburn, a 57-year-old female client has been diagnosed with gastroesophageal reflux disease (GERD). Which of the following treatment regimens is likely to best address the woman's health problem? A) Surgical correction of the incompetent pylorus B) Antacids; avoiding positions that exacerbate reflux; a soft-textured diet C) Weight loss and administration of calcium channel blocking medications D) Proton pump inhibitors; avoiding large meals; remaining upright after meals

Ans: D Feedback: Proton pump inhibitors block the final stage of gastric acid production, effectively controlling the root cause of the esophageal damage associated with GERD. The pylorus is not involved, and a soft diet is not indicated. Calcium channelñblocking drugs would not address the problem. Calcium channel blockers are primarily heart disease drugs that relax blood vessels and increase the supply of blood and oxygen to the heart while also reducing the heart's workload.

4. A 48-year-old woman has been diagnosed with extrahepatic cholestasis following a thorough history, ultrasound, and blood work. Which of the following symptoms most likely caused her to seek medical treatment, and what consequence to her health problem would the medical team anticipate? A) Complaints of lower flank pain with consequences of impaired fat metabolism B) Anorexia with consequences of impaired drug metabolism and blood filtration C) Skin xanthomas (focal accumulations of cholesterol) with consequences of increased risk of bleeding due to deficient clotting factors D) Pruritus with consequences of deficient levels of fat-soluble vitamins

Ans: D Feedback: Pruritus is the most common symptom of cholestasis, and deficiencies in fat-soluble vitamins such as A, D, and K are frequent. Flank pain is not a noted complaint, and bile does not contribute to drug metabolism, blood filtration, or clotting factor production.

12. A 40-year-old woman who experiences severe seasonal allergies has been referred by her family physician to an allergist for weekly allergy injections. The woman is confused as to why repeated exposure to substances that set off her allergies would ultimately benefit her. Which of the following phenomena best captures the rationale for allergy desensitization therapy? A) Repeated exposure to offending allergens binds the basophils and mast cells that mediate the allergic response. B) Allergens in large, regular quantities overwhelm the IgE antibodies that mediate the allergic response. C) Repeated exposure stimulates adrenal production of epinephrine, mitigating the allergic response. D) Injections of allergens simulate production of IgG, combining with the antigens to prevent activation of IgE antibodies.

Ans: D Feedback: Repeated exposure to allergens causes an increase in IgG, which binds with antigens before they can stimulate IgE. It does not bind mast cells or basophils, nor does it overwhelm the IgE antibodies to stimulate epinephrine production.

17. A 37-year-old male with HIV who has recently become symptomatic has begun highly active antiretroviral therapy (HAART). Among the numerous medications that the man now regularly takes are several that inhibit the change of HIV RNA to DNA in a CD4+ cell. Which of the following classes of medications addresses this component of the HIV replication cycle? A) Entry inhibitors B) Protease inhibitors C) Integrase inhibitors D) Non-nucleoside reverse transcriptase inhibitors

Ans: D Feedback: Reverse transcriptase inhibitors inhibit HIV replication by acting on the enzyme reverse transcriptase. Non-nucleotide reverse transcriptase inhibitors block the copying of RNA into DNA. Entry inhibitors, protease inhibitors, and integrase inhibitors do not address this aspect of the HIV replication cycle.

6. A client has been identified as having an excess of macrophage inhibitory factor, causing the client to have inhibited movement and activity of macrophages. Which of the following processes listed below would the health care team member expect to remain unaffected? A) Amplification of the immune response B) Destruction of virus-infected or tumor cells C) Initiation of adaptive immunity D) Specificity and memory of the immune response

Ans: D Feedback: Specificity and memory are the defining characteristics of the adaptive immune system, and macrophages do not perform this particular role. Amplification of the immune response, destruction of virus-infected or tumor cells, and initiation of adaptive immunity are all components of macrophage activity.

19. A 4-month-old infant and his mother are at an appointment with a pediatrician to follow up his nonaccommodative strabismus and to determine a treatment plan. Which of the following treatments is most likely to prevent future loss of vision? A) Prescribing glasses once the infant is 6 months of age B) Use of beta-adrenergic blockers and latanoprost eye drops C) Regularly scheduled eye exams and monitoring of self-correction of his eyes D) Surgical correction of the musculature

Ans: D Feedback: Strabismus is ideally treated by surgery rather than with glasses in the case of infants. The condition will not resolve independently, and medications are unlikely to help the condition.

3. A 16-year-old girl has broken her arm while snowboarding. When she arrives at hospital, she is shocked at the amount of swelling at the injury site. Which of the following statements best explains the physiological rationale for her swelling? A) Migration and proliferation of mast cells, neutrophils, and platelets to the injury site occupy an increased volume of tissue. B) Potent vasodilation increases the total volume of vascular space at the site of inflammation. C) Osmotic flow of plasma into the intravascular space causes increased blood volume and interstitial fluid. D) Loss of plasma proteins causes an increase in interstitial osmotic pressure.

Ans: D Feedback: Swelling is the result of plasma proteins leaving the interstitial space, resulting in increased osmotic pressure of interstitial fluid and movement of fluid into tissues. Blood components, vasodilation, nor increased intravascular volume accounts for swelling.

8. A 46-year-old male has presented to the emergency department because of the eye pain, severe headache, and blurred vision that have followed an eye exam at an optometrist's office earlier in the day. The client tells the triage nurse that he received eye drops during the exam ìto keep my pupils wide open.î What differential diagnosis will the care team first suspect? A) Infectious conjunctivitis B) Keratitis C) Corneal trauma D) Angle-closure glaucoma

Ans: D Feedback: Symptoms of acute angle-closure glaucoma are related to sudden, intermittent increases in intraocular pressure. These occur after prolonged periods in the dark, emotional upset, and other conditions that cause extensive and prolonged dilation of the pupil. Administration of pharmacologic agents such as atropine that cause pupillary dilation (mydriasis) also can precipitate an acute episode of increased intraocular pressure in persons with the potential for angle-closure glaucoma. Attacks of increased intraocular pressure are manifested by ocular pain and blurred or iridescent vision. The man's symptomatology is not characteristic of conjunctivitis, corneal trauma, or keratitis.

10. Which of the following statements best communicates the nature of movements in the colon? A) The internal and external anal sphincters control the movement of feces into and through the colon. B) Haustrations move the colon contents along with 2- to 3-minute rest periods between movements. C) Segmentation waves ensure that all surfaces of the feces are exposed to the colon surface. D) Large segments of the colon contract as a unit for around 30 seconds.

Ans: D Feedback: The colon, unlike the small intestine, moves contents by the simultaneous contraction of large segments. The anal sphincters control defecation rather than movements within the colon, and haustrations perform mixing, not motility. Segmentation waves are present in the small intestine.

5. When explaining a cystometry test to measure bladder pressure during filling and voiding in a normal adult, the nurse informs the nursing students that the normal capacity when adults have a desire to void is A) 100 to 150 mL. B) 200 to 250 mL. C) 300 to 399 mL. D) 400 to 500 mL.

Ans: D Feedback: The desire to void occurs when the bladder is full (normal capacity is approximately 400 to 500 mL). At this point, a definite sensation of fullness occurs; the pressure rises sharply to 40 to 100 cm H2O; and voiding occurs around the catheter.

15. A 20-year-old woman has visited her family physician due to occasional bouts of bloody diarrhea over the past several weeks, a phenomenon that she experienced 2 years prior as well. Her physician has diagnosed her with ulcerative colitis based on her history and visualization of the affected region by colonoscopy and sigmoidoscopy. Which of the following pathophysiological phenomena is most likely to underlie the client's health problem? A) Fissures and crevices developing in the mucosa that are seen as a characteristic ìcobblestoneî appearance B) Erosion of the endothelial lining of the distal small intestine by a combination of genetic, autoimmune, and environmental factors C) Compromise of the mucosal layer of the large intestinal surface by the effects of H. pylori D) Mucosal hemorrhages that have developed into crypt abscesses, which have in turn necrotized and ulcerated

Ans: D Feedback: The etiology and course of ulcerative colitis involves mucosal hemorrhages developing into crypt abscesses, with consequent necrosis and ulceration. ìCobblestoneî appearance of intestinal mucosa is associated with Crohn disease. Ulcerative colitis is confined to the colon and rectum, and H. pylori is not commonly implicated in the etiology.

3. During descent, an airplane passenger is complaining that his ìears are plugged.î What aspect of the structure and function of the ear best accounts for the passenger's complaint? A) The inner ear adjusts its volume in response to atmospheric pressure, increasing during low pressure and decreasing in high pressure. B) The eustachian tubes must remain patent to equalize pressure between the middle ear and inner ear. C) The tympanic membrane is selectively permeable in order to accommodate pressure changes, and this capacity is often impaired during upper respiratory infections. D) Air must be able to flow between the middle ear and nasopharynx in order to accommodate pressure changes.

Ans: D Feedback: The eustachian tubes between the middle ear and nasopharynx must be patent to allow for changes in atmospheric pressure. Pressure is not accommodated by changing the volume of the middle ear, and the tympanic membrane is not selectively permeable to air.

13. A 51-year-old male professional is in the habit of consuming six to eight rum and cokes each evening after work. He assures the nurse practitioner who is performing his regular physical exam that his drinking is under control and does not have negative implications for his work or family life. How could the nurse best respond to the client's statement? A) ìYou are more than likely inflicting damage on your liver, but this damage would cease as soon as you quit drinking.î B) ìThat may be the case, but you are still creating a high risk of hepatitis A or B or liver cancer.î C) ìIn spite of that, the amount of alcohol you are drinking is likely to result first in cirrhosis and, if you continue, in hepatitis or fatty liver changes.î D) ìWhen your body has to regularly break down that much alcohol, your blood and the functional cells in your liver accumulate a lot of potentially damaging toxic byproducts.î

Ans: D Feedback: The hepatic effects of alcohol use are related to the accumulation of toxic metabolites in the hepatocytes and blood. Damage can continue even after an individual stops drinking. Specific consequences do not usually include HAV, HBV, or liver cancer. Cirrhosis represents the culmination, not the beginning, of negative hepatic effects.

15. Which of the following statements best captures the relationship between the hypothalamus and the pituitary gland as it relates to endocrine function? A) The hypothalamus directly measures the levels of most hormones throughout the body and inhibits or stimulates the pituitary accordingly. B) The pituitary gland coordinates and dictates the release of hormones from the hypothalamus that act on their intended target cells. C) The pituitary gland and hypothalamus have two-way communication that mediates the signals from neuronal inputs. D) The hypothalamus receives input from numerous sources throughout the body and directs the pituitary to then control many target glands and cells.

Ans: D Feedback: The hypothalamus can be viewed as a bridge by which signals from multiple systems are relayed to the pituitary gland. The hypothalamus collects data from sources throughout the body rather than directly measuring levels, and communication normally flows from the hypothalamus to the pituitary.

12. A 53-year-old female hospital patient has received a kidney transplant following renal failure secondary to hypertension. As part of the teaching while she was on the organ wait list, she was made aware that she would need to take antirejection drugs for the rest of her life. Which aspect of the immune system underlies this necessity? A) The lack of identifiable major histocompatibility complex (MHC) molecules will stimulate the innate immune response. B) Donor organ antibodies will be identified as foreign and stimulate an immune response. C) Antirejection drugs will stimulate the production of familiar MHC molecules. D) MHC molecules will never develop in the cells of the donor organ, and effector cells will be continually stimulated.

Ans: D Feedback: The lack of familiar MHC molecules will stimulate an immune response by effector cells in the absence of antirejection drugs. An innate immune response is not central to the response, but rather the adaptive immune system. Lack of known MHC molecules, not foreign antibodies, accounts for the immune response, and familiar MHC molecules will not be produced by the donor kidney cells.

8. A person who has been diagnosed with HIV infection 12 years ago and still has a CD4+ cell count of 800 cells/µL and a low viral load is considered clinical to be a A) rapid progressor. B) typical progressor. C) slow progressor. D) long-term nonprogressor.

Ans: D Feedback: There is a subset of slow progressors: the long-term nonprogressors, who account for 1% of all HIV infections. These people have been infected for at least 8 years, are antiretroviral naive, have high CD4+ cell counts, and usually have very low viral loads. They are being investigated to determine how they maintain viral suppression of HIV.

11. Members of an AIDS support group who have more advanced cases are sharing some of their recent health problems with a member who has just been diagnosed. Which of the member's statements is most accurate? A) ìOne of the scariest things out there now is the huge increase in drug-resistant tuberculosis.î B) ìThe eradication of Pneumocystis jiroveci pneumonia (PCP) has helped extend the life expectancy of a lot of persons living with AIDS.î C) ìThose of us with HIV are so much more prone to loss of vision and hearing.î D) ìAs people with HIV live longer, most of us are eventually succumbing to the cancers that are associated with HIV.î

Ans: D Feedback: There is an increased risk of AIDS-associated cancers as persons with age live longer. Drug resistance in tuberculosis is on the decline in recent years, and PCP has not been eradicated. Sensory loss is not a noted HIV-related manifestation.

12. A 24-year-old woman undergoing a premarital screening test is found to have elevated levels of AST, ALT, and IgG, but no antibody-specific markers for viral hepatitis. A liver biopsy reveals inflammation and cellular damage. Which of the following treatments is most likely to be effective for her? A) Lamivudine B) Peginterferon and ribavirin C) Interferon alfa-2b D) Corticosteroids and immunosuppressant drugs

Ans: D Feedback: This woman's hepatitis is probably caused by an autoimmune disorder rather than a virus. Lamivudine, peginterferon/ribavirin, and interferon alfa-2b are all antiviral agents. Autoimmune hepatitis is a chronic, usually progressive, inflammatory disease of the liver. Corticosteroids are the mainstay of treatment and have been shown to produce remission. Azathioprine, an immunosuppressant medication, is sometimes used along with prednisone. Using both medications may allow you to take a smaller dose of prednisone, reducing its side effects. Most people with this disease require long-term maintenance treatment.

10. A client with a history of an endocrine disorder exhibits signs and symptoms of hormone deficiency. Which of the following processes would the client's care team most likely rule out first as a contributing factor? A) The client's target cells lack sufficient receptors for the hormone in question. B) Hormone production is sufficient, but affinity on the part of the target cells is lacking. C) The process of down-regulation has resulted in decreased hormone sensitivity. D) Up-regulation has increased the sensitivity of the body to particular hormone levels.

Ans: D Feedback: Up-regulation is a response to low hormone levels in which the number of receptors increases. As such, it would not likely result in signs and symptoms of deficiency but is rather a compensatory mechanism that counters a deficiency. Insufficient numbers of receptors, low affinity, and down-regulation could all contribute to signs and symptoms of a hormone deficiency.

3. A 32-year-old man is complaining of burning, itching, photophobia, and severe pain in his right eye after swimming in the ocean. To determine that the eye condition was a corneal rather than a conjunctival disease, which of the following would be the distinguishing symptom? A) Burning B) Itching C) Photophobia D) Severe pain

Ans: D Feedback: While burning, itching, and photophobia are all important symptoms of conjunctivitis, severe pain suggests corneal rather than conjunctival disease.

14. A stroke affecting which of the following areas of the brain would be most likely to leave an individual's vestibular system intact and posture and balance maintained? A) The brain stem B) The thalamus C) The temporal and parietal cortex D) The limbic system of the cerebrum

Ans: D Feedback: While the brain stem, thalamus, and temporal and parietal cortex are components of the vestibular apparatus, the limbic system is not a central component of the maintenance of posture and balance.

9. Which of the following individuals is at the highest risk of developing a urinary tract infection (UTI)? A) A 60-year-old man with a history of cardiovascular disease who is recovering in hospital from a coronary artery bypass graft B) A 66-year-old man undergoing dialysis for the treatment of chronic renal failure secondary to hypertension C) A 38-year-old man with high urine output due to antidiuretic hormone insufficiency D) A 30-year-old woman with poorly controlled diabetes mellitus

Ans: D Feedback: Young women as well as persons with diabetes are at high risk of UTIs. Neither postsurgical recovery nor renal failure is necessarily a direct risk for UTI development, and high urine output would prevent decrease rather than increase in UTI risk.

2. Place the following components of the gastrointestinal tract in the chronological order that a bolus of food would pass through them. Use all the options. A) Ileum B) Pylorus C) Jejunum D) Hiatus E) Cecum F) Duodenum

Ans: D, B, F, C, A, E Feedback: Ingested food and fluids enter the stomach through the hiatus, exit through the pylorus, and pass through the three subdivisions of the small intestine: the duodenum, jejunum, and ileum. The cecum is a component of the large bowel.


Related study sets

Law Chapter 6, Law chapter 6 Criminal law and business

View Set

Day 1 P.6-8 - Author's Purpose (Activity)

View Set